851
Pass Program Pass Program Clues Clues You ain’t told me squat You ain’t told me squat till you tell me the till you tell me the CLUE!!!” CLUE!!!”

USMLE Step 1 clues 2- 800 slides

Embed Size (px)

DESCRIPTION

 

Citation preview

Page 1: USMLE Step 1 clues 2- 800 slides

Pass ProgramPass Program CluesClues

““You ain’t told me squat till You ain’t told me squat till you tell me the CLUE!!!”you tell me the CLUE!!!”

Page 2: USMLE Step 1 clues 2- 800 slides

What are the 5 bacteria What are the 5 bacteria causing Heart Block?causing Heart Block?

• Lyme DiseaseLyme Disease

• Salmonella typhii (typhoid)Salmonella typhii (typhoid)

• Chagas Disease (Whipple’s)Chagas Disease (Whipple’s)

• LegionellaLegionella

• DiptheriaDiptheria

• Lets Stop Doing Long ContractionsLets Stop Doing Long Contractions

Page 3: USMLE Step 1 clues 2- 800 slides

What bacteria cause What bacteria cause Reiter’s Syndrome?Reiter’s Syndrome?• ShigellaShigella• IBDIBD

– Crohn’sCrohn’s

• ChlamydiaChlamydia• YersinaYersina

• Reiter & Crohn Saw Yersina and got Reiter & Crohn Saw Yersina and got Chlamyia Chlamyia

Page 4: USMLE Step 1 clues 2- 800 slides

What are the Low Complement What are the Low Complement bugs causing bugs causing Cryoglobuniemia?Cryoglobuniemia?• InfluenzaeInfluenzae

• AdenovirusAdenovirus

• MycoplasmaMycoplasma

• Hepatitis CHepatitis C

• EBVEBV

• I AM HEI AM HE

Page 5: USMLE Step 1 clues 2- 800 slides

What are the drugs induced What are the drugs induced SLE?SLE?• HydralazineHydralazine

• INHINH

• PhenytoinPhenytoin

• ProcainamideProcainamide

• PenicillaminePenicillamine

• EthosuximideEthosuximide

• H I PPPEH I PPPE

Page 6: USMLE Step 1 clues 2- 800 slides

What are the drugs that What are the drugs that Blast the BM?Blast the BM?

• AZTAZT

• BenzeneBenzene

• ChloramphenicolChloramphenicol

• VinblastineVinblastine

• Vinblastine Anilate Bone CellsVinblastine Anilate Bone Cells

Page 7: USMLE Step 1 clues 2- 800 slides

What are the Comma What are the Comma Shaped bugs?Shaped bugs?

• VibrioVibrio

• CampylobacterCampylobacter

• ListeriaListeria

• H. pyloriH. pylori

• Campylobacter Has Very Long Campylobacter Has Very Long Comma GenesComma Genes

Page 8: USMLE Step 1 clues 2- 800 slides

What is the cresent shaped What is the cresent shaped protozoa?protozoa?

• Giardia lambliaGiardia lamblia

Page 9: USMLE Step 1 clues 2- 800 slides

What bacteria looks like What bacteria looks like Chinese letters?Chinese letters?

• CorynebacterCorynebacter

Page 10: USMLE Step 1 clues 2- 800 slides

What are the TB Rx?What are the TB Rx?

• RifampinRifampin

• PyrazinamidePyrazinamide

• INHINH

• EthambutanolEthambutanol

• StreptomycinStreptomycin

• R E S P IR E S P I

Page 11: USMLE Step 1 clues 2- 800 slides

What are the 6 Low Complement What are the 6 Low Complement assocs. with Nephrotic Syndrome?assocs. with Nephrotic Syndrome?

• Serum SicknessSerum Sickness

• PSGNPSGN

• SLESLE

• SBESBE

• CryoglobinemiaCryoglobinemia

• MPGN IIMPGN II

Page 12: USMLE Step 1 clues 2- 800 slides

What drugs Induce p450?What drugs Induce p450?

• BAG 4 CPR QTSBAG 4 CPR QTS

• Car Grabs Queens Car Grabs Queens Tets to Rev Up Tets to Rev Up

• Alcoholic doing drugs Alcoholic doing drugs and stinking up carand stinking up car

• BarbituratesBarbiturates

• AlcoholAlcohol

• GriseofulvinGriseofulvin

• CarbamazapineCarbamazapine

• RifampinRifampin

• QuinidineQuinidine

• TetracyclineTetracycline

• Sulfa drugsSulfa drugs

Page 13: USMLE Step 1 clues 2- 800 slides

What drugs Inhibit p450?What drugs Inhibit p450?

• I Do SMACK I Do SMACK QuinolonesQuinolones

• INHINH

• DapsoneDapsone

• SpirolactonesSpirolactones

• MacrolidesMacrolides

• AmiodaroneAmiodarone

• CimetidineCimetidine

• KetoconazoleKetoconazole

• QuinilonesQuinilones

Page 14: USMLE Step 1 clues 2- 800 slides

What drugs are P450 What drugs are P450 Dependent?Dependent?• WarfarinWarfarin

• EstrogenEstrogen

• PhenytoinPhenytoin

• TheophyllineTheophylline

• DigoxinDigoxin

• Theo came from war & dig inside WDEPT Theo came from war & dig inside WDEPT taking Estrogen & now is Phenytointaking Estrogen & now is Phenytoin

Page 15: USMLE Step 1 clues 2- 800 slides

What disease is a Neutophil What disease is a Neutophil Deficiency?Deficiency?

• CGDCGD

Page 16: USMLE Step 1 clues 2- 800 slides

What is another name for What is another name for CGD?CGD?

• Chronic Granulomatous diseaseChronic Granulomatous disease

• NADPH Oxidase deficiency NADPH Oxidase deficiency

Page 17: USMLE Step 1 clues 2- 800 slides

What are the Side effects of What are the Side effects of Statins?Statins?

• MyositisMyositis

• HepatitisHepatitis

• Increased liver enzymesIncreased liver enzymes

Page 18: USMLE Step 1 clues 2- 800 slides

What are the painful genital What are the painful genital Lesions?Lesions?

• ChancroidChancroid

• HerpesHerpes

• Lymphogranuloma inguinaleLymphogranuloma inguinale

Page 19: USMLE Step 1 clues 2- 800 slides

What is the painful What is the painful chancroid lesion due to?chancroid lesion due to?

• Hemophilus ducreyiHemophilus ducreyi

Page 20: USMLE Step 1 clues 2- 800 slides

What are the 4 hormones with What are the 4 hormones with disulfide bonds?disulfide bonds?

• Prolactin Prolactin

• InsulinInsulin

• InhibinInhibin

• GHGH

• I PIG on BONDSI PIG on BONDS

Page 21: USMLE Step 1 clues 2- 800 slides

What are the Hookworms?What are the Hookworms?

• Necatur americanisNecatur americanis

• Enterobius vermicularisEnterobius vermicularis

• Ankylostoma duodenaleAnkylostoma duodenale

• Trichuris trichuriumTrichuris trichurium

• Ascaris lumbercoidesAscaris lumbercoides

• StrongyloidesStrongyloides

• Hooks AS NEATHooks AS NEAT

Page 22: USMLE Step 1 clues 2- 800 slides

What are the X-Linked What are the X-Linked enzyme Deficiencies?enzyme Deficiencies?• G6-PDG6-PD

• CGD CGD

• Pyruvate dehydrogenase Def.Pyruvate dehydrogenase Def.

• Fabry’sFabry’s

• Hunter’sHunter’s

• Lesch-nyhanLesch-nyhan

• Lesch-Nyhan Hunter Puts Fabrys on G6 Lesch-Nyhan Hunter Puts Fabrys on G6 ClothesClothes

Page 23: USMLE Step 1 clues 2- 800 slides

What diseases do we screen What diseases do we screen for at birth? for at birth? • PleasePlease

• CheckCheck

• BeforeBefore

• GoingGoing

• HomeHome

• PKUPKU

• CAH(Congential CAH(Congential Adrenal Adrenal Hyperplasia)Hyperplasia)

• BiotinidaseBiotinidase

• GalactosemiaGalactosemia

• HypothyroidismHypothyroidism

Page 24: USMLE Step 1 clues 2- 800 slides

HLA-AntigensHLA-Antigens

• HLA-DR2HLA-DR2= = Narcolepsy, Allergy, Narcolepsy, Allergy, Goodpasture’s, MSGoodpasture’s, MS

• HLA-DR3HLA-DR3= = DM, Chronic Active Hepatitis, DM, Chronic Active Hepatitis, Sjogren’s, SLE, Celiac SprueSjogren’s, SLE, Celiac Sprue

• HLA-DR3 & 4HLA-DR3 & 4= = IDDM(Type I)IDDM(Type I)• HLA-DR4HLA-DR4= = Rheumatoid Arthritis, Pemphigus Rheumatoid Arthritis, Pemphigus

VulgarisVulgaris• HLA-DR5HLA-DR5= = JRA, Pernicious AnemiaJRA, Pernicious Anemia• HLA-DR7HLA-DR7= Nephrotic Syndrome(Steroid = Nephrotic Syndrome(Steroid

induced)induced)

Page 25: USMLE Step 1 clues 2- 800 slides

HLA-AntigensHLA-Antigens

• HLA-DR 3 & B8=HLA-DR 3 & B8=Celiac DiseaseCeliac Disease• HLA-A3= HLA-A3= Hemochromatosis(chromo. 6, Hemochromatosis(chromo. 6,

point mut.-cysteine>tyrosine)point mut.-cysteine>tyrosine)• HLA-B8=HLA-B8=MGMG• HLA-B13= PsoriasisHLA-B13= Psoriasis• HLA-B27= Psoriais(only if w/arthritis) HLA-B27= Psoriais(only if w/arthritis)

Ankylosing Spondylitis, IBD, Reiter’s, Ankylosing Spondylitis, IBD, Reiter’s, Postgonococcal ArthritisPostgonococcal Arthritis

• HLA-BW 47= 21 alpha Hydroxylas def.HLA-BW 47= 21 alpha Hydroxylas def.(Vit.D)(Vit.D)

Page 26: USMLE Step 1 clues 2- 800 slides

What are the actions of What are the actions of Steroids?Steroids?

• Kills helper T-cells & eosinophilsKills helper T-cells & eosinophils

• Inhibits Macrophage migrationInhibits Macrophage migration

• Inhibits Mast cell degranulationInhibits Mast cell degranulation

• Inhibits Phospholipase AInhibits Phospholipase A

• Stimulates protein synthesisStimulates protein synthesis

• Stablizes endotheliumStablizes endothelium

Page 27: USMLE Step 1 clues 2- 800 slides

What are the causes of What are the causes of Monocytosis? Monocytosis?

• Salmonella (typhoid)Salmonella (typhoid)

• TBTB

• EBVEBV

• ListeriaListeria

• SyphillisSyphillis

Page 28: USMLE Step 1 clues 2- 800 slides

E. Coli is the most common E. Coli is the most common cause of what?cause of what?

• UTIUTI

• Spontaneous bacterial peritonitis Spontaneous bacterial peritonitis

• Abdominal abscessAbdominal abscess

• CholecystitisCholecystitis

• Ascending cholangitisAscending cholangitis

• AppendicitisAppendicitis

Page 29: USMLE Step 1 clues 2- 800 slides

What are the one dose What are the one dose treatments for Gonorrhea?treatments for Gonorrhea?

• CeftriaxoneCeftriaxone

• CefiximeCefixime

• CefoxineCefoxine

• CiprofloxinCiprofloxin

• OflaxacinOflaxacin

• GatifolxacinGatifolxacin

Page 30: USMLE Step 1 clues 2- 800 slides

What is the one dose What is the one dose treatment for Chlamydia?treatment for Chlamydia?

• AzithromycinAzithromycin

Page 31: USMLE Step 1 clues 2- 800 slides

What are the “Big Mama” What are the “Big Mama” anaerobes?anaerobes?

• Strep bovisStrep bovis

• Clostridium melanogosepticusClostridium melanogosepticus

• Bacteriodes fragilisBacteriodes fragilis

Page 32: USMLE Step 1 clues 2- 800 slides

What are the “Big Mama” What are the “Big Mama” Rx?Rx?

• ClindamycinClindamycin

• MetranidazoleMetranidazole

• CefoxitinCefoxitin

Page 33: USMLE Step 1 clues 2- 800 slides

What “big mama” bugs are What “big mama” bugs are associated with colon cancer?associated with colon cancer?

• Strep. BovisStrep. Bovis

• Clostridium melanogosepticusClostridium melanogosepticus

Page 34: USMLE Step 1 clues 2- 800 slides

What do you see in the serum What do you see in the serum with low volume state?with low volume state?• K+?K+?• DecreasesDecreases

• Na+?Na+?• DecreasesDecreases

• Cl-?Cl-?

• DecreasesDecreases

• pH?pH?• IncreasesIncreases

• BP?BP?• IncreasesIncreases

Page 35: USMLE Step 1 clues 2- 800 slides

What are psammoma What are psammoma bodies?bodies?

• Calcified CA’sCalcified CA’s

Page 36: USMLE Step 1 clues 2- 800 slides

In what diseases are In what diseases are Psammoa Bodies present?Psammoa Bodies present?

• Papillary carcinoma of the ThyroidPapillary carcinoma of the Thyroid

• Serous cystadenocarcinoma of the Serous cystadenocarcinoma of the ovaryovary

• MeningiomaMeningioma

• MesotheliomaMesothelioma

Page 37: USMLE Step 1 clues 2- 800 slides

What are the Urease (+) What are the Urease (+) Bacteria?Bacteria?

• ProteusProteus

• PseudomonasPseudomonas

• Ureaplasma urealyticumUreaplasma urealyticum

• Nocardia speciesNocardia species

• Cryptococcus neoformansCryptococcus neoformans

• H. pyloriH. pylori

Page 38: USMLE Step 1 clues 2- 800 slides

What types of stones are What types of stones are formed from Proteus?formed from Proteus?

• Struvite (90%)Struvite (90%)

Page 39: USMLE Step 1 clues 2- 800 slides

What type of motility do What type of motility do Proteus have?Proteus have?

• swarmingswarming

Page 40: USMLE Step 1 clues 2- 800 slides

What are 5 indications of What are 5 indications of Surgery?Surgery?

• Intractable painIntractable pain

• Hemorrhage (massive)Hemorrhage (massive)

• Obstruction (from scarring)Obstruction (from scarring)

• PerforationPerforation

Page 41: USMLE Step 1 clues 2- 800 slides

What drugs cause Cardiac What drugs cause Cardiac Fibrosis?Fibrosis?

• Adriamycin (Doxyrubicin)Adriamycin (Doxyrubicin)

• Phen-fenPhen-fen

Page 42: USMLE Step 1 clues 2- 800 slides

What drug is used to tx What drug is used to tx cardiac fibrosis?cardiac fibrosis?

• DozaroxsinDozaroxsin

Page 43: USMLE Step 1 clues 2- 800 slides

What is the MCC of any What is the MCC of any ….penia?….penia?

• #1 = Virus#1 = Virus

• #2 = Drugs#2 = Drugs

Page 44: USMLE Step 1 clues 2- 800 slides

What is seen in the What is seen in the Salmonella Triad?Salmonella Triad?

• High FeverHigh Fever

• Rose spots (rash)Rose spots (rash)

• Intestinal fireIntestinal fire

Page 45: USMLE Step 1 clues 2- 800 slides

What drugs cause Myositis?What drugs cause Myositis?

• RifampinRifampin

• INHINH

• PredinsonePredinsone

• StatinsStatins

Page 46: USMLE Step 1 clues 2- 800 slides

What are the 7 Gram -What are the 7 Gram -encapsulated bacteria?encapsulated bacteria?• SomeSome

• StrangeStrange

• KillersKillers

• HaveHave

• Pretty Pretty

• NiceNice

• CapsulesCapsules

• SalmonellaSalmonella

• Strep. Pneumo Strep. Pneumo (gr+)(gr+)

• KlebsiellaKlebsiella

• H. influenzaH. influenza

• PseudomonasPseudomonas

• NeisseriaNeisseria

• CryptococcusCryptococcus

Page 47: USMLE Step 1 clues 2- 800 slides

What is the Jones Criteria for What is the Jones Criteria for Rheumatic Fever?Rheumatic Fever?

• SubQ nodulesSubQ nodules

• PolyarthritisPolyarthritis

• Erythema marginatumErythema marginatum

• CarditisCarditis

• ChoreaChorea

Page 48: USMLE Step 1 clues 2- 800 slides

What are the causes of What are the causes of Eosinophilla? Eosinophilla?

• NeoplasmsNeoplasms

• Allergies/AsthmaAllergies/Asthma

• Addison’s DzAddison’s Dz

• Collagen Vascular DzCollagen Vascular Dz

• ParasitesParasites

Page 49: USMLE Step 1 clues 2- 800 slides

What are the Risk Factors What are the Risk Factors for Liver CA?for Liver CA?• Hep B,C,DHep B,C,D• AflatoxinAflatoxin• Vinyl chlorideVinyl chloride• EthanolEthanol• Carbon TetrachlorideCarbon Tetrachloride• Anyline DyesAnyline Dyes• SmokingSmoking• HemochromatosisHemochromatosis• BenzeneBenzene• SchistomiasisSchistomiasis

Page 50: USMLE Step 1 clues 2- 800 slides

What are the 9 Live What are the 9 Live Vaccines?Vaccines?• MeaslesMeasles• MumpsMumps• RubellaRubella• Oral Polio (sabin)Oral Polio (sabin)• RotavirusRotavirus• Small poxSmall pox• BCGBCG• Yellow feverYellow fever• VaricellaVaricella

Page 51: USMLE Step 1 clues 2- 800 slides

What are the Killed What are the Killed Vaccines?Vaccines?• SIR Hep ASIR Hep A • Salk (polio)Salk (polio)

• InfluenzaInfluenza

• RubellaRubella

• Hepatitis AHepatitis A

Page 52: USMLE Step 1 clues 2- 800 slides

What are the IgA What are the IgA Nephropathies?Nephropathies?

• Henoch-Schoenlein P. (HSP)Henoch-Schoenlein P. (HSP)

• Alport’sAlport’s

• Berger’sBerger’s

Page 53: USMLE Step 1 clues 2- 800 slides

What are the Drugs that cause What are the Drugs that cause Autoimmune hemolytic anemia?Autoimmune hemolytic anemia?

• PCNPCN

• αα-methyldopa-methyldopa

• CephalosporinsCephalosporins

• SulfaSulfa

• PTUPTU

• Anti-malarialsAnti-malarials

• DapsoneDapsone

Page 54: USMLE Step 1 clues 2- 800 slides

What are the drugs that cause What are the drugs that cause Autoimmune Autoimmune thrombocytopenia?thrombocytopenia?• ASAASA

• HeparinHeparin

• QuinidineQuinidine

Page 55: USMLE Step 1 clues 2- 800 slides

What are the enzymes that What are the enzymes that show after an MI?show after an MI?

• Troponin ITroponin I

• CKMBCKMB

• LDHLDH

Page 56: USMLE Step 1 clues 2- 800 slides

What is the first MI enzyme What is the first MI enzyme to appear?to appear?• Troponin ITroponin I

• AppearsAppears

• PeaksPeaks

• GoneGone

• 2 hrs2 hrs

• 2 days2 days

• 7 days7 days

Page 57: USMLE Step 1 clues 2- 800 slides

What is the 2What is the 2ndnd MI enzyme MI enzyme to appear?to appear?• CK-MBCK-MB

• AppearsAppears

• PeaksPeaks

• GoneGone

• 6 hrs6 hrs

• 12 hrs12 hrs

• 24 hrs24 hrs

Page 58: USMLE Step 1 clues 2- 800 slides

What is the 3What is the 3rdrd MI enzyme to MI enzyme to appear?appear?• LDHLDH

• AppearsAppears

• PeaksPeaks

• GoneGone

• 1 day1 day

• 2 days2 days

• 3 days3 days

Page 59: USMLE Step 1 clues 2- 800 slides

What bacteria have Silver What bacteria have Silver Stains?Stains?

• LegionellaLegionella

• Pneumocysitis cariniiPneumocysitis carinii

• H. pyloriH. pylori

• Bartonella henseslae (lymph node)Bartonella henseslae (lymph node)

• Candida (yeast)Candida (yeast)

Page 60: USMLE Step 1 clues 2- 800 slides

What are the sulfa containing What are the sulfa containing drugs?drugs?

• SulfonamidesSulfonamides

• SulfonylureaSulfonylurea

• CelebrexCelebrex

Page 61: USMLE Step 1 clues 2- 800 slides

What is another name for What is another name for celebrex?celebrex?

• CelecoxibCelecoxib

Page 62: USMLE Step 1 clues 2- 800 slides

What type of inhibitor is What type of inhibitor is Celebrex?Celebrex?

• COX 2 specificCOX 2 specific

Page 63: USMLE Step 1 clues 2- 800 slides

What COX-2 specific drug can What COX-2 specific drug can you give to a pt with sulfa you give to a pt with sulfa allergy?allergy?• Vioxx (Rofecoxib)Vioxx (Rofecoxib)

Page 64: USMLE Step 1 clues 2- 800 slides

What drugs inhibit What drugs inhibit dihydrofolate reductase?dihydrofolate reductase?

• Pyremethamin/SulfadiazinePyremethamin/Sulfadiazine

• Trimethoprim/SulfamethoxazoleTrimethoprim/Sulfamethoxazole

Page 65: USMLE Step 1 clues 2- 800 slides

What drugs cause Pulmonary What drugs cause Pulmonary Fibrosis?Fibrosis?

• BleomycinBleomycin

• BulsufanBulsufan

• AmiodaroneAmiodarone

• TocainideTocainide

Page 66: USMLE Step 1 clues 2- 800 slides

What are the macrophage What are the macrophage deficiency diseases?deficiency diseases?

• Chediak-HigashiChediak-Higashi

• NADPH-oxidase deficiencyNADPH-oxidase deficiency

Page 67: USMLE Step 1 clues 2- 800 slides

What are the SE of Loops and What are the SE of Loops and Thiazides?Thiazides?

• HyperglycemiaHyperglycemia

• HyperuricemiaHyperuricemia

• HypovolemiaHypovolemia

• HypokalemiaHypokalemia

Page 68: USMLE Step 1 clues 2- 800 slides

What are the SE of Loop What are the SE of Loop diuretics?diuretics?

• OH DANGOH DANG • OtotoxicityOtotoxicity

• HypokalemiaHypokalemia

• DehydrationDehydration

• AllergyAllergy

• Nephritis Nephritis (interstitial)(interstitial)

• GoutGout

Page 69: USMLE Step 1 clues 2- 800 slides

What are the only 3 Pansystolic What are the only 3 Pansystolic Murmurs and when are they Murmurs and when are they heard?heard?• MRMR

• TRTR

• VSDVSD

• Decrease on Decrease on inspiration (^exp)inspiration (^exp)

• Increase on Increase on inspirationinspiration

• Decrease on Decrease on inspiration (^exp)inspiration (^exp)

Page 70: USMLE Step 1 clues 2- 800 slides

Macrophages in various Macrophages in various organsorgans• BrainBrain• LungLung• LiverLiver• SpleenSpleen• KidneyKidney• Lymph nodesLymph nodes• SkinSkin• BoneBone• CTCT

• MircogliaMircoglia• Type I pneumocyteType I pneumocyte• Kupffer cellKupffer cell• RESRES• MesangialMesangial• DendriticDendritic• LangerhansLangerhans• OsteoclastsOsteoclasts• Histiocytes orHistiocytes or• Giant cells or Giant cells or • Epithelioid cellsEpithelioid cells

Page 71: USMLE Step 1 clues 2- 800 slides

What are the 7 Rashes of the What are the 7 Rashes of the Palms & Soles?Palms & Soles?

• TSSTSS

• Rocky Mountain Spotted FeverRocky Mountain Spotted Fever

• Coxsackie A (Hand/Foot & mouth dz)Coxsackie A (Hand/Foot & mouth dz)

• KawasakiKawasaki

• SyphillisSyphillis

• Scarlet FeverScarlet Fever

• Staph Scalded Skin Syndrome Staph Scalded Skin Syndrome

Page 72: USMLE Step 1 clues 2- 800 slides

What is seen in every What is seen in every restrictive lung dz and low restrictive lung dz and low volume state?volume state?• TachypneaTachypnea

• Decrease pCO2Decrease pCO2

• Decrease pO2Decrease pO2

• Increase pHIncrease pH

Page 73: USMLE Step 1 clues 2- 800 slides

What are the different 2What are the different 2ndnd messenger systems?messenger systems?

• cAMPcAMP

• cGMPcGMP

• IP3/DAGIP3/DAG

• Ca:CalmodulinCa:Calmodulin

• Ca+Ca+

• Tyrosine kinaseTyrosine kinase

• NO NO

Page 74: USMLE Step 1 clues 2- 800 slides

What is the clue for cAMP?What is the clue for cAMP?

• It is the 90%It is the 90%

• SympatheticSympathetic

• CRH (cortisol)CRH (cortisol)

• CatabolicCatabolic

Page 75: USMLE Step 1 clues 2- 800 slides

What is the clue for cGMP?What is the clue for cGMP?

• ParasympatheticParasympathetic

• AnabolicAnabolic

Page 76: USMLE Step 1 clues 2- 800 slides

What are the clues for What are the clues for IP3/DAG?IP3/DAG?

• NeurotransmitterNeurotransmitter

• GHRHGHRH

• All hypothalamic hormones xc All hypothalamic hormones xc cortisolcortisol

• Used by what and for what?Used by what and for what?

• Smooth muscle for contractionSmooth muscle for contraction

Page 77: USMLE Step 1 clues 2- 800 slides

What is the clue for What is the clue for Ca:Calmodulin?Ca:Calmodulin?

• Used by smooth muscle for Used by smooth muscle for contraction by distentioncontraction by distention

Page 78: USMLE Step 1 clues 2- 800 slides

What is the clue for Ca+?What is the clue for Ca+?

• Used by Gastrin onlyUsed by Gastrin only

Page 79: USMLE Step 1 clues 2- 800 slides

What is the clue for Tyrosine What is the clue for Tyrosine Kinase?Kinase?

• Used by InsulinsUsed by Insulins

• Used by ALL growth factorsUsed by ALL growth factors

Page 80: USMLE Step 1 clues 2- 800 slides

What is the clue for NO?What is the clue for NO?

• NitratesNitrates

• ViagraViagra

• ANPANP

• LPSLPS

Page 81: USMLE Step 1 clues 2- 800 slides

What are the T & B cell What are the T & B cell deficiencies?deficiencies?

• WAS WAS

• SCIDSCID

• CVIDCVID

• HIVHIV

• HTLV-1HTLV-1

Page 82: USMLE Step 1 clues 2- 800 slides

What are the CLUES for What are the CLUES for WAS?WAS?

• ThrombocytopeniaThrombocytopenia

• IL-4IL-4

• InfectionInfection

• EczemaEczema

• Decrease IgMDecrease IgM

• IgE???IgE???

Page 83: USMLE Step 1 clues 2- 800 slides

What are the CLUES for What are the CLUES for SCID?SCID?

• Framshift/Nonsense mutationFramshift/Nonsense mutation

• Adenosine deaminase deficiencyAdenosine deaminase deficiency

• T-cell>B-cellT-cell>B-cell

• Bacterial infectionsBacterial infections

• Fungal infectionsFungal infections

Page 84: USMLE Step 1 clues 2- 800 slides

What are the CLUES for What are the CLUES for CVID?CVID?

• Late onsetLate onset

• Frameshift/Missense mutationFrameshift/Missense mutation

• Tyrosine Kinase deficiencyTyrosine Kinase deficiency

Page 85: USMLE Step 1 clues 2- 800 slides

What are the CLUES for HIV & What are the CLUES for HIV & HTLV-1?HTLV-1?

• T-cell>B-cellT-cell>B-cell

• CD4 richCD4 rich

• BrainBrain

• TesticlesTesticles

• CervixCervix

• Blood vesselsBlood vessels

Page 86: USMLE Step 1 clues 2- 800 slides

What are the inhibitors of What are the inhibitors of Complex 1 of the ETC?Complex 1 of the ETC?

• AmytalAmytal

• RotenoneRotenone

Page 87: USMLE Step 1 clues 2- 800 slides

What are the inhibitors of What are the inhibitors of Complex 2 of the ETC?Complex 2 of the ETC?

• MalonateMalonate

Page 88: USMLE Step 1 clues 2- 800 slides

What are the inhibitors of What are the inhibitors of Complex 3 of the ETC?Complex 3 of the ETC?

• Antimycin DAntimycin D

Page 89: USMLE Step 1 clues 2- 800 slides

What are the inhibitors of What are the inhibitors of Complex 4 of the ETC?Complex 4 of the ETC?

• CN-CN-

• COCO

• ChloramphenicolChloramphenicol

Page 90: USMLE Step 1 clues 2- 800 slides

What are the inhibitors of What are the inhibitors of Complex 5 of the ETC?Complex 5 of the ETC?

• OligomycinOligomycin

Page 91: USMLE Step 1 clues 2- 800 slides

What are the ETC chemical What are the ETC chemical uncouplers?uncouplers?

• DNPDNP

• Free Fatty acidsFree Fatty acids

• AspirinAspirin

Page 92: USMLE Step 1 clues 2- 800 slides

What type of uncoupler is What type of uncoupler is Aspirin?Aspirin?

• Physical uncouplerPhysical uncoupler

Page 93: USMLE Step 1 clues 2- 800 slides

What are the 4 sources of What are the 4 sources of Renal Acid?Renal Acid?

• PlasmaPlasma

• Urea cycleUrea cycle

• Collecting ductsCollecting ducts

• GlutaminaseGlutaminase

Page 94: USMLE Step 1 clues 2- 800 slides

What is the one dose tx for What is the one dose tx for Hemophilus ducreyi?Hemophilus ducreyi?

• AzithromycinAzithromycin

1 gram po1 gram po

• CeftriazoneCeftriazone

250 mg im250 mg im

Page 95: USMLE Step 1 clues 2- 800 slides

What is the one dose tx for What is the one dose tx for Chlaymdia?Chlaymdia?

• AzithromycinAzithromycin

1 gram po1 gram po

Page 96: USMLE Step 1 clues 2- 800 slides

What is the one dose tx for What is the one dose tx for Candidiasis?Candidiasis?

• KetoconazoleKetoconazole

150mg150mg

Page 97: USMLE Step 1 clues 2- 800 slides

What is the one dose tx for What is the one dose tx for Vaginal Candidiasis?Vaginal Candidiasis?

• DifluccanDifluccan

1 pill1 pill

Page 98: USMLE Step 1 clues 2- 800 slides

What is the one dose tx for What is the one dose tx for Trichomonas?Trichomonas?

• MetronidazoleMetronidazole

2 grams2 grams

Page 99: USMLE Step 1 clues 2- 800 slides

What is the one dose tx for What is the one dose tx for Gardnerella?Gardnerella?

• MetronidazoleMetronidazole

2 grams2 grams

Page 100: USMLE Step 1 clues 2- 800 slides

What are the 3 cephalosporins & What are the 3 cephalosporins & doses used as one dose doses used as one dose treatments for Gonorrhea?treatments for Gonorrhea?• CeftriaxoneCeftriaxone

250 mg im250 mg im

• CefiximeCefixime

400 mg po400 mg po

• CefoxitinCefoxitin

400 mg po400 mg po

Page 101: USMLE Step 1 clues 2- 800 slides

What are the 3 Quinolones & What are the 3 Quinolones & doses used as one dose doses used as one dose treatments for Gonorrhea?treatments for Gonorrhea?• CiprofloxacinCiprofloxacin

500 mg po500 mg po

• OfloxacinOfloxacin

400 mg po400 mg po

• GatifloxacinGatifloxacin

400 mg im400 mg im

Page 102: USMLE Step 1 clues 2- 800 slides

What are the 4 enzymes needed What are the 4 enzymes needed to break down glycogen?to break down glycogen?

• Phosphorylase (Pi)Phosphorylase (Pi)

• Debranching enzymeDebranching enzyme

• Alpha-1,6 –GlucosidaseAlpha-1,6 –Glucosidase

• PhosphatasePhosphatase

Page 103: USMLE Step 1 clues 2- 800 slides

What are the 2 enzymes What are the 2 enzymes needed to make glycogen?needed to make glycogen?

• Glycogen synthaseGlycogen synthase

• Branching enzymeBranching enzyme

Page 104: USMLE Step 1 clues 2- 800 slides

What are the branching What are the branching enzymes?enzymes?

• Glycogen alpha-1,4 glycosyl Glycogen alpha-1,4 glycosyl transferasetransferase

• Glycogen alpha-1,6 glycosyl Glycogen alpha-1,6 glycosyl transferasetransferase

Page 105: USMLE Step 1 clues 2- 800 slides

What is the rate limiting What is the rate limiting enzyme in the break down of enzyme in the break down of glycogen?glycogen?• Phosphorylase (Pi)Phosphorylase (Pi)

Page 106: USMLE Step 1 clues 2- 800 slides

What values do you see in What values do you see in obstructive pulmonary dz?obstructive pulmonary dz?

• pO2?pO2?

NormalNormal

• pCO2?pCO2?

Normal or increasedNormal or increased

• pH?pH?

DecreasedDecreased

Page 107: USMLE Step 1 clues 2- 800 slides

What values do you see in What values do you see in restrictive pulmonary dz?restrictive pulmonary dz?

• pO2?pO2?

Decreased Decreased

• pCO2?pCO2?

Decreased Decreased

• pH?pH?

IncreasedIncreased

Page 108: USMLE Step 1 clues 2- 800 slides

What type of acidosis do you What type of acidosis do you see with obstructive pulmonary see with obstructive pulmonary dz?dz?• Respiratory acidosisRespiratory acidosis

Page 109: USMLE Step 1 clues 2- 800 slides

What are the Lysosomal Storage What are the Lysosomal Storage Disease & what is the deficiency?Disease & what is the deficiency?

• Fabry’sFabry’s

• Krabbe’sKrabbe’s

• Gaucher’sGaucher’s

• Niemann – PickNiemann – Pick

• Tay-SachsTay-Sachs

• Metachromatic Metachromatic leukodystrophyleukodystrophy

• Hurler’sHurler’s

• Hunter’sHunter’s

• αα – galactosidase – galactosidase

• GalactosylceramideGalactosylceramide

• ββ – glucocerebrosidase – glucocerebrosidase

• SphingomyelinaseSphingomyelinase

• HexosaminidaseHexosaminidase

• ArylsulfataseArylsulfatase

• αα – L – iduronidase – L – iduronidase

• Iduronidase sulfataseIduronidase sulfatase

Page 110: USMLE Step 1 clues 2- 800 slides

What dz’s are associated with What dz’s are associated with HLA B27?HLA B27?

• PsoriasisPsoriasis

• Ankylosing spondylitisAnkylosing spondylitis

• IBD (Ulcerative colitis)IBD (Ulcerative colitis)

• Reiter’s SyndromeReiter’s Syndrome

Page 111: USMLE Step 1 clues 2- 800 slides

What HLA is Psorisis w/RA What HLA is Psorisis w/RA associated with?associated with?

• HLA-13HLA-13

Page 112: USMLE Step 1 clues 2- 800 slides

What are the Glycogen Storage What are the Glycogen Storage Diseases & the deficiency?Diseases & the deficiency?

• Von Gierke’sVon Gierke’s

• Pompe’sPompe’s

• Cori’sCori’s

• McArdle’sMcArdle’s

• Glucose – 6 – phosphateGlucose – 6 – phosphate

• αα – 1 – 4 glucosidase – 1 – 4 glucosidase

• Debranching enzymeDebranching enzyme

• Glycogen phosphorylaseGlycogen phosphorylase

Page 113: USMLE Step 1 clues 2- 800 slides

What are 6 places of the TCA cycle What are 6 places of the TCA cycle where amino acids feed in/out?where amino acids feed in/out?

• Pyruvate?Pyruvate?• GlycineGlycine• AlanineAlanine• SerineSerine

• Acetyl CoAAcetyl CoA ? ? • PhenylalaninePhenylalanine• IsoleucineIsoleucine• ThreonineThreonine• TryptophanTryptophan• LysineLysine• LeucineLeucine

Page 114: USMLE Step 1 clues 2- 800 slides

What are 6 places of the TCA cycle What are 6 places of the TCA cycle where amino acids feed in/out?where amino acids feed in/out?

• Alpha-KGAlpha-KG ? ?

• GlutamateGlutamate

• GlutamineGlutamine

• Succinyl CoA?Succinyl CoA?

• PhenylalaninePhenylalanine

• TryptophanTryptophan

• TyrosineTyrosine

Page 115: USMLE Step 1 clues 2- 800 slides

What are 6 places of the TCA cycle What are 6 places of the TCA cycle where amino acids feed in/out?where amino acids feed in/out?

• FumerateFumerate ? ?

• ProlineProline

• Oxaloacetate?Oxaloacetate?

• AspartateAspartate

• AsparigineAsparigine

Page 116: USMLE Step 1 clues 2- 800 slides

What are the 4 steps of B-What are the 4 steps of B-oxidation?oxidation?

• Oxidation – 7 NADH – 21 ATPOxidation – 7 NADH – 21 ATP

• HydrationHydration

• Oxidation - 7FADH – 14 ATPOxidation - 7FADH – 14 ATP

• Thiolysis – 8 AcCoA –Thiolysis – 8 AcCoA – 96ATP 96ATP 131 ATP – 2 (to bring it 131 ATP – 2 (to bring it

in)in)

Page 117: USMLE Step 1 clues 2- 800 slides

What are the blood gases in What are the blood gases in neuromuscular disease (= neuromuscular disease (= restrictive blood gases)?restrictive blood gases)?• pO2?pO2?

DecreasedDecreased• pCO2?pCO2?

DecreasedDecreased• PCWP?PCWP?

Decreased Decreased (b/c it’s a pressure problem)(b/c it’s a pressure problem)

• Respiratory Rate?Respiratory Rate?IncreasedIncreased

• pH?pH?IncreasedIncreased

• SZ?SZ?IncreasedIncreased

Page 118: USMLE Step 1 clues 2- 800 slides

What are 5 Hormones produced by What are 5 Hormones produced by small cell (oat cell) lung CA?small cell (oat cell) lung CA?

• ACTHACTH

• ADHADH

• PTHPTH

• TSHTSH

• ANPANP

Page 119: USMLE Step 1 clues 2- 800 slides

What Autoimmune Disease has What Autoimmune Disease has the following Autoimmune the following Autoimmune Antibodies?Antibodies?

•Anti-smith Anti-smith •Anti cardiolipin Anti cardiolipin •Anti-ds DNAAnti-ds DNA •SLESLE

Page 120: USMLE Step 1 clues 2- 800 slides

What Autoimmune Disease has What Autoimmune Disease has the following Autoimmune the following Autoimmune Antibodies?Antibodies?• Anti – histone?Anti – histone?

• Drug induced SLEDrug induced SLE

Page 121: USMLE Step 1 clues 2- 800 slides

What Autoimmune Disease has What Autoimmune Disease has the following Autoimmune the following Autoimmune Antibodies?Antibodies?• Anti-topoisomerase?Anti-topoisomerase?

• PSS (Progressive Systemic Sclerosis)PSS (Progressive Systemic Sclerosis)

Page 122: USMLE Step 1 clues 2- 800 slides

What Autoimmune Disease has What Autoimmune Disease has the following Autoimmune the following Autoimmune Antibodies?Antibodies?• Anti TSH receptors?Anti TSH receptors?

• GravesGraves

Page 123: USMLE Step 1 clues 2- 800 slides

What Autoimmune Disease has What Autoimmune Disease has the following Autoimmune the following Autoimmune Antibodies?Antibodies?• Anti-centromere?Anti-centromere?

• CRESTCREST

Page 124: USMLE Step 1 clues 2- 800 slides

What Autoimmune Disease has What Autoimmune Disease has the following Autoimmune the following Autoimmune Antibodies?Antibodies?• Anti-GBM?Anti-GBM?

• Goodpasture’sGoodpasture’s

Page 125: USMLE Step 1 clues 2- 800 slides

What does Goodpastures have What does Goodpastures have antibody to?antibody to?

• Type IV collagenType IV collagen

Page 126: USMLE Step 1 clues 2- 800 slides

What Autoimmune Disease has What Autoimmune Disease has the following Autoimmune the following Autoimmune Antibodies?Antibodies?• Anti-mitochondria?Anti-mitochondria?

• Primary biliary cirrhosisPrimary biliary cirrhosis

Page 127: USMLE Step 1 clues 2- 800 slides

What Autoimmune Disease has What Autoimmune Disease has the following Autoimmune the following Autoimmune Antibodies?Antibodies?• Anti-hair follicle?Anti-hair follicle?

• Alopecia areataAlopecia areata

Page 128: USMLE Step 1 clues 2- 800 slides

What Autoimmune Disease has What Autoimmune Disease has the following Autoimmune the following Autoimmune Antibodies?Antibodies?• Anti-IgG?Anti-IgG?

• Rheumatoid arthritisRheumatoid arthritis

Page 129: USMLE Step 1 clues 2- 800 slides

What Autoimmune Disease has What Autoimmune Disease has the following Autoimmune the following Autoimmune Antibodies? Antibodies?

• Anti-myelin receptors?Anti-myelin receptors?

• MSMS

Page 130: USMLE Step 1 clues 2- 800 slides

What Autoimmune Disease has What Autoimmune Disease has the following Autoimmune the following Autoimmune Antibodies?Antibodies?• Anti-gliaden?Anti-gliaden?

• Anti-gluten?Anti-gluten?

• Celiac sprueCeliac sprue

Page 131: USMLE Step 1 clues 2- 800 slides

What Autoimmune Disease has What Autoimmune Disease has the following Autoimmune the following Autoimmune Antibodies?Antibodies?• Anti-islet cell receptor?Anti-islet cell receptor?

• DM Type IDM Type I

Page 132: USMLE Step 1 clues 2- 800 slides

What Autoimmune Disease has What Autoimmune Disease has the following Autoimmune the following Autoimmune Antibodies?Antibodies?• Anti-melanocyte?Anti-melanocyte?

• ViteligoViteligo

Page 133: USMLE Step 1 clues 2- 800 slides

What Autoimmune Disease has What Autoimmune Disease has the following Autoimmune the following Autoimmune Antibodies?Antibodies?• Anti-ACh receptor?Anti-ACh receptor?

• MGMG

Page 134: USMLE Step 1 clues 2- 800 slides

What Autoimmune Disease has What Autoimmune Disease has the following Autoimmune the following Autoimmune Antibodies?Antibodies?• Anti-ribonuclear protein?Anti-ribonuclear protein?

• Mixed Connective Tissue dz (MCTD)Mixed Connective Tissue dz (MCTD)

Page 135: USMLE Step 1 clues 2- 800 slides

What Autoimmune Disease has What Autoimmune Disease has the following Autoimmune the following Autoimmune Antibodies?Antibodies?• Anti-parietal cell receptor?Anti-parietal cell receptor?

• Pernicious anemiaPernicious anemia

Page 136: USMLE Step 1 clues 2- 800 slides

What does Pernicious Anemia What does Pernicious Anemia have antibody to?have antibody to?

• Intrinsic factorIntrinsic factor

Page 137: USMLE Step 1 clues 2- 800 slides

What Autoimmune Disease has What Autoimmune Disease has the following Autoimmune the following Autoimmune Antibodies?Antibodies?• Anti-epidermal anchoring protein Anti-epidermal anchoring protein

receptors?receptors?

• Pemphigus vulgarisPemphigus vulgaris

Page 138: USMLE Step 1 clues 2- 800 slides

What does Pemphigus vulgaris What does Pemphigus vulgaris have antibody to?have antibody to?

• Intercelluar junctions of epidermal Intercelluar junctions of epidermal cellscells

Page 139: USMLE Step 1 clues 2- 800 slides

What Autoimmune Disease has What Autoimmune Disease has the following Autoimmune the following Autoimmune Antibodies?Antibodies?• Anti-epidermal basement membrane Anti-epidermal basement membrane

protein?protein?

• Bullous pemphigoidBullous pemphigoid

Page 140: USMLE Step 1 clues 2- 800 slides

What do you see with bullous What do you see with bullous pemphigoid?pemphigoid?

• IgG sub-epidermal blistersIgG sub-epidermal blisters

•Oral blistersOral blisters

Page 141: USMLE Step 1 clues 2- 800 slides

What Autoimmune Disease has What Autoimmune Disease has the following Autoimmune the following Autoimmune Antibodies?Antibodies?• Anti-platelet?Anti-platelet?

• ITPITP

Page 142: USMLE Step 1 clues 2- 800 slides

What does ITP have antibody What does ITP have antibody to?to?

• Glycoprotein IIb/IIIaGlycoprotein IIb/IIIa

Page 143: USMLE Step 1 clues 2- 800 slides

What Autoimmune Disease has What Autoimmune Disease has the following Autoimmune the following Autoimmune Antibodies?Antibodies?• Anti-thyroglobulin?Anti-thyroglobulin?

• Anti-microsomal?Anti-microsomal?

• Hashimoto’sHashimoto’s

Page 144: USMLE Step 1 clues 2- 800 slides

What Autoimmune Disease has What Autoimmune Disease has the following Autoimmune the following Autoimmune Antibodies?Antibodies?• Anti-smooth muscle?Anti-smooth muscle?

• Anti-scl-70?Anti-scl-70?

• SclerodermaScleroderma

Page 145: USMLE Step 1 clues 2- 800 slides

What Autoimmune Disease has What Autoimmune Disease has the following Autoimmune the following Autoimmune Antibodies?Antibodies?• Anti-rho (SS-A)?Anti-rho (SS-A)?

• Anti-la?Anti-la?

• Sjogren’sSjogren’s

Page 146: USMLE Step 1 clues 2- 800 slides

What Autoimmune Disease has What Autoimmune Disease has the following Autoimmune the following Autoimmune Antibodies?Antibodies?• Anti-proteinase?Anti-proteinase?

• C-ANCA?C-ANCA?

• Wegener’sWegener’s

Page 147: USMLE Step 1 clues 2- 800 slides

What Autoimmune Disease has What Autoimmune Disease has the following Autoimmune the following Autoimmune Antibodies?Antibodies?• P-ANCA?P-ANCA?

• Polyarteritis nodosaPolyarteritis nodosa

Page 148: USMLE Step 1 clues 2- 800 slides

What antigen & What antigen & immunoglobulin is Polyarteritis immunoglobulin is Polyarteritis nodosa associated with?nodosa associated with?•Hepatitis B antigenHepatitis B antigen

•IgMIgM

Page 149: USMLE Step 1 clues 2- 800 slides

What are the viruses that directly What are the viruses that directly cause CA and which CA do they cause CA and which CA do they cause?cause?• Papilloma virusPapilloma virus??

Cervical CACervical CA• EBVEBV??

BurkittsBurkittsNasopharyngeal CANasopharyngeal CA

• HepB & CHepB & C??Liver CALiver CA

• HIVHIV? ? Kaposi’s SarcomaKaposi’s Sarcoma

Page 150: USMLE Step 1 clues 2- 800 slides

What are the 7 Nephrotic Patterns What are the 7 Nephrotic Patterns seen with every Vasculitis?seen with every Vasculitis?

• Clot in front of renal artery?Clot in front of renal artery?

Renal artery stenosis Renal artery stenosis • Clot off whole renal artery?Clot off whole renal artery?

Renal failureRenal failure• Inflamed glomeruli?Inflamed glomeruli?

Glumerulo nephritisGlumerulo nephritis• Clot in papilla?Clot in papilla?

Papillary necrosis Papillary necrosis • Clot off medulla?Clot off medulla?

Interstitial nephritis Interstitial nephritis • Clot off pieces of nephron?Clot off pieces of nephron?

Focal segmental GN (HIV, drug use association)Focal segmental GN (HIV, drug use association)• Clot off lots of nephrons?Clot off lots of nephrons?

Rapidly Progressive GN Rapidly Progressive GN

Page 151: USMLE Step 1 clues 2- 800 slides

What is the most common What is the most common nephrotic disease seen in kids nephrotic disease seen in kids and when does it occur?and when does it occur?•Min. change diseaseMin. change disease

•2 wks post URI2 wks post URI

Page 152: USMLE Step 1 clues 2- 800 slides

What is the most common What is the most common vasculitity leading to rapidly vasculitity leading to rapidly progressive progressive glomerulonephrosis?glomerulonephrosis?• Goodpasture’s Goodpasture’s

Page 153: USMLE Step 1 clues 2- 800 slides

What is the most common What is the most common malignant renal tumor in malignant renal tumor in children?children?• Wilm’s tumorWilm’s tumor

Page 154: USMLE Step 1 clues 2- 800 slides

What is the most common What is the most common malignant renal tumor in malignant renal tumor in adults?adults?• AdenocarcinomaAdenocarcinoma

Page 155: USMLE Step 1 clues 2- 800 slides

What is the most common What is the most common renal mass?renal mass?

• CystCyst

Page 156: USMLE Step 1 clues 2- 800 slides

What is the most common What is the most common renal disease in renal disease in Blacks/Hispanics?Blacks/Hispanics?• Focal Segmental GNFocal Segmental GN

Page 157: USMLE Step 1 clues 2- 800 slides

What is the most common What is the most common nephrotic disease in adults?nephrotic disease in adults?

• Membranous GNMembranous GN

Page 158: USMLE Step 1 clues 2- 800 slides

Thrombolytics & InhibitorsThrombolytics & Inhibitors

• What does tPA, Streptokinase, What does tPA, Streptokinase, Urokinase inhibit?Urokinase inhibit?

• Aminocaproic acidAminocaproic acid

• What doe Warfarin inhibit? What doe Warfarin inhibit?

• Vitamin KVitamin K

• What does Heparin inhibit? What does Heparin inhibit?

• Protamine SulfateProtamine Sulfate

Page 159: USMLE Step 1 clues 2- 800 slides

What is the dosage of tPA?What is the dosage of tPA?

• IV push?IV push?

• 20mg20mg

• Drip?Drip?

• 40mg40mg

Page 160: USMLE Step 1 clues 2- 800 slides

What is the dosage for What is the dosage for Streptokinase?Streptokinase?

• IV push?IV push?

• 750K750K

• Drip?Drip?

• 750K750K

Page 161: USMLE Step 1 clues 2- 800 slides

What is Urokinase used for?What is Urokinase used for?

• Used ONLY for such things as:Used ONLY for such things as:

• Feeding tubesFeeding tubes

• Central linesCentral lines

• Fistulas Fistulas

Page 162: USMLE Step 1 clues 2- 800 slides

What is Alopecia Areata?What is Alopecia Areata?

• Loss of a patch of hairLoss of a patch of hair

Page 163: USMLE Step 1 clues 2- 800 slides

What is Alopecia Totalis?What is Alopecia Totalis?

• Loss of ALL hair on head “bald”Loss of ALL hair on head “bald”

Page 164: USMLE Step 1 clues 2- 800 slides

What is Alopecia What is Alopecia Universalis?Universalis?

• Loss of hair on entire body “hairless”Loss of hair on entire body “hairless”

Page 165: USMLE Step 1 clues 2- 800 slides

What is Loffler syndrome?What is Loffler syndrome?

• Pneumonitis with endocarditis = Pneumonitis with endocarditis = pulmonary infiltrate with severe pulmonary infiltrate with severe eosinophiliaeosinophilia

Page 166: USMLE Step 1 clues 2- 800 slides

What is Loffler syndrome also What is Loffler syndrome also known as?known as?

• PIE syndromePIE syndrome

Page 167: USMLE Step 1 clues 2- 800 slides

What are the 5 Parasites What are the 5 Parasites associated with associated with Loffler Loffler Syndrome?Syndrome?•Necator americanusNecator americanus

•Ankylostoma duodenaleAnkylostoma duodenale

•ShistosomiasisShistosomiasis

•StrongyloidesStrongyloides

•Ascaris lumbricoidesAscaris lumbricoides

Page 168: USMLE Step 1 clues 2- 800 slides

What happens when a patient What happens when a patient is on prednisone for > 7 days?is on prednisone for > 7 days?

•ImmunocompromisedImmunocompromised

Page 169: USMLE Step 1 clues 2- 800 slides

What are 2 enzymes used by What are 2 enzymes used by B12?B12?

• Homocystine methyl transferaseHomocystine methyl transferase

• Methyl malonyl-coA mutaseMethyl malonyl-coA mutase

Page 170: USMLE Step 1 clues 2- 800 slides

What does Mitochondrial What does Mitochondrial inheritance mean?inheritance mean?

• No male transmissionNo male transmission

• All females pass it onAll females pass it on

Page 171: USMLE Step 1 clues 2- 800 slides

Who are 4 pt’s who would be Who are 4 pt’s who would be susceptable to pseudomonas susceptable to pseudomonas and staph infxns?and staph infxns?•Burn patientsBurn patients

•Cystic fibrosisCystic fibrosis

•DMDM

•Neutropenic patientsNeutropenic patients

Page 172: USMLE Step 1 clues 2- 800 slides

In a neutropenic patient, what In a neutropenic patient, what do you cover for?do you cover for?

• cover 1x for Staph aureus during 1st cover 1x for Staph aureus during 1st weekweek

• cover 2x for Pseudo after 2nd weekcover 2x for Pseudo after 2nd week

Page 173: USMLE Step 1 clues 2- 800 slides

What are the 3 main concepts What are the 3 main concepts causing a widened S2 splitting?causing a widened S2 splitting?

• Increased pO2 Increased pO2

• Delayed opening/closing of the Delayed opening/closing of the pulmonary valuepulmonary value

• Increased volume in the right Increased volume in the right ventricleventricle

Page 174: USMLE Step 1 clues 2- 800 slides

What are causes for a What are causes for a widened S2 splitting?widened S2 splitting?

• Blood transfusionBlood transfusion• Increased Tidal VolumeIncreased Tidal Volume• Giving O2Giving O2• Right sided heart failureRight sided heart failure• Pregnancy due to increase volumePregnancy due to increase volume• IV fluidsIV fluids• ASD/VSDASD/VSD• Deep breathingDeep breathing• HypernateremiaHypernateremia• SIADHSIADH• Pulmonary regurgePulmonary regurge• Pulmonary stenosisPulmonary stenosis• Right bundle branch blockRight bundle branch block

Page 175: USMLE Step 1 clues 2- 800 slides

What are the 8 common What are the 8 common cavities of blood loss?cavities of blood loss?

• PPericardiumericardium• IIntracranial ntracranial • MMediastinumediastinum• PPleural cavityleural cavity• TThighshighs• RRetroperitoneumetroperitoneum• AAbdominal cavitybdominal cavity• PPelviselvis

Page 176: USMLE Step 1 clues 2- 800 slides

What is the special list for What is the special list for Penicillin?Penicillin?

• Gram +Gram +

• Basement membrane suppressorBasement membrane suppressor

• Works on simple anaerobesWorks on simple anaerobes

• The #1 cause of anaphylaxisThe #1 cause of anaphylaxis

• Causes interstial neprititsCauses interstial nepritits

• Causes nonspecific rashesCauses nonspecific rashes

• Acts as a hapten causing hemolytic Acts as a hapten causing hemolytic anemia anemia

Page 177: USMLE Step 1 clues 2- 800 slides

What is the #1 cause of What is the #1 cause of anaphylaxis?anaphylaxis?

• Penicillin Penicillin

Page 178: USMLE Step 1 clues 2- 800 slides

What are the Chron’s Gifts?What are the Chron’s Gifts?

• GranulomaGranuloma

• IleumIleum

• FistulaFistula

• TransmuralTransmural

• Skip LesionSkip Lesion

Page 179: USMLE Step 1 clues 2- 800 slides

What are the negative-What are the negative-stranded RNA Clues?stranded RNA Clues?

• Prodromal periodProdromal period before before symptoms = 1-3 weekssymptoms = 1-3 weeks

• Why is there a prodromal period?Why is there a prodromal period?

• Because must switch to positive Because must switch to positive stranded before replicationstranded before replication

Page 180: USMLE Step 1 clues 2- 800 slides

What are the clues for positive What are the clues for positive stranded RNA?stranded RNA?

• Symptoms within 1 week or less Symptoms within 1 week or less • EXCEPTIONSEXCEPTIONS: : • HantaHanta• EbolaEbola• Yellow feverYellow fever• They are -ve stranded = don’t have to They are -ve stranded = don’t have to

switch to positive before replicatingswitch to positive before replicating

Page 181: USMLE Step 1 clues 2- 800 slides

What are the Most common What are the Most common cyanotic heart diseases?cyanotic heart diseases?• Transposition of the great arteriesTransposition of the great arteries• Tetrology of FallotTetrology of Fallot• Truncus ArteriosusTruncus Arteriosus• Tricuspid AtresiaTricuspid Atresia• Total anomalous pulmunary Venous Return Total anomalous pulmunary Venous Return

• Hypoplastic Left heart syndromeHypoplastic Left heart syndrome• Ebstein’s anomalyEbstein’s anomaly• Aortic atresiaAortic atresia• Pulmonary atresiaPulmonary atresia

Page 182: USMLE Step 1 clues 2- 800 slides

What cyanotic heart disease is What cyanotic heart disease is – boot shaped?– boot shaped?

• Tetrology of FallotTetrology of Fallot

Page 183: USMLE Step 1 clues 2- 800 slides

What cyanotic heart disease is What cyanotic heart disease is associated with mom taking associated with mom taking lithium during pregnancy?lithium during pregnancy?• Ebstein’s Anomaly Ebstein’s Anomaly

Page 184: USMLE Step 1 clues 2- 800 slides

What things make the What things make the membrane less likely to membrane less likely to depolarize?depolarize?• HypokalemiaHypokalemia

• HypermagnesemiaHypermagnesemia

• Hypercalcemia (except atrium)Hypercalcemia (except atrium)

• HypernatremiaHypernatremia

Page 185: USMLE Step 1 clues 2- 800 slides

What things make the What things make the membrane more likely to membrane more likely to depolarize?depolarize?• HyperkalemiaHyperkalemia

• HypomagnasemiaHypomagnasemia

• Hypocalcemia (except atrium)Hypocalcemia (except atrium)

• HyponatremiaHyponatremia

Page 186: USMLE Step 1 clues 2- 800 slides

What is Plan F?What is Plan F?

• TTPPP – Thiamin – B1P – Thiamin – B1

• LLipoic Acid – B4ipoic Acid – B4

• CoCoAA – Pantothenic acid – B5 – Pantothenic acid – B5

• FFAD – Riboflavin – B2AD – Riboflavin – B2

•NAD – Niacin – B3AD – Niacin – B3

Page 187: USMLE Step 1 clues 2- 800 slides

What are the 8 x-linked What are the 8 x-linked inherited diseases?inherited diseases?• Bruton’s AgammaglobulinemiaBruton’s Agammaglobulinemia• CGD (NADPH def)CGD (NADPH def)• DMDDMD• Color BlindnessColor Blindness• G6PDG6PD• HemophiliaHemophilia• Lesch-NyhanLesch-Nyhan• Vit D resist. Rickets (X-linked dominant)Vit D resist. Rickets (X-linked dominant)• FabrysFabrys• HuntersHunters

Page 188: USMLE Step 1 clues 2- 800 slides

What are the 7 B-cell What are the 7 B-cell deficiencies?deficiencies?

• Bruton’s agammaglobulinemiaBruton’s agammaglobulinemia• CVID (Common Variant Imm. Def)CVID (Common Variant Imm. Def)• LeukemiasLeukemias• LymphomasLymphomas• SCIDSCID• WASWAS• Job Buckley SyndromeJob Buckley Syndrome

Page 189: USMLE Step 1 clues 2- 800 slides

What is the Tyrosine kinase What is the Tyrosine kinase deficiency?deficiency?

• Bruton’s agammaglobulinemiaBruton’s agammaglobulinemia

Page 190: USMLE Step 1 clues 2- 800 slides

What are the B-cell What are the B-cell deficiencies with T-cell deficiencies with T-cell overlap?overlap?• SCIDSCID

• WASWAS

• Job Buckley SyndromeJob Buckley Syndrome

Page 191: USMLE Step 1 clues 2- 800 slides

What are the 4 itchiest What are the 4 itchiest rashes?rashes?

• ScabiesScabies

• Lichen PlanusLichen Planus

• UrticariaUrticaria

• Dermatitis HerpetiformisDermatitis Herpetiformis

Page 192: USMLE Step 1 clues 2- 800 slides

Tumor Markers/Oncongenes Tumor Markers/Oncongenes II• L-myc?L-myc? • Small cell lung CaSmall cell lung Ca

• C-myc?C-myc?• Promyelocytic leukemia (Burkitt’s lymphoma)Promyelocytic leukemia (Burkitt’s lymphoma)

• N-myc?N-myc?• Neuroblastoma Neuroblastoma • Small cell lung CASmall cell lung CA

• C-able?C-able?• CML CML • ALLALL

Page 193: USMLE Step 1 clues 2- 800 slides

Tumor Markers/Oncongenes Tumor Markers/Oncongenes IIII• C-myb?C-myb?• Colon CAColon CA• AMLAML

• C-sis ?C-sis ?• OsteosarcomaOsteosarcoma• Glioma Glioma • FibrosarcomaFibrosarcoma

Page 194: USMLE Step 1 clues 2- 800 slides

Tumor Markers/Oncongenes Tumor Markers/Oncongenes IIIIII

• C-erb B2?C-erb B2?• Epidermal growth factor receptorsEpidermal growth factor receptors

• CSF-1 ?CSF-1 ?• BreastBreast

Page 195: USMLE Step 1 clues 2- 800 slides

Tumor Markers/Oncongenes Tumor Markers/Oncongenes IVIV

• Erb-B2?Erb-B2?• Breast CABreast CA• Ovarian CAOvarian CA• Gastric CAGastric CA

• Ret?Ret?• Medullary CA of thyroidMedullary CA of thyroid• Men II & IIIMen II & III• Papillary carcinomaPapillary carcinoma

Page 196: USMLE Step 1 clues 2- 800 slides

Tumor Markers/Oncongenes Tumor Markers/Oncongenes VV• Ki-ras?Ki-ras?• Lung CALung CA• Colon CAColon CA

• Bcl-2?Bcl-2?• BurkittsBurkitts• Follicular lymphomaFollicular lymphoma

• Erb?Erb?• RetinoblastomaRetinoblastoma

Page 197: USMLE Step 1 clues 2- 800 slides

What are 6 Hormones What are 6 Hormones produced by the placenta?produced by the placenta?• hCGhCG• InhibinInhibin• Human placental lactogen (HPL)Human placental lactogen (HPL)• Oxytocin (drug lactation, pit gland Oxytocin (drug lactation, pit gland

prod it also)prod it also)• ProgesteroneProgesterone• EstrogenEstrogen• RelaxinRelaxin

Page 198: USMLE Step 1 clues 2- 800 slides

What is cancer grading?What is cancer grading?

• Severity of microscopic changeSeverity of microscopic change

• Degree of differentiationDegree of differentiation

Page 199: USMLE Step 1 clues 2- 800 slides

What is cancer staging?What is cancer staging?

• Degree of dissemination of tumorDegree of dissemination of tumor

• What the surgeon seesWhat the surgeon sees

Page 200: USMLE Step 1 clues 2- 800 slides

What are the rashes associated What are the rashes associated with cancer and what cancer with cancer and what cancer are they associated with?are they associated with?

• Urticaria/Hives?Urticaria/Hives?• Any CA, especially lymphomaAny CA, especially lymphoma• Pagets Ds (ulcers around nipples)Pagets Ds (ulcers around nipples)

• Seborrheic keratosis (waxy warts)?Seborrheic keratosis (waxy warts)?• Colon CA Colon CA • HIV if sudden increase in number HIV if sudden increase in number • Normal with agingNormal with aging

Page 201: USMLE Step 1 clues 2- 800 slides

What are the rashes associated What are the rashes associated with cancer and what cancer with cancer and what cancer are they associated with?are they associated with?

• Actinic keratosis?Actinic keratosis? – Dry scaly plaques on sun-exposed skinDry scaly plaques on sun-exposed skin

• Squamous Cell CA of skinSquamous Cell CA of skin

• Dermatomyositis?Dermatomyositis? – violacious, heliotropic rash, malar areaviolacious, heliotropic rash, malar area

• Colon CAColon CA

Page 202: USMLE Step 1 clues 2- 800 slides

What are the rashes associated What are the rashes associated with Cancer and the cancer with Cancer and the cancer they are associated with?they are associated with?

• Akanthosis nigricans?Akanthosis nigricans? – dark lines in skin foldsdark lines in skin folds

• Any visceral CAAny visceral CA• End organ damageEnd organ damage

• Erythema nodosum?Erythema nodosum? – ant aspect of legs, tender nodulesant aspect of legs, tender nodules

• Anything granulomatousAnything granulomatous• NOT assoc. w/ bacteriaNOT assoc. w/ bacteria

Page 203: USMLE Step 1 clues 2- 800 slides

What is carried by HDL?What is carried by HDL?

• Apo EApo E

• Apo AApo A

• Apo CIIApo CII

• L-CAT L-CAT – lecithin cholesterol acetyl transferaselecithin cholesterol acetyl transferase

• CholesterolCholesterol– from periphery to liverfrom periphery to liver

Page 204: USMLE Step 1 clues 2- 800 slides

What is carried by VLDL?What is carried by VLDL?

• Apo B-100Apo B-100

• Apo EApo E

• Apo C IIApo C II

• Triglcyerides (95%)Triglcyerides (95%)

• Cholesterol (5%)Cholesterol (5%)

Page 205: USMLE Step 1 clues 2- 800 slides

What is carried by IDL?What is carried by IDL?

• Apo B-100Apo B-100

• Apo EApo E

• Apo CIIApo CII

• Triglycerides (< VLDL)Triglycerides (< VLDL)

• Cholesterol (>VLDL)Cholesterol (>VLDL)

Page 206: USMLE Step 1 clues 2- 800 slides

What is carried by LDL?What is carried by LDL?

• Apo B-100Apo B-100

• CholesterolCholesterol– from liver to tissuefrom liver to tissue

• NOT a good thing!!!!! NOT a good thing!!!!!

Page 207: USMLE Step 1 clues 2- 800 slides

What do chylomicrons What do chylomicrons carry?carry?• Apo AApo A• Apo B-48Apo B-48• Apo EApo E• Apo C IIApo C II

• Triglycerides from:Triglycerides from:• GI to liver (25% of the time)GI to liver (25% of the time)• GI to endothelium (75% of the time)GI to endothelium (75% of the time)

Page 208: USMLE Step 1 clues 2- 800 slides

Which lipoprotein carries Which lipoprotein carries the most cholesterol?the most cholesterol?

•LDLLDL

Page 209: USMLE Step 1 clues 2- 800 slides

Where are the AVMs?Where are the AVMs?

• Clue = HEALClue = HEAL• Heart?Heart?• Machinery murmurMachinery murmur

• Elbow?Elbow?• Fistula from dialysis in renal diseaseFistula from dialysis in renal disease

• Abdomen/Brain?Abdomen/Brain?• Von Hippel-Lindau = clot off with coilsVon Hippel-Lindau = clot off with coils• Increase incidence of Renal cell CA on chrom 3Increase incidence of Renal cell CA on chrom 3

• Lungs?Lungs?• Osler Weber Rendu SyndromeOsler Weber Rendu Syndrome

Page 210: USMLE Step 1 clues 2- 800 slides

What is the Ranson’s criteria What is the Ranson’s criteria for acute pancreatitis (for acute pancreatitis (at at admissionadmission)?)?• Glucose > 200Glucose > 200

• Age > 55Age > 55

• LDH >350LDH >350

• AST > 250AST > 250

• WBC > 16,000WBC > 16,000

Page 211: USMLE Step 1 clues 2- 800 slides

What is the Ranson’s criteria What is the Ranson’s criteria for acute pancreatitis (for acute pancreatitis (at less at less than 48 hrs)?than 48 hrs)?• Calcium <8 mg/dlCalcium <8 mg/dl

• HCT drop > 10%HCT drop > 10%

• O2 < 60 (PaO2)O2 < 60 (PaO2)

• Base deficit > 4Base deficit > 4

• BUN > 5 mg/dlBUN > 5 mg/dl

• Sequestration > 6LSequestration > 6L

Page 212: USMLE Step 1 clues 2- 800 slides

What 2 diseases is What 2 diseases is pilocarpine used for?pilocarpine used for?

• CFCF

• Glaucoma Glaucoma – Painful, red, teary eyePainful, red, teary eye

Page 213: USMLE Step 1 clues 2- 800 slides

What is dysguzia?What is dysguzia?

• Problem with sense of tasteProblem with sense of taste

Page 214: USMLE Step 1 clues 2- 800 slides

What are 3 causes of What are 3 causes of dysgusia?dysgusia?

• MetronidazoleMetronidazole

• ClarithromycinClarithromycin

• Zinc deficiencyZinc deficiency

Page 215: USMLE Step 1 clues 2- 800 slides

What is the triad of Carcinoid What is the triad of Carcinoid syndrome?syndrome?

• FlushingFlushing

• Wheezing Wheezing

• diarrheadiarrhea

Page 216: USMLE Step 1 clues 2- 800 slides

What do you measure for What do you measure for carcinoid syndrome?carcinoid syndrome?

• Serotonin Serotonin – 5-HIAA5-HIAA

Page 217: USMLE Step 1 clues 2- 800 slides

Where are the 2 most Where are the 2 most common places a carcinoid common places a carcinoid tumor is found?tumor is found?• PancreasPancreas

• IleumIleum

Page 218: USMLE Step 1 clues 2- 800 slides

What are the phage What are the phage mediated toxins?mediated toxins?

• MnemonicMnemonic: BEDS: BEDS

• BotulinumBotulinum

• Erythrogenic toxinErythrogenic toxin– from strep pyogenesfrom strep pyogenes

• DiptheriaDiptheria

• SalmonellaSalmonella– Has O antigenHas O antigen

Page 219: USMLE Step 1 clues 2- 800 slides

What is the story used to What is the story used to remember the segmented RNA remember the segmented RNA viruses?viruses?• I sprayed I sprayed ORTHOORTHO on my on my BUNYABUNYA at at

the the ARENAARENA down in down in REOREO to kill to kill SEGMENTED WORMSSEGMENTED WORMS

Page 220: USMLE Step 1 clues 2- 800 slides

Name the 3 major types of Name the 3 major types of adhesion moleculesadhesion molecules

• ICAMsICAMs

• IntegrinsIntegrins

• SelectinsSelectins

Page 221: USMLE Step 1 clues 2- 800 slides

What does IgCam do?What does IgCam do?

• Bind proteinsBind proteins

Page 222: USMLE Step 1 clues 2- 800 slides

What do integrins do?What do integrins do?

• Stop the leukocytesStop the leukocytes

Page 223: USMLE Step 1 clues 2- 800 slides

What do selectins do?What do selectins do?

• Bind carbohydratesBind carbohydrates

• Mediate the rolling to slow Mediate the rolling to slow leukocytes downleukocytes down

Page 224: USMLE Step 1 clues 2- 800 slides

What are the functions of What are the functions of adhesion molecules?adhesion molecules?

• Homing of lymphocytesHoming of lymphocytes– tells lymphocytes where to gotells lymphocytes where to go

• InflammationInflammation

• Cell-cell interactionCell-cell interaction

Page 225: USMLE Step 1 clues 2- 800 slides

Primary allergic response is Primary allergic response is due to what?due to what?•Contact Contact

Page 226: USMLE Step 1 clues 2- 800 slides

What cells are present in the What cells are present in the first 3 days?first 3 days?

• NeutrophilsNeutrophils

• The next cells to show up are?The next cells to show up are?

• B-cellsB-cells

• What do B-cells make?What do B-cells make?

• IgMIgM

Page 227: USMLE Step 1 clues 2- 800 slides

What day does IgM show What day does IgM show up? up?

• ThreeThree

• IgM peaks at what day?IgM peaks at what day?

• 1414

• When does IgM leave?When does IgM leave?

• In 2 monthsIn 2 months

Page 228: USMLE Step 1 clues 2- 800 slides

What shows up in 2 wks (14 What shows up in 2 wks (14 days)?days)?

• IgGIgG

• When does IgG peak?When does IgG peak?

• In 2 monthsIn 2 months

• When does IgG leave?When does IgG leave?

• In 1 yearIn 1 year

Page 229: USMLE Step 1 clues 2- 800 slides

What is Secondary Allergic What is Secondary Allergic response is due to?response is due to?• MEMORYMEMORY

Page 230: USMLE Step 1 clues 2- 800 slides

What shows up at day 3?What shows up at day 3?

• IgG with 5x concentrationIgG with 5x concentration

• Has the highest affinityHas the highest affinity

• When does IgG peak?When does IgG peak?

• In 5 yearsIn 5 years

• When does IgG leave?When does IgG leave?

• In 10 yearsIn 10 years

Page 231: USMLE Step 1 clues 2- 800 slides

What Ig has the hightest What Ig has the hightest affinity?affinity?

• IgGIgG

Page 232: USMLE Step 1 clues 2- 800 slides

What are the risk factors for What are the risk factors for Esophageal/Gastric CA?Esophageal/Gastric CA?

• SmokingSmoking

• AlcoholAlcohol

• NitritesNitrites

• JapaneseJapanese

Page 233: USMLE Step 1 clues 2- 800 slides

What are the risk factors for What are the risk factors for bladder CA?bladder CA?• Smoking Smoking • Aniline dyesAniline dyes• BenzeneBenzene• AflatoxinAflatoxin• CyclophosphamideCyclophosphamide• SchistosomiasisSchistosomiasis• 2 diseases:2 diseases:• Von Hippel-LindauVon Hippel-Lindau• Tubular sclerosisTubular sclerosis

Page 234: USMLE Step 1 clues 2- 800 slides

What is the NBT test?What is the NBT test?

• Nitro Blue Tetrazolium testNitro Blue Tetrazolium test

• What is it used for?What is it used for?

• Screening CGDScreening CGD

• What does a –ve test indicate?What does a –ve test indicate?

• +ve for the disease+ve for the disease

Page 235: USMLE Step 1 clues 2- 800 slides

What disease corresponds with What disease corresponds with the following inclusion bodies?the following inclusion bodies?• Howell-Jolly?Howell-Jolly?• Sickle cellSickle cell

• Heinz?Heinz?• G-6-P-DG-6-P-D

• Zebra?Zebra?• Niemann pickNiemann pick

Page 236: USMLE Step 1 clues 2- 800 slides

What disease corresponds with What disease corresponds with the following inclusion bodies?the following inclusion bodies?• Donovan? Donovan? • LeishmaniasisLeishmaniasis

• Mallory?Mallory?• AlcoholismAlcoholism

• Negri?Negri?• RabiesRabies

Page 237: USMLE Step 1 clues 2- 800 slides

What disease corresponds with What disease corresponds with the following inclusion bodies?the following inclusion bodies?

• Councilman?Councilman?

• Yellow feverYellow fever

• Call-exner?Call-exner?

• Ovarian tumors Ovarian tumors – granulosa origingranulosa origin

Page 238: USMLE Step 1 clues 2- 800 slides

What disease corresponds with What disease corresponds with the following inclusion bodies?the following inclusion bodies?

• Lewy?Lewy?• ParkinsonsParkinsons

• Pick?Pick?

• Pick’s diseasePick’s disease

• Barr body?Barr body?

• Normal femaleNormal female

Page 239: USMLE Step 1 clues 2- 800 slides

What disease corresponds with What disease corresponds with the following inclusion bodies?the following inclusion bodies?

• Aschoff?Aschoff?

• Rheumatic feverRheumatic fever

• Cowdry type A inclusions?Cowdry type A inclusions?

• Herpes virusHerpes virus

• Auer rods?Auer rods?

• AMLAML

Page 240: USMLE Step 1 clues 2- 800 slides

What disease corresponds with What disease corresponds with the following inclusion bodies?the following inclusion bodies?

• Globoid?Globoid?

• Krabbe’s lysosomal storage diseaseKrabbe’s lysosomal storage disease

• Russell? Russell?

• Multiple myelomaMultiple myeloma

Page 241: USMLE Step 1 clues 2- 800 slides

What disease corresponds with What disease corresponds with the following inclusion bodies?the following inclusion bodies?

• Schiller-Duvall?Schiller-Duvall?

• Yolk sac tumorYolk sac tumor

• Basal bodies?Basal bodies?

• Only found in smooth mmOnly found in smooth mm

Page 242: USMLE Step 1 clues 2- 800 slides

What are the 4 types of What are the 4 types of hypersensitivities?hypersensitivities?• Mnemonic?Mnemonic?• ACIDACID

• Type IType I• Anaphylaxis/AtopicAnaphylaxis/Atopic

• Type IIType II• Cytotoxic (Humoral)Cytotoxic (Humoral)

• Type IIIType III• Immune complex mediationImmune complex mediation

• Type IVType IV• Delayed hypersensitivity/Cell mediatedDelayed hypersensitivity/Cell mediated

Page 243: USMLE Step 1 clues 2- 800 slides

What are the Characteristics What are the Characteristics of Type I hypersensitivity?of Type I hypersensitivity?

• AtopicAtopic

• IgE (Asthma) binds to mast cellIgE (Asthma) binds to mast cell

• IgA activates IP3 cascade degrading IgA activates IP3 cascade degrading mast cellsmast cells

Page 244: USMLE Step 1 clues 2- 800 slides

What are the Characteristics What are the Characteristics of Type II hypersensitivity?of Type II hypersensitivity?

• HumoralHumoral

• What are examples of type II?What are examples of type II?

• Rh diseaseRh disease

• GoodpasturesGoodpastures

• Autoimmune hemolytic AnemiaAutoimmune hemolytic Anemia

• All Autoimmune diseases All Autoimmune diseases – except RA and SLEexcept RA and SLE

Page 245: USMLE Step 1 clues 2- 800 slides

What are the Characteristics What are the Characteristics of Type III hypersensitivity?of Type III hypersensitivity?

• Ag-Ab complementAg-Ab complement

• What are examples of Type III?What are examples of Type III?

• RARA

• SLESLE

• VasculitidesVasculitides

• Some GN?Some GN?

Page 246: USMLE Step 1 clues 2- 800 slides

What are the Characteristics What are the Characteristics of Type IV hypersensitivity?of Type IV hypersensitivity?

• Cell mediatedCell mediated

• What are examples of Type IV?What are examples of Type IV?

• TB skin testTB skin test

• Contact dermatitisContact dermatitis

• Transplant rejectionTransplant rejection

Page 247: USMLE Step 1 clues 2- 800 slides

What structures have no What structures have no known function?known function?

• AppendixAppendix

• EpithalamusEpithalamus

• Palmaris longus Palmaris longus – musclemuscle

• Pancreatic polypeptide Pancreatic polypeptide – hormones in F-cellshormones in F-cells

Page 248: USMLE Step 1 clues 2- 800 slides

What diseases can progress What diseases can progress to RPGN?to RPGN?

• GoodpasturesGoodpastures

• WegenersWegeners

• DMDM

• HTNHTN

Page 249: USMLE Step 1 clues 2- 800 slides

What are causes of papillary What are causes of papillary necrosis?necrosis?

• VasculitisVasculitis

• AIDSAIDS

Page 250: USMLE Step 1 clues 2- 800 slides

Cytic fibrosis Questions?Cytic fibrosis Questions?

• Tx?Tx?• Pilocarpine Pilocarpine • also used for glaucomaalso used for glaucoma

• Test used to detect CF?Test used to detect CF?• Pilocarpine sweat testPilocarpine sweat test

• What ion does this test measure?What ion does this test measure?• Cl-Cl-

• Definitive presence of disease has a test value of what?Definitive presence of disease has a test value of what?• >60>60

Page 251: USMLE Step 1 clues 2- 800 slides

Cytic fibrosis Questions?Cytic fibrosis Questions?

• What is the value in a normal person?What is the value in a normal person?• <20<20

• What is the value in a heterozygous person?What is the value in a heterozygous person?• 30 – 6030 – 60

• What chromosome is the CF gene on?What chromosome is the CF gene on?• Chrom 7Chrom 7

• What Second messenger is used?What Second messenger is used?• IP3/DAGIP3/DAG

Page 252: USMLE Step 1 clues 2- 800 slides

What are the gram +ve spore What are the gram +ve spore formers?formers?

• Bacillus anthracisBacillus anthracis

• Clostridium perfringensClostridium perfringens

• Clostridium tetaniClostridium tetani

• Clostridium melangosepticusClostridium melangosepticus

• What is the chemical in spores?What is the chemical in spores?

• Calcium dipocholinateCalcium dipocholinate

Page 253: USMLE Step 1 clues 2- 800 slides

What does strep mutans What does strep mutans ferment?ferment?

• Lactic acidLactic acid

Page 254: USMLE Step 1 clues 2- 800 slides

What type of receptors do all What type of receptors do all sphincters in the body have?sphincters in the body have?

• Alpha-receptorsAlpha-receptors

Page 255: USMLE Step 1 clues 2- 800 slides

Strep. Salivarius ag is used Strep. Salivarius ag is used for what test?for what test?

• Cold agglutinin testing Cold agglutinin testing – IgMIgM

Page 256: USMLE Step 1 clues 2- 800 slides

Types of amyloid found in various Types of amyloid found in various SystemicSystemic amyloidoses: amyloidoses:

• AA amyloid?AA amyloid?

• Chronic active diseaseChronic active disease

• AL amyloid from Ig light chain?AL amyloid from Ig light chain?

• MyelomaMyeloma

Page 257: USMLE Step 1 clues 2- 800 slides

Types of amyloid found in various Types of amyloid found in various SystemicSystemic amyloidoses: amyloidoses:

• Beta 2 microglobulin?Beta 2 microglobulin?

• Chronic hemodialysisChronic hemodialysis

• AA amyloid from SAA?AA amyloid from SAA?

• Nephrotic hereditary forms Nephrotic hereditary forms – eg. Mediterranean fevereg. Mediterranean fever

Page 258: USMLE Step 1 clues 2- 800 slides

Types of amyloid found in various Types of amyloid found in various SystemicSystemic amyloidoses: amyloidoses:

• Pre-albumin/transthyretin?Pre-albumin/transthyretin?

• Cardiomyopathic hereditary formsCardiomyopathic hereditary forms– senile systemic amyloidosissenile systemic amyloidosis

• Neuropathic hereditary syndromesNeuropathic hereditary syndromes

Page 259: USMLE Step 1 clues 2- 800 slides

Types of amyloid found in Types of amyloid found in various various Local Local amyloidoses:amyloidoses:

• ANP fibrils are caused by?ANP fibrils are caused by?• Senile cardiac amyloisosisSenile cardiac amyloisosis

• Cerebral amyloid in Alzheimer’s Cerebral amyloid in Alzheimer’s disease/Down’s?disease/Down’s?

• Cerebral amyloidosisCerebral amyloidosis

Page 260: USMLE Step 1 clues 2- 800 slides

Types of amyloid found in Types of amyloid found in various various Local Local amyloidoses:amyloidoses:

• Calcitonin precursors?Calcitonin precursors?• Medullary CA of thyroidMedullary CA of thyroid

• AL from light chains?AL from light chains?• Isolated, massive, nodular depositsIsolated, massive, nodular deposits

– lung, skin, urogenital tractlung, skin, urogenital tract

Page 261: USMLE Step 1 clues 2- 800 slides

What type of dementia do What type of dementia do you get in Pick’s dis?you get in Pick’s dis?

• Frontotemporal dementiaFrontotemporal dementia

Page 262: USMLE Step 1 clues 2- 800 slides

Describe Pick’s diseaseDescribe Pick’s disease

• Atrophy of frontal and temporal Atrophy of frontal and temporal cortex with sparing of remaining cortex with sparing of remaining neocortical regionsneocortical regions

Page 263: USMLE Step 1 clues 2- 800 slides

What 3 things do Pick What 3 things do Pick bodies contain?bodies contain?

• Altered neurofilamentsAltered neurofilaments

• Tau proteinTau protein

• ubiquitinubiquitin

Page 264: USMLE Step 1 clues 2- 800 slides

What drugs can cause a What drugs can cause a disulfiram reaction?disulfiram reaction?• Mnemonic?Mnemonic?• CLAMCLAM

• ChloramphenicolChloramphenicol• Lactams Lactams

– CefamandoleCefamandole– CefoperazoneCefoperazone

• AntabuseAntabuse– disulfiram disulfiram

• MetronidazoleMetronidazole

Page 265: USMLE Step 1 clues 2- 800 slides

What is the mode of action of the What is the mode of action of the Clostridium botulinum toxin?Clostridium botulinum toxin?

• Prevents pre-synaptic release of AchPrevents pre-synaptic release of Ach

• How do babies get it?How do babies get it?

• From spores in honey or molassesFrom spores in honey or molasses

• How do adults get it?How do adults get it?

• From canned foodFrom canned food

Page 266: USMLE Step 1 clues 2- 800 slides

What are 3 Toxins of What are 3 Toxins of Bacillus?Bacillus?

• Lethal factor – (black necrosis)Lethal factor – (black necrosis)

• Protective factorProtective factor

• Edema factorEdema factor

Page 267: USMLE Step 1 clues 2- 800 slides

Who has Poly-D Glutamic Who has Poly-D Glutamic acid?acid?

• AnthracisAnthracis

• CereusCereus

Page 268: USMLE Step 1 clues 2- 800 slides

Name that B-blocker:Name that B-blocker:

• B1-selective?B1-selective?• A – MA – M• Non-selective?Non-selective?• N – ZN – Z• Exceptions?Exceptions?• Carbetalol and Labetolol are non-Carbetalol and Labetolol are non-

selectiveselective

Page 269: USMLE Step 1 clues 2- 800 slides

What are the 4 facts of Fanconi What are the 4 facts of Fanconi Syndrome?Syndrome?

• Problem in proximal tubuleProblem in proximal tubule

• Can’t reabsorbCan’t reabsorb

• Low energy state causing anemiaLow energy state causing anemia

• Can be due to old tetracyclineCan be due to old tetracycline

Page 270: USMLE Step 1 clues 2- 800 slides

Where is glutaminase Where is glutaminase found?found?

• In the collecting duct of the kidneyIn the collecting duct of the kidney

• What does glutmainase help the What does glutmainase help the kidney absorb?kidney absorb?

• Ammonia if the liver failsAmmonia if the liver fails

Page 271: USMLE Step 1 clues 2- 800 slides

Name 3 anatomical spots Name 3 anatomical spots where renal stones get stuck:where renal stones get stuck:

• HilumHilum

• Pelvic brimPelvic brim

• Entering the bladderEntering the bladder

Page 272: USMLE Step 1 clues 2- 800 slides

Renal failure is the most Renal failure is the most common cause of death in common cause of death in what 3 diseases?what 3 diseases?• SLESLE

• Endometrial CAEndometrial CA

• Cervical CACervical CA

Page 273: USMLE Step 1 clues 2- 800 slides

What is the rate-limiting What is the rate-limiting enzyme in the urea cycle?enzyme in the urea cycle?

• Carbamoyl synthase ICarbamoyl synthase I

• Where is it found 90% of the time?Where is it found 90% of the time?

• LiverLiver

• Where is it found 10% of the time?Where is it found 10% of the time?

• Collecting duct of the kidneyCollecting duct of the kidney

Page 274: USMLE Step 1 clues 2- 800 slides

What type of charge does What type of charge does heparin have?heparin have?

•-ve charge-ve charge

Page 275: USMLE Step 1 clues 2- 800 slides

What type of charge does What type of charge does protamine sulfate have?protamine sulfate have?

•+ve charge+ve charge

• What is it used for?What is it used for?

• Reversing the effects of heparinReversing the effects of heparin

Page 276: USMLE Step 1 clues 2- 800 slides

What is commonly seen in What is commonly seen in all vasculitides?all vasculitides?• T-cells and macrophagesT-cells and macrophages• SchistocytesSchistocytes• Decreased plateletsDecreased platelets• Decreased RBCsDecreased RBCs• Bleeding from mucosal surfacesBleeding from mucosal surfaces• Bleeding from skin and GIBleeding from skin and GI• PetechiaePetechiae• EcchymosesEcchymoses

Page 277: USMLE Step 1 clues 2- 800 slides

What happens if you expose What happens if you expose the blood to the basement the blood to the basement membrane?membrane?• The following deveop:The following deveop:

• ClotsClots

• DICDIC

• Pulmonary embolismPulmonary embolism

• DVTDVT

• MIMI

• StrokeStroke

Page 278: USMLE Step 1 clues 2- 800 slides

Signs and symptoms in all Signs and symptoms in all vasculitidesvasculitides

• Tachypnea and SOBTachypnea and SOB

• Most common cause of death?Most common cause of death?

• Heart failureHeart failure

Page 279: USMLE Step 1 clues 2- 800 slides

What is the MOA of What is the MOA of Erythromycin?Erythromycin?

• Inhibits the translocation step of Inhibits the translocation step of ribosomal protein synthesisribosomal protein synthesis

Page 280: USMLE Step 1 clues 2- 800 slides

What is the MOA of What is the MOA of Chloramphenicol?Chloramphenicol?

• Inhibits ribosomal peptidyl Inhibits ribosomal peptidyl transferase in prokaryotestransferase in prokaryotes

Page 281: USMLE Step 1 clues 2- 800 slides

What is the MOA of What is the MOA of Puromycin?Puromycin?

• Inhibits elongation by binding to “A” Inhibits elongation by binding to “A” site and prematurely terminating site and prematurely terminating chain growth in pro and eukaryoteschain growth in pro and eukaryotes

Page 282: USMLE Step 1 clues 2- 800 slides

What is the MOA of What is the MOA of Streptomycin?Streptomycin?

• Causes misreading of code during Causes misreading of code during initiation in prokaryotesinitiation in prokaryotes

Page 283: USMLE Step 1 clues 2- 800 slides

What is the MOA of What is the MOA of Tetracycline?Tetracycline?

• Prevents binding of aminoacyl-t-RNA Prevents binding of aminoacyl-t-RNA to ribosome on prokaryotes therefore to ribosome on prokaryotes therefore inhibiting initiationinhibiting initiation

Page 284: USMLE Step 1 clues 2- 800 slides

What is the MOA of What is the MOA of Cyclohexamide?Cyclohexamide?

• Inhibits ribosomal peptidyl Inhibits ribosomal peptidyl transferase in eukaryotes transferase in eukaryotes – cell wall inhibitorcell wall inhibitor

Page 285: USMLE Step 1 clues 2- 800 slides

What is the MOA of What is the MOA of Rifampin?Rifampin?

• Blocks B-subunit of RNA polymeraseBlocks B-subunit of RNA polymerase

• Prophylaxis for contacts of N. Prophylaxis for contacts of N. meningitidismeningitidis

Page 286: USMLE Step 1 clues 2- 800 slides

What is the MOA of What is the MOA of Vancomycin?Vancomycin?

• Cell wall inhibitorCell wall inhibitor

• Binds irreversibly to Phopholipase Binds irreversibly to Phopholipase carriercarrier

• BacteriacidalBacteriacidal

• Covers all gram +vesCovers all gram +ves

• LinezolidLinezolid

Page 287: USMLE Step 1 clues 2- 800 slides

What is the MOA of What is the MOA of Warfarin?Warfarin?

• Blocks vitamin k dependent gamma-Blocks vitamin k dependent gamma-carboxylation of prothrombin and carboxylation of prothrombin and factors 2, 7, 9, 10, proteins C & Sfactors 2, 7, 9, 10, proteins C & S

Page 288: USMLE Step 1 clues 2- 800 slides

What is the MOA of What is the MOA of Clindamycin?Clindamycin?

• Blocks translation by binding the 50S Blocks translation by binding the 50S subunitsubunit

Page 289: USMLE Step 1 clues 2- 800 slides

Hemolytic properties of Hemolytic properties of Streptococcus:Streptococcus:• What type of hemolysis is alpha What type of hemolysis is alpha

hemolysis?hemolysis?• Partial hemolysisPartial hemolysis

• What color is it’s zone?What color is it’s zone?• GreenGreen

• What type of hemolysis is beta-hemolysis?What type of hemolysis is beta-hemolysis?• Complete hemolysisComplete hemolysis

Page 290: USMLE Step 1 clues 2- 800 slides

Hemolytic properties of Hemolytic properties of Streptococcus:Streptococcus:

• What color is it’s zoneWhat color is it’s zone• Clear Clear

– eg. Streptokinaseeg. Streptokinase

• What type of hemolysis is gamma-hemolysis?What type of hemolysis is gamma-hemolysis?• No hemolysisNo hemolysis

• What color is it’s zone?What color is it’s zone?• RedRed

Page 291: USMLE Step 1 clues 2- 800 slides

What are the 5 notable What are the 5 notable things about RTA I?things about RTA I?

• High urine PH (??????not sure about High urine PH (??????not sure about this)this)

• AcidosisAcidosis

• UTI sUTI s

• StonesStones

• Babies die < 1 yr oldBabies die < 1 yr old

Page 292: USMLE Step 1 clues 2- 800 slides

What are 3 notable things What are 3 notable things about RTA II?about RTA II?

• Acidosis Acidosis – urine PH = 2, normal is 5-6urine PH = 2, normal is 5-6

• HypokalmiaHypokalmia

• Patients have NO carbonic anhydrasePatients have NO carbonic anhydrase

Page 293: USMLE Step 1 clues 2- 800 slides

What are 3 notable things What are 3 notable things about RTA III?about RTA III?

• It is a combination of RTA I & IIIIt is a combination of RTA I & III

• Normal urine pHNormal urine pH

• HypokalemiaHypokalemia

Page 294: USMLE Step 1 clues 2- 800 slides

What are 3 notable things What are 3 notable things about RTA IV?about RTA IV?

• Seen in diabeticsSeen in diabetics

• HyperkalemiaHyperkalemia

• NO aldosterone b/c JG apparatus has NO aldosterone b/c JG apparatus has infarctedinfarcted

Page 295: USMLE Step 1 clues 2- 800 slides

What are the members of What are the members of Streptococcus Group D?Streptococcus Group D?

• ViridansViridans

• MutansMutans

• SanguisSanguis

• SalivariusSalivarius

• BovisBovis

Page 296: USMLE Step 1 clues 2- 800 slides

What Steptococcus has What Steptococcus has green pigment?green pigment?

• ViridansViridans

Page 297: USMLE Step 1 clues 2- 800 slides

What Streptococcus causes What Streptococcus causes SBE?SBE?

• ViridansViridans

Page 298: USMLE Step 1 clues 2- 800 slides

What Streptococcus causes What Streptococcus causes cavities?cavities?

• MutansMutans

Page 299: USMLE Step 1 clues 2- 800 slides

What do you see in What do you see in Nephritic Syndrome?Nephritic Syndrome?

• HTNHTN

• HematuriaHematuria

• RBC castsRBC casts

Page 300: USMLE Step 1 clues 2- 800 slides

What do you see in What do you see in Nephrotic Syndrome?Nephrotic Syndrome?

• Increase EdemaIncrease Edema

• Increase LipidemiaIncrease Lipidemia

• Increase CholesterolemiaIncrease Cholesterolemia

• Increase CoagulabilityIncrease Coagulability

• Decrease serum AlbuminDecrease serum Albumin

• Increase urinary AlbuminIncrease urinary Albumin

Page 301: USMLE Step 1 clues 2- 800 slides

What is the #1 cause of What is the #1 cause of Sinusitis, Otitis, Bronchitis, Sinusitis, Otitis, Bronchitis, Pneumonia?Pneumonia?• Strep. PneumoStrep. Pneumo

• What is the #2 cause?What is the #2 cause?

• Hemophilus influenzaHemophilus influenza

• What is the #3 cause?What is the #3 cause?

• Neisseria meningitidesNeisseria meningitides

Page 302: USMLE Step 1 clues 2- 800 slides

What is the #1 method to What is the #1 method to paralyze cilia?paralyze cilia?• VirusesViruses

• Which are secondary to what?Which are secondary to what?

• Bacterial infectionsBacterial infections

• What is the #2 method to paralyze cilia?What is the #2 method to paralyze cilia?

• SmokingSmoking

Page 303: USMLE Step 1 clues 2- 800 slides

If you develop gastroenteritis If you develop gastroenteritis within 8hrs of eating what are within 8hrs of eating what are the most common likely bugs? the most common likely bugs? • Staph aureusStaph aureus

• Clostridium perfringensClostridium perfringens

• Bacillus cereus….from what?Bacillus cereus….from what?

• Fried riceFried rice

Page 304: USMLE Step 1 clues 2- 800 slides

Gastroenteritis within 8hrs of Gastroenteritis within 8hrs of eating what toxin?eating what toxin?

• PreformedPreformed

Page 305: USMLE Step 1 clues 2- 800 slides

What does Clostridum tetani What does Clostridum tetani inhibit? inhibit? • Release of glycine from spinal cordRelease of glycine from spinal cord

• What physical finding would you see?What physical finding would you see?• Lock jawLock jaw

• What is the tx?What is the tx?• Antitoxin and ToxoidAntitoxin and Toxoid

• Where is it injected?Where is it injected?• Injected in different areas of bodyInjected in different areas of body

Page 306: USMLE Step 1 clues 2- 800 slides

The Most common cause of UTI The Most common cause of UTI is?is?

• E. coliE. coli

• Followed by?Followed by?

• ProteusProteus

• Followed by?Followed by?

• KlebsiellaKlebsiella

Page 307: USMLE Step 1 clues 2- 800 slides

The most frequent cause of UTI The most frequent cause of UTI in females between 5-10?in females between 5-10?• Staph saprophyticusStaph saprophyticus

• Why?Why?• They stick things in themselvesThey stick things in themselves

• 18-24 yoa?18-24 yoa?• Staph saprophyticusStaph saprophyticus• Why?Why?• Because they stick things inside themselvesBecause they stick things inside themselves

• Why no UTI’s after 24?Why no UTI’s after 24?• Because women are use to penises and Staph Because women are use to penises and Staph

saprophyticus lives on penis (becomes part of normal flora).saprophyticus lives on penis (becomes part of normal flora).

Page 308: USMLE Step 1 clues 2- 800 slides

Staph aureus is the most Staph aureus is the most common cause of what common cause of what bone disease?bone disease?• OsteomyelitisOsteomyelitis

• Because of what?Because of what?

• CollagenaseCollagenase

Page 309: USMLE Step 1 clues 2- 800 slides

What is the Most Common What is the Most Common cause of infections one week cause of infections one week post burn injury?post burn injury?

• Staph. aureusStaph. aureus

Page 310: USMLE Step 1 clues 2- 800 slides

What is the triad of SSSS?What is the triad of SSSS?

• ShockShock

• RashRash

• HypotensionHypotension

Page 311: USMLE Step 1 clues 2- 800 slides

Most common cause of UTI?Most common cause of UTI?

• E. coliE. coli

• Then?Then?

• ProteusProteus

• Then?Then?

• KlebsiellaKlebsiella

Page 312: USMLE Step 1 clues 2- 800 slides

Newborn meningitis is Newborn meningitis is caused by?caused by?

• Group B Strep (agalactiae)Group B Strep (agalactiae)

• E. coliE. coli

• ListeriaListeria

Page 313: USMLE Step 1 clues 2- 800 slides

What is normal rectal flora What is normal rectal flora from momfrom mom

• Group B Strep (Strep. Agalactiae)Group B Strep (Strep. Agalactiae)

• E. coliE. coli

• ListeriaListeria

Page 314: USMLE Step 1 clues 2- 800 slides

What is associated with What is associated with colon CA?colon CA?

• Clostridium melanogosepticusClostridium melanogosepticus

• Strep bovisStrep bovis

• What color pigment is produced?What color pigment is produced?

• BlackBlack

Page 315: USMLE Step 1 clues 2- 800 slides

What Ig do you look for with What Ig do you look for with affinity?affinity?

• IgGIgG

• What about Avidity?What about Avidity?

• IgMIgM

Page 316: USMLE Step 1 clues 2- 800 slides

What is transductionWhat is transduction

• Virus inject it’s DNA into bacteriaVirus inject it’s DNA into bacteria

Page 317: USMLE Step 1 clues 2- 800 slides

What is transformation?What is transformation?

• Virus injects it’s DNA into it bacteria Virus injects it’s DNA into it bacteria in a hospital or nursing home setting, in a hospital or nursing home setting, then becomes deadly.then becomes deadly.

Page 318: USMLE Step 1 clues 2- 800 slides

Conjuction occurs only with Conjuction occurs only with what?what?

• Bacteria with PiliBacteria with Pili

Page 319: USMLE Step 1 clues 2- 800 slides

What causes mutiple cerebral What causes mutiple cerebral abscesses in newborns?abscesses in newborns?

• CitrobacterCitrobacter

Page 320: USMLE Step 1 clues 2- 800 slides

What are the 2 gram –ves that What are the 2 gram –ves that are strict anaerobes?are strict anaerobes?

• Hemophilus influenzaHemophilus influenza

• NeisseriaNeisseria

Page 321: USMLE Step 1 clues 2- 800 slides

What type of complement What type of complement problem do you have in problem do you have in recurrent infections with recurrent infections with encapsulated organisms?encapsulated organisms?

• C3C3

Page 322: USMLE Step 1 clues 2- 800 slides

What does complement What does complement fight against?fight against?

• Gram negative bacteriaGram negative bacteria

Page 323: USMLE Step 1 clues 2- 800 slides

What do you see in serum with What do you see in serum with prerenal failure and what are prerenal failure and what are the values?the values?• BUN BUN

• >20>20

• Fractional Na+ excertionFractional Na+ excertion

• <1%<1%

• CreatineCreatine

• >40>40

Page 324: USMLE Step 1 clues 2- 800 slides

What do you see in Renal What do you see in Renal failure and what are the failure and what are the values?values?• BUNBUN

• 10-1510-15

• Fractional Na+ excretionFractional Na+ excretion

• >2%>2%

• CreatinineCreatinine

• <20<20

Page 325: USMLE Step 1 clues 2- 800 slides

What is the extravasation What is the extravasation order?order?

• PavementingPavementing

• MarginationMargination

• DiapediesisDiapediesis

• MigrationMigration

Page 326: USMLE Step 1 clues 2- 800 slides

What is the rate limiting What is the rate limiting enzyme for Glycolysis?enzyme for Glycolysis?

• PFK-1PFK-1

Page 327: USMLE Step 1 clues 2- 800 slides

What is the rate limiting What is the rate limiting enzyme of Gluconeogenesis?enzyme of Gluconeogenesis?

• Pyruvate CarboxlyasePyruvate Carboxlyase

Page 328: USMLE Step 1 clues 2- 800 slides

If treating a disease that If treating a disease that initiates the cell mediated initiates the cell mediated response, what are you response, what are you treating first?treating first?• ViralViral

• If resistant to tx, what next?If resistant to tx, what next?

• FungalFungal

• MycobacteriumMycobacterium

• ProtozoaProtozoa

• ParasiteParasite

• NeoplasmNeoplasm

Page 329: USMLE Step 1 clues 2- 800 slides

What bugs can you pick up What bugs can you pick up during birth?during birth?

• Step. Group BStep. Group B– Strep agalactiaeStrep agalactiae

• Strep. PneumoniaStrep. Pneumonia

• Herpes simplex virusHerpes simplex virus

• Neisseria gonorrheaNeisseria gonorrhea

• ChlyamydiaChlyamydia

Page 330: USMLE Step 1 clues 2- 800 slides

What is another name for What is another name for Adenoma sebaceum?Adenoma sebaceum?

• Perivascular angiofibromataPerivascular angiofibromata

Page 331: USMLE Step 1 clues 2- 800 slides

What is another name for What is another name for Addison’s?Addison’s?

• Primary Adrenocoritcal InsufficiencyPrimary Adrenocoritcal Insufficiency

Page 332: USMLE Step 1 clues 2- 800 slides

What is another name for What is another name for Alkaptonuria?Alkaptonuria?

• OchronosisOchronosis

Page 333: USMLE Step 1 clues 2- 800 slides

What is another name for What is another name for Churg-Strauss?Churg-Strauss?

• Allergic GranulomatosisAllergic Granulomatosis

• AngiitisAngiitis

Page 334: USMLE Step 1 clues 2- 800 slides

What is another name for What is another name for Craniopharyngioma?Craniopharyngioma?

• AmeloblastomaAmeloblastoma

• What is Ameloblast?What is Ameloblast?

• Tooth materialTooth material

Page 335: USMLE Step 1 clues 2- 800 slides

What is another name for What is another name for Chrons?Chrons?

• Regional enteritisRegional enteritis

• Granulomatous ileitisGranulomatous ileitis

• IleocolitisIleocolitis

Page 336: USMLE Step 1 clues 2- 800 slides

What is another name for What is another name for DeQuervain’s?DeQuervain’s?

• Subacute Granulomatous ThyroiditisSubacute Granulomatous Thyroiditis

Page 337: USMLE Step 1 clues 2- 800 slides

What is another name for What is another name for Intraductal Ca?Intraductal Ca?

• Comedo CaComedo Ca

Page 338: USMLE Step 1 clues 2- 800 slides

What is another name for I-What is another name for I-Cell Disease?Cell Disease?

• Mucolipidosis IIMucolipidosis II

Page 339: USMLE Step 1 clues 2- 800 slides

What is another name for What is another name for Kawassaki Disease?Kawassaki Disease?

• MLNSMLNS

• Mucocutaneous Lymph Node Mucocutaneous Lymph Node SyndromeSyndrome

Page 340: USMLE Step 1 clues 2- 800 slides

What is another name for What is another name for Leydig cells?Leydig cells?

• Interstitial cellsInterstitial cells

Page 341: USMLE Step 1 clues 2- 800 slides

What is another name for What is another name for Sertoli cells?Sertoli cells?

• Sustentacular cellsSustentacular cells

Page 342: USMLE Step 1 clues 2- 800 slides

What is another name for What is another name for Temporal arteritis?Temporal arteritis?

• Giant cell arteritis (granulomatous)Giant cell arteritis (granulomatous)

Page 343: USMLE Step 1 clues 2- 800 slides

What is another name for What is another name for Waldenstrom’s Waldenstrom’s macroglobulinemia?macroglobulinemia?• Hyperviscosity syndromeHyperviscosity syndrome

Page 344: USMLE Step 1 clues 2- 800 slides

HHV I causes?HHV I causes?

• Oral Oral

• Trigeminal gangliaTrigeminal ganglia

Page 345: USMLE Step 1 clues 2- 800 slides

HHV II causes?HHV II causes?

• GenitalGenital

• Sacral plexusSacral plexus

Page 346: USMLE Step 1 clues 2- 800 slides

HHV III causes?HHV III causes?

• Varicella zosterVaricella zoster

Page 347: USMLE Step 1 clues 2- 800 slides

HHV IV causes? HHV IV causes?

• EBVEBV

• MononucleosisMononucleosis

• Burkitt’sBurkitt’s

Page 348: USMLE Step 1 clues 2- 800 slides

HHV V causes?HHV V causes?

• CMVCMV

• Inclusion bodiesInclusion bodies

Page 349: USMLE Step 1 clues 2- 800 slides

HHV VI causes?HHV VI causes?

• RoseolaRoseola

• Duke DiseaseDuke Disease

• Exanthem subitumExanthem subitum

Page 350: USMLE Step 1 clues 2- 800 slides

HHV VII causes?HHV VII causes?

• Pityriasis roseaPityriasis rosea

Page 351: USMLE Step 1 clues 2- 800 slides

HHV VIII causes?HHV VIII causes?

• Kaposi’s sarcomaKaposi’s sarcoma

Page 352: USMLE Step 1 clues 2- 800 slides

Answer the following questions Answer the following questions about Coumadin/Warfarin.about Coumadin/Warfarin.

• What is the MOA?What is the MOA?• Interferes with normal synthesis and Interferes with normal synthesis and

gama carboxylation of Vit. K dependent gama carboxylation of Vit. K dependent clotting factors via vitamin K antagonism.clotting factors via vitamin K antagonism.

• Is it long or short acting?Is it long or short acting?• Long half-lifeLong half-life• 8-10 hours to act8-10 hours to act

Page 353: USMLE Step 1 clues 2- 800 slides

Answer the following questions Answer the following questions about Coumadin/Warfarin.about Coumadin/Warfarin.

• Clinical use?Clinical use?

• Chronic anticoagulationChronic anticoagulation

• Contra-indication?Contra-indication?

• Pregnancy because it can cross the Pregnancy because it can cross the placentaplacenta

Page 354: USMLE Step 1 clues 2- 800 slides

Answer the following questions Answer the following questions about Coumadin/Warfarin.about Coumadin/Warfarin.

• What pathway does it affect?What pathway does it affect?

• Extrinsic pathwayExtrinsic pathway

• What does it do to PT?What does it do to PT?

• ProlongsProlongs

• PTPT

Page 355: USMLE Step 1 clues 2- 800 slides

Answer the following questions Answer the following questions about Coumadin/Warfarin.about Coumadin/Warfarin.

• What are the toxicities?What are the toxicities?

• BleedingBleeding

• TeratogenicTeratogenic

• Drug-drug interactionsDrug-drug interactions

• How is it activatied?How is it activatied?

• Tissue activatedTissue activated

Page 356: USMLE Step 1 clues 2- 800 slides

Answer the following questions Answer the following questions about Coumadin/Warfarin.about Coumadin/Warfarin.

• Administration?Administration?

• popo

Page 357: USMLE Step 1 clues 2- 800 slides

What are the Vitamin K What are the Vitamin K dependent clotting factors?dependent clotting factors?

• IIII

• VIIVII

• IXIX

• XX

• Protein CProtein C

• Protein SProtein S

Page 358: USMLE Step 1 clues 2- 800 slides

Answer the following questions Answer the following questions about Heparin.about Heparin.

• What is the MOA?What is the MOA?

• Catalyzes the activation of antithrombin IIICatalyzes the activation of antithrombin III

• Decreases thrombin and XaDecreases thrombin and Xa

• Is it long or short acting?Is it long or short acting?

• Short half-lifeShort half-life

• Acts immediatelyActs immediately

Page 359: USMLE Step 1 clues 2- 800 slides

Answer the following questions Answer the following questions about Heparin.about Heparin.

• Clinical use?Clinical use?

• Immediate anticoagulation of Immediate anticoagulation of pulmonary embolism, stroke, angina, pulmonary embolism, stroke, angina, MI, DVT.MI, DVT.

• Contra-indication?Contra-indication?

• Can be used during pregnancy because Can be used during pregnancy because it does not cross the placentait does not cross the placenta

Page 360: USMLE Step 1 clues 2- 800 slides

Answer the following questions Answer the following questions about Heparin.about Heparin.

• What pathway does it affect?What pathway does it affect?

• Intrinsic pathwayIntrinsic pathway

• What value should you follow?What value should you follow?

• PTTPTT

Page 361: USMLE Step 1 clues 2- 800 slides

Answer the following questions Answer the following questions about Heparin.about Heparin.

• What are the toxicities?What are the toxicities?

• BleedingBleeding

• ThrombocytopeniaThrombocytopenia

• Drug-drug interactionsDrug-drug interactions

• How is it activatied?How is it activatied?

• Blood activatedBlood activated

Page 362: USMLE Step 1 clues 2- 800 slides

Answer the following questions Answer the following questions about Heparin.about Heparin.

• Administration?Administration?

• I.V.I.V.

• Drug of choice for what?Drug of choice for what?

• DVTDVT

Page 363: USMLE Step 1 clues 2- 800 slides

Answer the following questions Answer the following questions about Heparin.about Heparin.

• What is good about the newer low-What is good about the newer low-molecular-weight heparins?molecular-weight heparins?

• They act more on XaThey act more on Xa

• Have better bioavailability Have better bioavailability

• Have 2 to 4 times longer half lifeHave 2 to 4 times longer half life

• Can be administered subcutaneously Can be administered subcutaneously and without laboratory monitoring.and without laboratory monitoring.

Page 364: USMLE Step 1 clues 2- 800 slides

What do you use for rapid What do you use for rapid reversal of heparinization?reversal of heparinization?

• Protamine sulfateProtamine sulfate

Page 365: USMLE Step 1 clues 2- 800 slides

How do you treat Lead How do you treat Lead Poisoning?Poisoning?

• DimercaprolDimercaprol

Page 366: USMLE Step 1 clues 2- 800 slides

How do you treat How do you treat Benzodiazepine poisoning?Benzodiazepine poisoning?

• FlumazenilFlumazenil

Page 367: USMLE Step 1 clues 2- 800 slides

How do you treat How do you treat Anticholinesterase poisoning?Anticholinesterase poisoning?

• PralidoximePralidoxime

Page 368: USMLE Step 1 clues 2- 800 slides

How do you treat Iron How do you treat Iron poisoning?poisoning?

• DeferoxamineDeferoxamine

Page 369: USMLE Step 1 clues 2- 800 slides

How do you treat Opioid How do you treat Opioid poisoning?poisoning?

• NaloxeneNaloxene

Page 370: USMLE Step 1 clues 2- 800 slides

How do you treat Barbituate How do you treat Barbituate poisoning?poisoning?

• BicarbonateBicarbonate

• Doxapram Doxapram

Page 371: USMLE Step 1 clues 2- 800 slides

What does Doxapram do?What does Doxapram do?

• Activates the respiratory center in Activates the respiratory center in the brainthe brain

Page 372: USMLE Step 1 clues 2- 800 slides

What is the treatment for What is the treatment for Hypercholesterolemia?Hypercholesterolemia?

• ProvostatinProvostatin

• AtrovastatinAtrovastatin

• LovastatinLovastatin

• SimvastatinSimvastatin

Page 373: USMLE Step 1 clues 2- 800 slides

What statin is renally What statin is renally excreted?excreted?

• ProvastatinProvastatin

Page 374: USMLE Step 1 clues 2- 800 slides

What statins do you have to What statins do you have to follow liver enzymes every 3 follow liver enzymes every 3 months?months?• AtrovastatinAtrovastatin

• LovastatinLovastatin

• SimvastatinSimvastatin

Page 375: USMLE Step 1 clues 2- 800 slides

What do statins inhibit?What do statins inhibit?

• HMG-CoA reductaseHMG-CoA reductase

• When is it most active?When is it most active?

• 8:00pm on8:00pm on

Page 376: USMLE Step 1 clues 2- 800 slides

If statins are insufficient what If statins are insufficient what do you add?do you add?

• CholestipolCholestipol

• CholestyramineCholestyramine

• If nothing works what do you give?If nothing works what do you give?

• ProbucolProbucol

• NiacinNiacin

Page 377: USMLE Step 1 clues 2- 800 slides

What are the side effects of What are the side effects of Niacin?Niacin?

• FlushingFlushing

• ItchingItching

Page 378: USMLE Step 1 clues 2- 800 slides

What 2 statins bind bile What 2 statins bind bile salts?salts?

• CholestipolCholestipol

• CholestyramineCholestyramine

Page 379: USMLE Step 1 clues 2- 800 slides

What are 4 causes of severe What are 4 causes of severe pain (in order)?pain (in order)?1.Pancreatitis1.Pancreatitis• Due to What?Due to What?• ETOHETOH

2.Kidney stones2.Kidney stones • Due to What?Due to What?• AlcoholAlcohol

Page 380: USMLE Step 1 clues 2- 800 slides

What are 4 causes of severe What are 4 causes of severe pain (in order)?pain (in order)?

3.AAA3.AAA• How is this described?How is this described?• Ripping pain down backRipping pain down back

4.Ischemic bowel4.Ischemic bowel• What is symptom?What is symptom?• Bloody diarrheaBloody diarrhea

Page 381: USMLE Step 1 clues 2- 800 slides

What are 5 causes of What are 5 causes of SIADH?SIADH?

• Small cell Ca of lungSmall cell Ca of lung

• Increased intracranial pressureIncreased intracranial pressure

• Pain (most common)Pain (most common)

• DrugsDrugs

• Hypoxic Lung Disease/Restrictive Hypoxic Lung Disease/Restrictive Lung diseaseLung disease

Page 382: USMLE Step 1 clues 2- 800 slides

What drug causes SIADH?What drug causes SIADH?

• CarbamazepineCarbamazepine

Page 383: USMLE Step 1 clues 2- 800 slides

What are the cells of neural What are the cells of neural crest origin?crest origin?• Parafollicular cells of thyroidParafollicular cells of thyroid• Odontoblasts (predentin)Odontoblasts (predentin)• Pseudounipolar cellsPseudounipolar cells• Spiral membrane of heartSpiral membrane of heart• Chromaffin cellsChromaffin cells• All Ganglion cells (Schwann, Adrenal All Ganglion cells (Schwann, Adrenal

medulla)medulla)• MelanocytesMelanocytes• Laryngeal/Tracheal cartilageLaryngeal/Tracheal cartilage

Page 384: USMLE Step 1 clues 2- 800 slides

What are the triple repeat What are the triple repeat diseases?diseases?

• Huntington’sHuntington’s

• Fragile XFragile X

• Myotonic DystrophyMyotonic Dystrophy

• Prauder WilliePrauder Willie

• Spinal/bulbar muscular atrophy Spinal/bulbar muscular atrophy (Fredicks ataxia)(Fredicks ataxia)

Page 385: USMLE Step 1 clues 2- 800 slides

How do you determine the How do you determine the maximum sinus rate?maximum sinus rate?

• 220 - age220 - age

Page 386: USMLE Step 1 clues 2- 800 slides

What are the 3 low volume What are the 3 low volume states with acidosis rather than states with acidosis rather than alkalosis?alkalosis?• RTA RTA

• DiarrheaDiarrhea

• Diabetic ketoacidosis (DKA)Diabetic ketoacidosis (DKA)

Page 387: USMLE Step 1 clues 2- 800 slides

What are the causes of What are the causes of Croup & Bronchiolities?Croup & Bronchiolities?

• ParainfluenzaParainfluenza

• AdenovirusAdenovirus

• InfluenzaInfluenza

• RSVRSV

Page 388: USMLE Step 1 clues 2- 800 slides

What is asthma in a child What is asthma in a child less than 2 called?less than 2 called?

• BronchiolitisBronchiolitis

Page 389: USMLE Step 1 clues 2- 800 slides

What are the 4 D’s of What are the 4 D’s of Pellagra?Pellagra?

• DiarrheaDiarrhea

• DermatitisDermatitis

• DementiaDementia

• DeathDeath

Page 390: USMLE Step 1 clues 2- 800 slides

What are the uric acid What are the uric acid stones?stones?

• CysteineCysteine

• OrnithineOrnithine

• LysineLysine

• Arginine Arginine

Page 391: USMLE Step 1 clues 2- 800 slides

What is happening in the What is happening in the Atrium?Atrium?• Phase 0?Phase 0?• DepolarizationDepolarization

• Phase 1?Phase 1?

• No nameNo name

• Phase 2?Phase 2?

• Plateau phase (A-V node)Plateau phase (A-V node)

Page 392: USMLE Step 1 clues 2- 800 slides

What is happening in the What is happening in the Atrium?Atrium?

• Phase 3?Phase 3?

• RepolarizationRepolarization

• Phase 4?Phase 4?

• Automaticity (S-A node)Automaticity (S-A node)

Page 393: USMLE Step 1 clues 2- 800 slides

What do Na+ channels do What do Na+ channels do to the EKG?to the EKG?

• Wider QRSWider QRS

Page 394: USMLE Step 1 clues 2- 800 slides

What does Ca+ do to the What does Ca+ do to the EKG?EKG?

• Wider P-waveWider P-wave

• Longer PR intervalLonger PR interval

Page 395: USMLE Step 1 clues 2- 800 slides

What are the types of What are the types of kidney stones?kidney stones?

• Calcium oxalate (phosphate) stonesCalcium oxalate (phosphate) stones

• Struvite stonesStruvite stones

• Uric acid stonesUric acid stones

• Cysteine stonesCysteine stones

• Oxalate stones Oxalate stones

Page 396: USMLE Step 1 clues 2- 800 slides

What percent of kidney stones What percent of kidney stones are calcium oxalate?are calcium oxalate?

• 80%80%

Page 397: USMLE Step 1 clues 2- 800 slides

If you find oxalate stones in the If you find oxalate stones in the following what should you following what should you think of?think of?• 3 y/o white male?3 y/o white male?• CFCF

• 5 y/o black male?5 y/o black male?• Celiac SprueCeliac Sprue

Page 398: USMLE Step 1 clues 2- 800 slides

If you find oxalate stones in the If you find oxalate stones in the following what should you following what should you think of?think of?• Adult male?Adult male?

• Whipple’sWhipple’s

• Adult male or female?Adult male or female?

• Crohn’sCrohn’s

Page 399: USMLE Step 1 clues 2- 800 slides

If oxalate stones found in CF If oxalate stones found in CF what is the most common what is the most common cause?cause?• In 0-20 y/o?In 0-20 y/o?

• MalabsorptinMalabsorptin

• What age do they die?What age do they die?

• YoungYoung

Page 400: USMLE Step 1 clues 2- 800 slides

Answer the following questions Answer the following questions about pseudogout?about pseudogout?

•What type of crystals are What type of crystals are present?present?

•Calcium pyrophosphateCalcium pyrophosphate •Where are they found?Where are they found?• Joint spacesJoint spaces

Page 401: USMLE Step 1 clues 2- 800 slides

Answer the following questions Answer the following questions about pseudogout?about pseudogout?

•Who gets it?Who gets it?•Older patients M=FOlder patients M=F

•Tx?Tx?•Colchicine Colchicine

Page 402: USMLE Step 1 clues 2- 800 slides

What are the most common What are the most common non-cyanotic heart disease?non-cyanotic heart disease?

• VSDVSD

• ASDASD

• PDAPDA

• CoarctationCoarctation

Page 403: USMLE Step 1 clues 2- 800 slides

What murmur increases on What murmur increases on expiration?expiration?

• VSDVSD

• Mitral Mitral

Page 404: USMLE Step 1 clues 2- 800 slides

What murmur has fixed What murmur has fixed wide splitting?wide splitting?

• ASDASD

Page 405: USMLE Step 1 clues 2- 800 slides

What murmur has bounding What murmur has bounding pulses?pulses?

• PDAPDA

Page 406: USMLE Step 1 clues 2- 800 slides

What gives you differenital What gives you differenital pulses?pulses?

• CoarctationCoarctation

Page 407: USMLE Step 1 clues 2- 800 slides

What is increased incidence What is increased incidence in Turners?in Turners?

• Coarctation Coarctation

Page 408: USMLE Step 1 clues 2- 800 slides

What are 4 enzymes never What are 4 enzymes never seen in glycolysis?seen in glycolysis?

• Pyruvate carboxylasePyruvate carboxylase

• PEP carboxykinasePEP carboxykinase

• F-1,6 dPhosphataseF-1,6 dPhosphatase

• G-6-PhosphataseG-6-Phosphatase

Page 409: USMLE Step 1 clues 2- 800 slides

What are 3 enzymes seen What are 3 enzymes seen ONLY in glycolysis?ONLY in glycolysis?

• HexokinaseHexokinase

• PFK-1PFK-1

• Pyruvate kinasePyruvate kinase

Page 410: USMLE Step 1 clues 2- 800 slides

What are 2 hormones that What are 2 hormones that are acidophilic?are acidophilic?

• ProlactinProlactin

• GHGH

Page 411: USMLE Step 1 clues 2- 800 slides

What are the partially acid What are the partially acid fast Gram +ve?fast Gram +ve?

• NocardiaNocardia

Page 412: USMLE Step 1 clues 2- 800 slides

What are the partially acid What are the partially acid fast Protozoa?fast Protozoa?

• CryptosporidiumCryptosporidium

Page 413: USMLE Step 1 clues 2- 800 slides

What are the septic emboli What are the septic emboli of SBE?of SBE?

• Mycotic aneurysmMycotic aneurysm

• Roth spotsRoth spots

• Janeway lesionsJaneway lesions

• Osler’s nodesOsler’s nodes

• Splinter hemorrhagesSplinter hemorrhages

• EndocarditisEndocarditis

Page 414: USMLE Step 1 clues 2- 800 slides

Where are the following Where are the following lesions found?lesions found?• Janeway lesions?Janeway lesions?

• ToesToes

• Osler’s nodes?Osler’s nodes?

• FingersFingers

• Roth spots?Roth spots?

• RetinaRetina

Page 415: USMLE Step 1 clues 2- 800 slides

What is the most common What is the most common cause of endocarditis?cause of endocarditis?

• Strep. viridansStrep. viridans

Page 416: USMLE Step 1 clues 2- 800 slides

What causes What causes microsteatosis?microsteatosis?

• AcetaminophenAcetaminophen

• Reye SyndromeReye Syndrome

• PregnancyPregnancy

Page 417: USMLE Step 1 clues 2- 800 slides

What causes What causes macrosteatosis?macrosteatosis?

• AlcoholAlcohol

Page 418: USMLE Step 1 clues 2- 800 slides

What are 2 bacteria that What are 2 bacteria that release elastase?release elastase?

• Staph. AureusStaph. Aureus

• PseudomonasPseudomonas

Page 419: USMLE Step 1 clues 2- 800 slides

What are the 2 bacteria with What are the 2 bacteria with toxins that inhibit EF-2?toxins that inhibit EF-2?

• PseudomonasPseudomonas

• DiptheriaDiptheria

Page 420: USMLE Step 1 clues 2- 800 slides

How does Diptheria work?How does Diptheria work?

• It ADP ribosylates EF2 inhibiting protein It ADP ribosylates EF2 inhibiting protein synthesissynthesis

• Is it Gram +/-?Is it Gram +/-?

• ++

• Where and how does it get its exotoxin?Where and how does it get its exotoxin?

• From virus via transductionFrom virus via transduction

Page 421: USMLE Step 1 clues 2- 800 slides

How does Diptheria work?How does Diptheria work?

• What does it cause?What does it cause?

• Heart blockHeart block

• What do you give for Tx?What do you give for Tx?

• AntitoxinAntitoxin

• Never scrape membraneNever scrape membrane

Page 422: USMLE Step 1 clues 2- 800 slides

What are the different types of What are the different types of Emphysema and their causes?Emphysema and their causes?

• Bullous?Bullous?

• Staph aureusStaph aureus

• PseudomonasPseudomonas

• Centroacinar?Centroacinar?

• Smoking Smoking

Page 423: USMLE Step 1 clues 2- 800 slides

What are the different types of What are the different types of Emphysema and their causes?Emphysema and their causes?

• Distalacinar?Distalacinar?

• AgingAging

• Panacinar?Panacinar?

• Alpha-1 antitrypsin defAlpha-1 antitrypsin def

Page 424: USMLE Step 1 clues 2- 800 slides

What are the stages of What are the stages of erythropoiesis?erythropoiesis?• 4 mo gestation?4 mo gestation?• Yolk sacYolk sac

• 6 mo gestation?6 mo gestation?• Spleen, liver, flat bonesSpleen, liver, flat bones

Page 425: USMLE Step 1 clues 2- 800 slides

What are the stages of What are the stages of erythropoiesis?erythropoiesis?• 8 mo gestation?8 mo gestation?

• Long bonesLong bones

• 1 yr old?1 yr old?

• Long bonesLong bones

Page 426: USMLE Step 1 clues 2- 800 slides

If long bones become damaged If long bones become damaged after 1 yr what takes over?after 1 yr what takes over?

• Spleen can resume erythropoieses Spleen can resume erythropoieses causing splenomegalycausing splenomegaly

Page 427: USMLE Step 1 clues 2- 800 slides

What ions correspond with What ions correspond with the following EKG?the following EKG?•P-wave?P-wave?•Ca+Ca+

•QRS complex?QRS complex?•Na+Na+

•S-T?S-T?•Ca+Ca+

Page 428: USMLE Step 1 clues 2- 800 slides

What ions correspond with What ions correspond with the following EKG?the following EKG?

•T-wave?T-wave?

•K+K+

•U-wave?U-wave?

•Na+Na+

Page 429: USMLE Step 1 clues 2- 800 slides

What do Na+ channel What do Na+ channel blockers do to the EKG?blockers do to the EKG?

• QRSQRS

Page 430: USMLE Step 1 clues 2- 800 slides

What do Ca+ channel What do Ca+ channel blockers do to the EKG?blockers do to the EKG?

• Widens P-waveWidens P-wave

• PR interval longerPR interval longer

Page 431: USMLE Step 1 clues 2- 800 slides

P.P. Clue 4 Bio Stat P.P. Clue 4 Bio Stat DD WW

++ BB A+BA+B

__ CC C+DC+D

A+CA+C B+DB+D ALLALL

A

D

Sensitivity A/A+CSpecificity D/B+DPPV A/A+BNPV D/C+DOR AD/BC

RR (A/All)/(C/All)

AR (A/All)-(C/All)

Sensitivity: Truly Diseased PeopleSpecificity: Truly Well People

+ Predictive Value: Test +ve With DZ

- Predictive Value: Tested –ve w/o DZOR: Odds RatioRR: Relative RiskAR: Attributed Risk

Always in the numerator

A&D

Page 432: USMLE Step 1 clues 2- 800 slides

In EKG P-wave Represents?In EKG P-wave Represents?

• Atrium contractionAtrium contraction

• Phase zeroPhase zero

• CalciumCalcium

Page 433: USMLE Step 1 clues 2- 800 slides

In EKG P-R Interval means?In EKG P-R Interval means?

• AV NodeAV Node

• Phase 2Phase 2

• SodiumSodium

Page 434: USMLE Step 1 clues 2- 800 slides

In EKG Q-Wave means?In EKG Q-Wave means?

• SeptumSeptum

• Phase 2Phase 2

• SodiumSodium

Page 435: USMLE Step 1 clues 2- 800 slides

In EKG R-upstoke means?In EKG R-upstoke means?

• Anterior wallAnterior wall

• Phase 2Phase 2

• SodiumSodium

Page 436: USMLE Step 1 clues 2- 800 slides

In EKG S-down stroke In EKG S-down stroke means?means?

• Posterior wallPosterior wall

• Phase 2Phase 2

• SodiumSodium

Page 437: USMLE Step 1 clues 2- 800 slides

In EKG S-T Interval means?In EKG S-T Interval means?

• VentricleVentricle

• Phase 2Phase 2

• CalciumCalcium

Page 438: USMLE Step 1 clues 2- 800 slides

In EKG T-wave means?In EKG T-wave means?

• VentricleVentricle

• Phase 3Phase 3

• PotassiumPotassium

Page 439: USMLE Step 1 clues 2- 800 slides

In EKG U-wave meansIn EKG U-wave means

• VentricleVentricle

• Phase 4Phase 4

• SodiumSodium

Page 440: USMLE Step 1 clues 2- 800 slides

4 DZ associated with HLA-4 DZ associated with HLA-DR 2?DR 2?

• NarcolepsyNarcolepsy

• Allergy (hay fever)Allergy (hay fever)

• GoodpastureGoodpasture

• MSMS

Page 441: USMLE Step 1 clues 2- 800 slides

5 DZ associated with HLA-5 DZ associated with HLA-DR 3?DR 3?

• DMDM

• Chronic active HepititisChronic active Hepititis

• SjogrensSjogrens

• SLESLE

• Celiac sprueCeliac sprue

Page 442: USMLE Step 1 clues 2- 800 slides

DZ associated with HLA-DR DZ associated with HLA-DR 3&4?3&4?

• IDDM (DM Type 1)IDDM (DM Type 1)

Page 443: USMLE Step 1 clues 2- 800 slides

DZ associated with HLA-DR DZ associated with HLA-DR 4?4?

• Rheumatoid ArthritisRheumatoid Arthritis

• Pemphigus Vulgaris Pemphigus Vulgaris

Page 444: USMLE Step 1 clues 2- 800 slides

DZ associated with HLA-DR DZ associated with HLA-DR 55

• JRA (JUV RA)JRA (JUV RA)

• Pernicious anemiaPernicious anemia

Page 445: USMLE Step 1 clues 2- 800 slides

DZ associated with HLA-DR DZ associated with HLA-DR 7?7?

• Nephrotic syndrome (Steroid Nephrotic syndrome (Steroid induced) induced)

Page 446: USMLE Step 1 clues 2- 800 slides

DZ associated with HLA-DR DZ associated with HLA-DR 3 and HLA-B 8?3 and HLA-B 8?

• Celiac DiseaseCeliac Disease

Page 447: USMLE Step 1 clues 2- 800 slides

DZ Associated with HLA-A3?DZ Associated with HLA-A3?

• HemochromatosisHemochromatosis– chromosome 6chromosome 6– point mutation Cystine to Tyrosinepoint mutation Cystine to Tyrosine

Page 448: USMLE Step 1 clues 2- 800 slides

DZ Associated with HLA-A DZ Associated with HLA-A 3?3?

• Myasthenia gravis Myasthenia gravis

Page 449: USMLE Step 1 clues 2- 800 slides

DZ Associated with HLA-B DZ Associated with HLA-B 13?13?

• PsoriasisPsoriasis

Page 450: USMLE Step 1 clues 2- 800 slides

5 DZ Associated with HLA-B 5 DZ Associated with HLA-B 27?27?

• Psoriasis Psoriasis – only if with arthritisonly if with arthritis

• Ankylosing SpondylitiesAnkylosing Spondylities

• IBDIBD– Ulcerative CholitisUlcerative Cholitis

• Reiter’sReiter’s

• Post gonococcal arthritisPost gonococcal arthritis

Page 451: USMLE Step 1 clues 2- 800 slides

DZ Associated to HLA-BW DZ Associated to HLA-BW 4747

• 21 alpha hydroxylase deficiency21 alpha hydroxylase deficiency– Vit. DVit. D

Page 452: USMLE Step 1 clues 2- 800 slides

Facts about Diphtheria Facts about Diphtheria

• ADP ribosylates EF-2ADP ribosylates EF-2• Stops cell synthesisStops cell synthesis• Gr +ve Gr +ve • Gets exotoxin from virus via Gets exotoxin from virus via

transductiontransduction• Heart blockHeart block• Its toxoid therefore give antitoxinIts toxoid therefore give antitoxin

Page 453: USMLE Step 1 clues 2- 800 slides

MCC of Pneumonia in 6wks MCC of Pneumonia in 6wks to 18 yrs?to 18 yrs?

• RSV (infants only)RSV (infants only)

• MycoplasmaMycoplasma

• Chlamydia pneumoniaChlamydia pneumonia

• Strep pneumoniaStrep pneumonia

Page 454: USMLE Step 1 clues 2- 800 slides

MCC Pneumonia in 18 yrs to 40 MCC Pneumonia in 18 yrs to 40 yrs of age?yrs of age?

• MycoplasmaMycoplasma

• Chlamydia pneumoniaChlamydia pneumonia

• Strep. PneumoniaStrep. Pneumonia

Page 455: USMLE Step 1 clues 2- 800 slides

MCC of Pneumonia in 40 yrs MCC of Pneumonia in 40 yrs to 65 yrs of age?to 65 yrs of age?

• Strep pneumoniaStrep pneumonia

• H. influenzaH. influenza

• Anaerobes Anaerobes

Page 456: USMLE Step 1 clues 2- 800 slides

MCC of pneumonia in the MCC of pneumonia in the Elderly? Elderly?

• Strep pneumonia Strep pneumonia

• VirusesViruses

• AnaerobesAnaerobes

• H.influenzaH.influenza

• Gr –ve rodsGr –ve rods

Page 457: USMLE Step 1 clues 2- 800 slides

What are 4 Clues for IgA?What are 4 Clues for IgA?

• Monomer in bloodMonomer in blood

• Dimer in secretionDimer in secretion

• Located on mucosal surfaceLocated on mucosal surface

• Found in secretionFound in secretion

Page 458: USMLE Step 1 clues 2- 800 slides

What are Clues for IgD?What are Clues for IgD?

• Only functions as surface marker for Only functions as surface marker for Mature B-CellMature B-Cell

Page 459: USMLE Step 1 clues 2- 800 slides

What are Clues for IgE?What are Clues for IgE?

• Immediate Immediate hypersensitivity/anaphylaxishypersensitivity/anaphylaxis

• Parasite defenseParasite defense• WormsWorms• Fc region binds to mast cells and Fc region binds to mast cells and

basophilsbasophils• AllergiesAllergies• Does Not fix complementDoes Not fix complement

Page 460: USMLE Step 1 clues 2- 800 slides

What are Clues for IgG?What are Clues for IgG?

• Highest affinityHighest affinity• Memory respond at Memory respond at day 3day 3 five times the five times the

concentrationconcentration• Peaks in 5 years last for 10 yearsPeaks in 5 years last for 10 years• OpsonizesOpsonizes• Activates complementActivates complement• 22ndnd to show up in primary response to show up in primary response• Only one to show up for secondary respondOnly one to show up for secondary respond• Most abundant Ig in newbornMost abundant Ig in newborn• Antigenic differences in heavy chain and site of di-Antigenic differences in heavy chain and site of di-

sulfide bondsulfide bond• 4 subclasses G1 to G44 subclasses G1 to G4

Page 461: USMLE Step 1 clues 2- 800 slides

What are Clues for IgG1?What are Clues for IgG1?

• Crosses placenta due to fc portionCrosses placenta due to fc portion

Page 462: USMLE Step 1 clues 2- 800 slides

What are Clues for IgG2?What are Clues for IgG2?

• Most common sub-class deficiencyMost common sub-class deficiency

• Patient susceptible to encapsulated Patient susceptible to encapsulated organismsorganisms

Page 463: USMLE Step 1 clues 2- 800 slides

What are Clues for IgG3?What are Clues for IgG3?

• Most Most memorymemory antibody antibody

Page 464: USMLE Step 1 clues 2- 800 slides

What are Clues for IgG4?What are Clues for IgG4?

• Only IgG NOT fixing complementOnly IgG NOT fixing complement

Page 465: USMLE Step 1 clues 2- 800 slides

What are Clues for IgM?What are Clues for IgM?

• Responds in primary responseResponds in primary response

• Most efficient in agglutination and Most efficient in agglutination and complement fixationcomplement fixation

• Defenses against bacteria and Defenses against bacteria and virusesviruses

Page 466: USMLE Step 1 clues 2- 800 slides

What do Macrophages What do Macrophages release?release?

• MHC IIMHC II

Page 467: USMLE Step 1 clues 2- 800 slides

What does TH1 secrete? What does TH1 secrete?

• IL-2IL-2

• IF- GammaIF- Gamma

Page 468: USMLE Step 1 clues 2- 800 slides

What does TH2 Secrete?What does TH2 Secrete?

• IL-4IL-4

• IL-5IL-5

• IL-6IL-6

• IL-10IL-10

Page 469: USMLE Step 1 clues 2- 800 slides

What does TH-0 secrete?What does TH-0 secrete?

• TH-1TH-1

• TH-2TH-2

Page 470: USMLE Step 1 clues 2- 800 slides

MHC-1 are also called what? MHC-1 are also called what?

• CD8CD8

• CD8 becomes T-cytotoxic cellsCD8 becomes T-cytotoxic cells

Page 471: USMLE Step 1 clues 2- 800 slides

All T-Cells express what?All T-Cells express what?

• CD-3CD-3

• For what?For what?

• Signal transductionSignal transduction

• CD-2 CD-2

• For what?For what?

• AdherenceAdherence

Page 472: USMLE Step 1 clues 2- 800 slides

What do CD-4 cells What do CD-4 cells Become?Become?

• T helper cellsT helper cells

Page 473: USMLE Step 1 clues 2- 800 slides

What do CD-8 cells What do CD-8 cells Become?Become?

• T cytotoxic cellsT cytotoxic cells

Page 474: USMLE Step 1 clues 2- 800 slides

Neutrophils produce what Neutrophils produce what enzymes and what is their enzymes and what is their action?action?• MyeloperoxidaseMyeloperoxidase

• NADPHNADPH

• Will kill ALL Gr+veWill kill ALL Gr+ve– Ex..Hydrogen peroxide kills gr+Ex..Hydrogen peroxide kills gr+

Page 475: USMLE Step 1 clues 2- 800 slides

What do T-cells stimulate?What do T-cells stimulate?

• Clue 4x7=28Clue 4x7=28

• CD-4CD-4

• B-7B-7

• CD-28CD-28

Page 476: USMLE Step 1 clues 2- 800 slides

What are the Clues for Type-1 What are the Clues for Type-1 Hyperlipidemia?Hyperlipidemia?

• Increased ChylomicronIncreased Chylomicron

• Deficiency of Lipoprotein lipase Deficiency of Lipoprotein lipase enzymeenzyme

• Defect in liver onlyDefect in liver only

Page 477: USMLE Step 1 clues 2- 800 slides

What are the Clues for type-2 What are the Clues for type-2 hyperlipedimia?hyperlipedimia?

• Increased LDLIncreased LDL• Two types IIa and IIbTwo types IIa and IIb• Type IIa Receptor deficiency for LDL or Type IIa Receptor deficiency for LDL or

missing B-100missing B-100• Type II-b (LDL and VLDL problems) enzyme Type II-b (LDL and VLDL problems) enzyme

deficiency for LDL at adipose. Receptor deficiency for LDL at adipose. Receptor problem for VLDL. Most common in problem for VLDL. Most common in General PopulationGeneral Population

Page 478: USMLE Step 1 clues 2- 800 slides

What are the Clues for type-3 What are the Clues for type-3 Hyperlipedimia?Hyperlipedimia?

• Increased IDLIncreased IDL

• Receptor problem for APO-EReceptor problem for APO-E

Page 479: USMLE Step 1 clues 2- 800 slides

What are the Clues for Type-4 What are the Clues for Type-4 hyperlipedimia?hyperlipedimia?

• Increased VLDLIncreased VLDL

• Lipoprotein lipase enzyme deficiency Lipoprotein lipase enzyme deficiency at adipose tissue at adipose tissue

Page 480: USMLE Step 1 clues 2- 800 slides

What are the Clues for Type-5 What are the Clues for Type-5 hyperlipedimia?hyperlipedimia?

• Combination of Types 1&4Combination of Types 1&4

• Increased Chylomicron and VLDLIncreased Chylomicron and VLDL

• Enzyme and receptor deficiency at Enzyme and receptor deficiency at C-IIC-II

• Most common in diabeticsMost common in diabetics

Page 481: USMLE Step 1 clues 2- 800 slides

What is a Xanthoma?What is a Xanthoma?

• Deposition of Cholesterol on elbowsDeposition of Cholesterol on elbows

• Can cause what?Can cause what?

• CADCAD

Page 482: USMLE Step 1 clues 2- 800 slides

What is a Xanthelasma?What is a Xanthelasma?

• Deposition of Triglycerides on Deposition of Triglycerides on eyelids, faceeyelids, face

• Can cause what?Can cause what?

• PancreatitisPancreatitis

Page 483: USMLE Step 1 clues 2- 800 slides

Description of Description of RashesRashes

Page 484: USMLE Step 1 clues 2- 800 slides

ERYTHEMA MARGINATUMERYTHEMA MARGINATUM

• Little red spots w/ bright red marginsLittle red spots w/ bright red margins

• SandpaperySandpapery

• RF- Jones criteraRF- Jones critera

Page 485: USMLE Step 1 clues 2- 800 slides

ERYTHEMIA CHRONICUM ERYTHEMIA CHRONICUM MIGRANSMIGRANS

• Lymes diseaseLymes disease

• Target lesions (bulls eye)Target lesions (bulls eye)

Page 486: USMLE Step 1 clues 2- 800 slides

MEASLESMEASLES

• Morbiliform rashMorbiliform rash

• Preceded by coughPreceded by cough

• conjunctiivitisconjunctiivitis

Page 487: USMLE Step 1 clues 2- 800 slides

ROSEOLAROSEOLA

• Fever x 2 dayFever x 2 day

• Followed by rashFollowed by rash

• ONLY ONE WITH RASH FOLLOWING ONLY ONE WITH RASH FOLLOWING FEVER (HHV 6)FEVER (HHV 6)

Page 488: USMLE Step 1 clues 2- 800 slides

ERYTHEMA NODOSUMERYTHEMA NODOSUM

• Anterior aspect of legAnterior aspect of leg

• RednessRedness

• Tender nodulesTender nodules

Page 489: USMLE Step 1 clues 2- 800 slides

Erythema multiformeErythema multiforme

• Red macules, target lesionsRed macules, target lesions• Causes: allergy, virusesCauses: allergy, viruses• Mild:Mild: MCC virus, #2 drugs (sulfas) MCC virus, #2 drugs (sulfas)• Moderate:Moderate: Stevens-Johnsons Stevens-Johnsons

SyndromeSyndrome• Severe:Severe: Toxic epidermal necrolysis , Toxic epidermal necrolysis ,

skin peels offskin peels off

Page 490: USMLE Step 1 clues 2- 800 slides

SEBORRHEIC DERMATITISSEBORRHEIC DERMATITIS

• Scaly skin with oily shine on headlineScaly skin with oily shine on headline

Page 491: USMLE Step 1 clues 2- 800 slides

SEBORRHEIC KERATOSISSEBORRHEIC KERATOSIS

• Stuck on wartsStuck on warts

• Due to agingDue to aging

Page 492: USMLE Step 1 clues 2- 800 slides

PSORIASISPSORIASIS

• HLA-B27HLA-B27

• Extensor surfacesExtensor surfaces

• Silvery white plaquesSilvery white plaques

• Scaly skinScaly skin

• Pitted nailsPitted nails

Page 493: USMLE Step 1 clues 2- 800 slides

VARICELLA ZOSTER HHV 3VARICELLA ZOSTER HHV 3

• STAGESSTAGES• Red maculesRed macules• PapulesPapules• VesiclesVesicles• Pustules then scabsPustules then scabs• Different stages may appear at Different stages may appear at

same timesame time

Page 494: USMLE Step 1 clues 2- 800 slides

DERMATITIS DERMATITIS HERPATIFORMISHERPATIFORMIS

• Rash and blisters on ant. thighsRash and blisters on ant. thighs

• Assoc. with diarrheaAssoc. with diarrhea

• Assoc. with flare up of celiac sprueAssoc. with flare up of celiac sprue

Page 495: USMLE Step 1 clues 2- 800 slides

TYPHOID FEVERTYPHOID FEVER

• SEEN WITH SALMONEALLA INFXNSEEN WITH SALMONEALLA INFXN

• Rose spots assoc. with intestinal fireRose spots assoc. with intestinal fire

Page 496: USMLE Step 1 clues 2- 800 slides

DERMATOMYOSITISDERMATOMYOSITIS

• Heliotropic rashHeliotropic rash

Page 497: USMLE Step 1 clues 2- 800 slides

ERYSIPELASERYSIPELAS

• Reddened area on skin w/ raised Reddened area on skin w/ raised bordersborders

• DOES NOT BLANCHDOES NOT BLANCH

Page 498: USMLE Step 1 clues 2- 800 slides

TINEEA CRURISTINEEA CRURIS

• RednessRedness

• Itchy groinItchy groin

Page 499: USMLE Step 1 clues 2- 800 slides

PITYRIASIS ROSEAPITYRIASIS ROSEA

• Herald patch= dry skin patches that Herald patch= dry skin patches that follow skin linesfollow skin lines

• HHV 7HHV 7

Page 500: USMLE Step 1 clues 2- 800 slides

TINEA VERSICOLORTINEA VERSICOLOR

• Hypopigmented macules on upper Hypopigmented macules on upper back back

• Presents in a V patternPresents in a V pattern

• A.K.A. upside down christmas treeA.K.A. upside down christmas tree

• Tx: GriseofulvinTx: Griseofulvin

Page 501: USMLE Step 1 clues 2- 800 slides

What do you see in What do you see in SCABIES?SCABIES?

• Linear excoriations on belt line and Linear excoriations on belt line and finger websfinger webs

• What is the tx?What is the tx?

• LindaneLindane

• PermethrinPermethrin

Page 502: USMLE Step 1 clues 2- 800 slides

What is a T-CELL DEFICIENCY?What is a T-CELL DEFICIENCY?

• DiGeorges DiGeorges

• What ion imbalance will they have?What ion imbalance will they have?

• HypokalemiaHypokalemia

• What did not form?What did not form?

• 33rdrd and 4 and 4thth pharyngeal pouch pharyngeal pouch

• What chromosome?What chromosome?

• Deletion on chromosome 22Deletion on chromosome 22

Page 503: USMLE Step 1 clues 2- 800 slides

T-CELL DEFICIENCYT-CELL DEFICIENCY

• HIVHIV

• Also B-cell but less soAlso B-cell but less so

Page 504: USMLE Step 1 clues 2- 800 slides

What is MYCOSIS What is MYCOSIS FUNGOIDES?FUNGOIDES?

• NOT A FUNGUSNOT A FUNGUS

• Non-Hodgkins form of cutaneous T-Non-Hodgkins form of cutaneous T-cell lymphomacell lymphoma

Page 505: USMLE Step 1 clues 2- 800 slides

What is the job of What is the job of CHYLOMICRONS? CHYLOMICRONS?

• Transport TG’s from GI to liver and Transport TG’s from GI to liver and endotheliumendothelium

Page 506: USMLE Step 1 clues 2- 800 slides

What is the job of VLDL?What is the job of VLDL?

• Transports TG’s from liver to adiposeTransports TG’s from liver to adipose

Page 507: USMLE Step 1 clues 2- 800 slides

What is the job of IDL?What is the job of IDL?

• Transports TG’s from adipose to Transports TG’s from adipose to tissuetissue

Page 508: USMLE Step 1 clues 2- 800 slides

What is special about LDL’s?What is special about LDL’s?

• ONLY ONE THAT CARRIES ONLY ONE THAT CARRIES CHOLESTEROLCHOLESTEROL

Page 509: USMLE Step 1 clues 2- 800 slides

What do you develop with What do you develop with HYPERTRIGLYCERIDEMA?HYPERTRIGLYCERIDEMA?

• XANTHELASMAXANTHELASMA

• Where are they located?Where are they located?

• On eyelids and eyebrowsOn eyelids and eyebrows

Page 510: USMLE Step 1 clues 2- 800 slides

What do you develop with What do you develop with HYPERCHOLESTEROLEMIA?HYPERCHOLESTEROLEMIA?

• Xanthomas Xanthomas

• Where are they located?Where are they located?

• elbowselbows

Page 511: USMLE Step 1 clues 2- 800 slides

Where is VLDL made?Where is VLDL made?

• ONLY ONE MADE IN THE LIVERONLY ONE MADE IN THE LIVER

Page 512: USMLE Step 1 clues 2- 800 slides

What are IDL AND LDL formed What are IDL AND LDL formed from?from?

• ARE BREAK DOWN PRODUCTS OF ARE BREAK DOWN PRODUCTS OF VLDLVLDL

Page 513: USMLE Step 1 clues 2- 800 slides

What are the clues for What are the clues for HEMOPHILIUS INFLUENZA?HEMOPHILIUS INFLUENZA?

• Gram -/+?Gram -/+?• Pleomorphic gram (-) rodsPleomorphic gram (-) rods• What pattern?What pattern?• ““school of fish pattern”school of fish pattern”• What type is most common?What type is most common?• Type AType A

– 80%80%

Page 514: USMLE Step 1 clues 2- 800 slides

What are the clues for What are the clues for HEMOPHILIUS INFLUENZA?HEMOPHILIUS INFLUENZA?

• Capsule or no capsule?Capsule or no capsule?• non-encapsulatednon-encapsulated• Invasive or non invasive?Invasive or non invasive?• non-invasivenon-invasive

Page 515: USMLE Step 1 clues 2- 800 slides

What are the clues for What are the clues for HEMOPHILIUS INFLUENZA?HEMOPHILIUS INFLUENZA?

• Most common cause of what?Most common cause of what?• SinusitisSinusitis• OtitisOtitis• BronchitisBronchitis

Page 516: USMLE Step 1 clues 2- 800 slides

What are the clues for What are the clues for HEMOPHILIUS INFLUENZA? HEMOPHILIUS INFLUENZA? • What is the 2What is the 2ndnd most common type? most common type?• TYPE BTYPE B

– 20%20%• Encapsulated or non encapsulated?Encapsulated or non encapsulated?• EncapsulatedEncapsulated• What does it have in its capsule?What does it have in its capsule?• Polyribosyl phosphate in capsulePolyribosyl phosphate in capsule• Contains IgA proteaseContains IgA protease

Page 517: USMLE Step 1 clues 2- 800 slides

What are the clues for What are the clues for HEMOPHILIUS INFLUENZA? HEMOPHILIUS INFLUENZA?

• Invasive or non invasive?Invasive or non invasive?• Invasive Invasive • What does it cause most often? What does it cause most often? • #1 cause of epiglottitis #1 cause of epiglottitis • What are the signs of epiglottitis?What are the signs of epiglottitis?• StridorStridor• FeverFever• Thumb sign on xrayThumb sign on xray

Page 518: USMLE Step 1 clues 2- 800 slides

What are the most common What are the most common causes of MENINGITIS causes of MENINGITIS corresponding with the corresponding with the following ages?following ages?• 0-2 months?0-2 months?

• #1. Group B strep (agalactiae)#1. Group B strep (agalactiae)

• #2. E. coli#2. E. coli

• #3. Listera#3. Listera

Page 519: USMLE Step 1 clues 2- 800 slides

What are the most common What are the most common causes of MENINGITIS causes of MENINGITIS corresponding with the corresponding with the following ages?following ages?• 2 Months- 10 years?2 Months- 10 years?

• #1. strep pneumonia #1. strep pneumonia

• #2. n. meningitides#2. n. meningitides

• (adolescent years only)(adolescent years only)

Page 520: USMLE Step 1 clues 2- 800 slides

What are the most common What are the most common causes of MENINGITIS causes of MENINGITIS corresponding with the corresponding with the following ages?following ages?• 10yrs- 21 yrs?10yrs- 21 yrs?

• #1. n. meningitides#1. n. meningitides

Page 521: USMLE Step 1 clues 2- 800 slides

What are the most common What are the most common causes of MENINGITIS causes of MENINGITIS corresponding with the corresponding with the following ages?following ages?• > 21 years old?> 21 years old?

• #1 S. pneumoniae#1 S. pneumoniae

Page 522: USMLE Step 1 clues 2- 800 slides

Answer the following about the Answer the following about the Strep. Pneumonia vaccine.Strep. Pneumonia vaccine.

• At what age is it given?At what age is it given?• Given at 2,4,6 monthsGiven at 2,4,6 months• What strain does it cover?What strain does it cover?• Covers 23 strains (98% cases)Covers 23 strains (98% cases)

Page 523: USMLE Step 1 clues 2- 800 slides

Answer the following about the Answer the following about the Strep. Pneumonia vaccine.Strep. Pneumonia vaccine.

• Indications?Indications?• Anyone> 65y/oAnyone> 65y/o• Anyone splenectomized Anyone splenectomized

– Sickle cell anemiaSickle cell anemia

• Anyone with end-organ damageAnyone with end-organ damage– CFCF– RF RF – Nephrotic SyndromeNephrotic Syndrome

Page 524: USMLE Step 1 clues 2- 800 slides

STREP PYOGENES is the most STREP PYOGENES is the most common cause of what?common cause of what?

• MCC of all throat infectionsMCC of all throat infections

• #2 MCC of all what?#2 MCC of all what?

• Skin infections except linesSkin infections except lines

Page 525: USMLE Step 1 clues 2- 800 slides

What are the What are the STAPHYLOCOCCUS PIGMENTS?STAPHYLOCOCCUS PIGMENTS?

• St. aureus?St. aureus?

• Gold pigmentGold pigment

• St. epidermidis?St. epidermidis?

• White pigmentWhite pigment

• St. saprophyticus?St. saprophyticus?

• No pigmentNo pigment

Page 526: USMLE Step 1 clues 2- 800 slides

What is the clue for RUSTY What is the clue for RUSTY COLORED SPUTUM?COLORED SPUTUM?

• Strep. Pneumonia Strep. Pneumonia – pneumococcuspneumococcus

Page 527: USMLE Step 1 clues 2- 800 slides

Clues for GENERAL Clues for GENERAL INFECTIONSINFECTIONS• Skin Infections?Skin Infections?

• Say Staph. AureusSay Staph. Aureus

• Throat Infections? Throat Infections?

• Say Strep. PyogenesSay Strep. Pyogenes

• Small Intestine Infections?Small Intestine Infections?

• Say E. coliSay E. coli

Page 528: USMLE Step 1 clues 2- 800 slides

What disease is a NEUTROPHIL What disease is a NEUTROPHIL DEFICIENCY & T,B cell DEFICIENCY & T,B cell deficiency?deficiency?• Job Syndrome:Job Syndrome:

• IL-4IL-4

• Hyper IgEHyper IgE

• What do they look like?What do they look like?

• Red hairRed hair

• Fair complexionFair complexion

• FemaleFemale

Page 529: USMLE Step 1 clues 2- 800 slides

What are the NEUTROPHIL What are the NEUTROPHIL DEFICIENCY?DEFICIENCY?

• NADPH-OXIDASE DEF (CGD)NADPH-OXIDASE DEF (CGD)

• NEUTROPENIANEUTROPENIA

• MYLOPEROXIDASEMYLOPEROXIDASE

• Job-Buckley SyndromeJob-Buckley Syndrome

Page 530: USMLE Step 1 clues 2- 800 slides

What Hepatitis B antigen is What Hepatitis B antigen is found with an acute/recent found with an acute/recent infection?infection?• HbC antigenHbC antigen

• HbS antigenHbS antigen

Page 531: USMLE Step 1 clues 2- 800 slides

What Hepatitis B antigen & What Hepatitis B antigen & antibody is found with an antibody is found with an acute/recent infection?acute/recent infection?

• HbC antigenHbC antigen

• HbS antigenHbS antigen

• HbC antibodyHbC antibody

Page 532: USMLE Step 1 clues 2- 800 slides

What Hepatitis B antigen is What Hepatitis B antigen is found with Recent found with Recent immunization within the past immunization within the past 2wks?2wks?

• HbS antigen ONLYHbS antigen ONLY

Page 533: USMLE Step 1 clues 2- 800 slides

What Hepatitis B antibody is found with What Hepatitis B antibody is found with Recent immunization two wks after and can Recent immunization two wks after and can be due to vaccination immunity from a long be due to vaccination immunity from a long time ago?time ago?

• HbS antibody ONLYHbS antibody ONLY

Page 534: USMLE Step 1 clues 2- 800 slides

What Hepatitis B antibody & What Hepatitis B antibody & antigen is found with past antigen is found with past disease but now immune?disease but now immune?

• HbC antibodyHbC antibody

• HbS antibodyHbS antibody

• HbS antigenHbS antigen

Page 535: USMLE Step 1 clues 2- 800 slides

What Immunogloblin is found What Immunogloblin is found in Hepatitis B immunity?in Hepatitis B immunity?

• IgGIgG

Page 536: USMLE Step 1 clues 2- 800 slides

What Hepatitis B What Hepatitis B antigen/antibody is found in antigen/antibody is found in the chronic carrier state? the chronic carrier state?

• HbS antigen for >6monthsHbS antigen for >6months

• Can be with or without HbS antibodyCan be with or without HbS antibody

Page 537: USMLE Step 1 clues 2- 800 slides

What Hepatitis B antigen is What Hepatitis B antigen is found with the infectious state?found with the infectious state?

• HbE antigenHbE antigen

Page 538: USMLE Step 1 clues 2- 800 slides

What Hepatitis B antibody is What Hepatitis B antibody is found with the non-infectious found with the non-infectious state?state?

• HbE antibodyHbE antibody

Page 539: USMLE Step 1 clues 2- 800 slides

If patient has recovered from If patient has recovered from Hepatitis B what antigen will Hepatitis B what antigen will they have?they have?

• NEGATIVE HbS antigen NEGATIVE HbS antigen

Page 540: USMLE Step 1 clues 2- 800 slides

If patient is a chronic carrier If patient is a chronic carrier what antigen will they have?what antigen will they have?

• POSITIVE HbS antigenPOSITIVE HbS antigen

Page 541: USMLE Step 1 clues 2- 800 slides

What does the “window What does the “window period” build in Hepatitis B?period” build in Hepatitis B?

• HbE antibodyHbE antibody

• IgM HbC antibodyIgM HbC antibody

• What disappears?What disappears?

• HbS antigenHbS antigen

Page 542: USMLE Step 1 clues 2- 800 slides

What is the incubation period What is the incubation period for Hepatitis B?for Hepatitis B?

• 4 to 26 wks4 to 26 wks

• Average @ 8wksAverage @ 8wks

Page 543: USMLE Step 1 clues 2- 800 slides

How long is the acute disease How long is the acute disease period in Hepatitis B?period in Hepatitis B?

• 4 to 12 wks4 to 12 wks

Page 544: USMLE Step 1 clues 2- 800 slides

How long is the convalescence How long is the convalescence period in Hepatitis B?period in Hepatitis B?

• 4 to 20 wks4 to 20 wks

Page 545: USMLE Step 1 clues 2- 800 slides

How long is the recovery How long is the recovery period for Hepatitis B?period for Hepatitis B?

• YEARSYEARS

Page 546: USMLE Step 1 clues 2- 800 slides

Answer the following about Answer the following about HIV? HIV?

• MC infection?MC infection?

• CMVCMV

• MCC of death?MCC of death?

• PCPPCP

• What is p41 used for?What is p41 used for?

• Just a markerJust a marker

Page 547: USMLE Step 1 clues 2- 800 slides

Answer the following questions Answer the following questions about HIV? about HIV?

• What does Gp120 do? What does Gp120 do?

• Attachment to CD4Attachment to CD4

• What is Pol used for?What is Pol used for?

• IntegrationIntegration

• What is reverse transcriptase used for?What is reverse transcriptase used for?

• TranscriptionTranscription

• What are p17 & p24 antigens used for?What are p17 & p24 antigens used for?

• AssemblyAssembly

Page 548: USMLE Step 1 clues 2- 800 slides

Answer the following questions Answer the following questions about HIV? about HIV?

• What is the normal CD4 count?What is the normal CD4 count?

• 800-1200800-1200

• What can the CD4 count be up to in children?What can the CD4 count be up to in children?

• 15001500

• When do you begin treating with 2 nucleotide When do you begin treating with 2 nucleotide inhibitors and 1 protease inhibitor?inhibitors and 1 protease inhibitor?

• <500 <500 – (child at 750)(child at 750)

Page 549: USMLE Step 1 clues 2- 800 slides

Answer the following questions Answer the following questions about HIV? about HIV?

• AIDS is defined as a CD4 count of what?AIDS is defined as a CD4 count of what?• <200 <200 • With a CD4 count of <200 what do you With a CD4 count of <200 what do you

tx for?tx for?• PCPPCP• What do you treat for when CD count is What do you treat for when CD count is

<100?<100?• Mycobacterium aviam intracellularMycobacterium aviam intracellular

Page 550: USMLE Step 1 clues 2- 800 slides

What are the Antioxidants?What are the Antioxidants?

• Vitamin EVitamin E– #1#1

• Vitamin AVitamin A

• Vitamin CVitamin C

• BetakertineBetakertine

Page 551: USMLE Step 1 clues 2- 800 slides

What is Vitamin A a cofactor What is Vitamin A a cofactor for?for?

• ParathyroidParathyroid

• Along with what other cofactor?Along with what other cofactor?

• Mg+Mg+

Page 552: USMLE Step 1 clues 2- 800 slides

Too much Vitamin A will cause Too much Vitamin A will cause what?what?

• HyperparathyroidHyperparathyroid– Increase Ca+Increase Ca+– Decrease PhosphateDecrease Phosphate

• What will be the symptoms?What will be the symptoms?

• GoansGoans

• MoansMoans

• BonesBones

• StonesStones

Page 553: USMLE Step 1 clues 2- 800 slides

Too much Vitamin A will cause Too much Vitamin A will cause what?what?

• Pseudotumor CerebriPseudotumor Cerebri– Increase CSF production from Chorichoid Increase CSF production from Chorichoid

PlexusPlexus

Page 554: USMLE Step 1 clues 2- 800 slides

What is the only cause of ICP What is the only cause of ICP that does not cause that does not cause herniation?herniation?• Pseucotumor cerebriPseucotumor cerebri

Page 555: USMLE Step 1 clues 2- 800 slides

What does Vitamin A What does Vitamin A deficiency cause?deficiency cause?

• Nightvision problems/nightblindnessNightvision problems/nightblindness

• HypoparathyroidismHypoparathyroidism– Decrease Ca+Decrease Ca+– Increase PhosphateIncrease Phosphate

Page 556: USMLE Step 1 clues 2- 800 slides

What is Vitamin B1?What is Vitamin B1?

• THIAMINETHIAMINE

• What do you get with a decrease in What do you get with a decrease in thiamine?thiamine?

• Beri BeriBeri Beri

• What is the most common cause in US?What is the most common cause in US?

• ETOHETOH

Page 557: USMLE Step 1 clues 2- 800 slides

What do you get with Vitamin What do you get with Vitamin B1 deficiency?B1 deficiency?

• Wet Berry BerryWet Berry Berry

• With heart failureWith heart failure

• Dry Berry BerryDry Berry Berry

• Without heart failureWithout heart failure

Page 558: USMLE Step 1 clues 2- 800 slides

What do you get with Vitamin What do you get with Vitamin B1 deficiency?B1 deficiency?

• Wernicke’s EncephalopathyWernicke’s Encephalopathy

• Wernicke’s KorsakoffWernicke’s Korsakoff

Page 559: USMLE Step 1 clues 2- 800 slides

What is Wernicke’s What is Wernicke’s Encephalopathy?Encephalopathy?

• Alcoholic thymine deficiency of the Alcoholic thymine deficiency of the Temporal LobeTemporal Lobe

Page 560: USMLE Step 1 clues 2- 800 slides

What is Wernicke’s What is Wernicke’s Korsakoff?Korsakoff?

Page 561: USMLE Step 1 clues 2- 800 slides

What needs B1 as a What needs B1 as a Cofactor?Cofactor?

• 3 Dehydrogenases3 Dehydrogenases

• Pyruvate dehydrogenasePyruvate dehydrogenase

• Alpha ketoglutarate dehydrogenaseAlpha ketoglutarate dehydrogenase

• Branch chain amino acid Branch chain amino acid dehydrogenasedehydrogenase

• TransketolaseTransketolase

Page 562: USMLE Step 1 clues 2- 800 slides

What is B2What is B2

• RiboflavinRiboflavin

• What is a physical sign of this What is a physical sign of this deficiency?deficiency?

• Angular stomatitisAngular stomatitis

• Angular cheliosisAngular cheliosis

• Corneal NeurovasculazationsCorneal Neurovasculazations

Page 563: USMLE Step 1 clues 2- 800 slides

What is the best source of What is the best source of B2?B2?

• MilkMilk

• Also from FADAlso from FAD

Page 564: USMLE Step 1 clues 2- 800 slides

What is B3?What is B3?

• NiacinNiacin

• What is the clue?What is the clue?

• DiarrheaDiarrhea

• DermatitisDermatitis

• DementiaDementia

• DeathDeath

Page 565: USMLE Step 1 clues 2- 800 slides

What is the disease that What is the disease that presents like B3 deficiency?presents like B3 deficiency?

• Hartnup DiseaseHartnup Disease

• What is deficient in this disease?What is deficient in this disease?

• Tyrptophan Tyrptophan

• What is typtophan needed for?What is typtophan needed for?

• Needed for niacin formationNeeded for niacin formation

Page 566: USMLE Step 1 clues 2- 800 slides

What is B4?What is B4?

• Lipoic acidLipoic acid

• What is the deficiency caused by this What is the deficiency caused by this vitamin?vitamin?

• Not oneNot one

Page 567: USMLE Step 1 clues 2- 800 slides

What is B5?What is B5?

• Pantothenic acidPantothenic acid

• What is the deficiency caused by this What is the deficiency caused by this vitamin?vitamin?

• You guessed it…nothingYou guessed it…nothing

Page 568: USMLE Step 1 clues 2- 800 slides

What is B6?What is B6?

• PyridoxinePyridoxine

• What is the deficiency caused by this vitamin?What is the deficiency caused by this vitamin?

• NeuropathyNeuropathy

• Seizures Seizures

• Who do you need to give B6 to?Who do you need to give B6 to?

• Patient on INHPatient on INH

Page 569: USMLE Step 1 clues 2- 800 slides

What type of anemia is seen What type of anemia is seen with B6 Deficiency?with B6 Deficiency?

• SideroblasticSideroblastic

Page 570: USMLE Step 1 clues 2- 800 slides

What needs B6 as a What needs B6 as a cofactor?cofactor?

• ALL transaminasesALL transaminases

Page 571: USMLE Step 1 clues 2- 800 slides

What is B12?What is B12?

• CyanocobalamineCyanocobalamine

• What is the deficiency caused by this What is the deficiency caused by this vitamin?vitamin?

• Pernicious anemiaPernicious anemia

• Neuropathy Neuropathy

Page 572: USMLE Step 1 clues 2- 800 slides

What is the most common What is the most common cause of vitamin B12 cause of vitamin B12 deficiency?deficiency?• Pernicious anemiaPernicious anemia

Page 573: USMLE Step 1 clues 2- 800 slides

What 2 enzymes are needed What 2 enzymes are needed for synthesis of B12?for synthesis of B12?

• Methylmalonyl CoA MutaseMethylmalonyl CoA Mutase

• Homocysteine Methyl TransferaseHomocysteine Methyl Transferase

Page 574: USMLE Step 1 clues 2- 800 slides

Deficiency in Methylmalonyl Deficiency in Methylmalonyl CoA Mutase leads to what?CoA Mutase leads to what?

• NeuropathyNeuropathy

• Why?Why?

• Because it recycles myelin Because it recycles myelin

Page 575: USMLE Step 1 clues 2- 800 slides

Deficiency in Homocystiene Deficiency in Homocystiene Methyl Transferase leads to Methyl Transferase leads to what?what?• Megaloblastic anemiaMegaloblastic anemia

• What else is this enzyme needed for?What else is this enzyme needed for?

• Nucleotide synthesisNucleotide synthesis

Page 576: USMLE Step 1 clues 2- 800 slides

When is ANGULARE When is ANGULARE STOMATOSIS seen?STOMATOSIS seen?

• VITAMIN B2- RIBOFLAVIN deficiency VITAMIN B2- RIBOFLAVIN deficiency

Page 577: USMLE Step 1 clues 2- 800 slides

What are the 4 D’S of What are the 4 D’S of pellegra?pellegra?

• DIARRHEADIARRHEA

• DERMATITISDERMATITIS

• DEMENTIA DEMENTIA

• DEATHDEATH

Page 578: USMLE Step 1 clues 2- 800 slides

What causes a What causes a NEUROPATHY WHEN NEUROPATHY WHEN DEFICIENT & also DEFICIENT & also needs TRANSAMINASE?needs TRANSAMINASE?

• PYRIDOXINE B6PYRIDOXINE B6

Page 579: USMLE Step 1 clues 2- 800 slides

What vitamin is What vitamin is deficient with deficient with PERNICIOUS ANEMIA & PERNICIOUS ANEMIA & NEUROPATHY?NEUROPATHY?

• B12 CYANOCOBALAMINEB12 CYANOCOBALAMINE

Page 580: USMLE Step 1 clues 2- 800 slides

What is the first vitamin to run What is the first vitamin to run out with disease of rapidly out with disease of rapidly dividing cells?dividing cells?

• FolateFolate

Page 581: USMLE Step 1 clues 2- 800 slides

What type of anemia is seen What type of anemia is seen with Folate deficiency?with Folate deficiency?

• Megaloblastic anemiaMegaloblastic anemia

• With neuropathy?With neuropathy?

• NO NEUROPHATHYNO NEUROPHATHY

• What else is Folate used for?What else is Folate used for?

• Nucleotide synthase (THF)Nucleotide synthase (THF)

Page 582: USMLE Step 1 clues 2- 800 slides

What is another name for What is another name for Vitamin C?Vitamin C?

• Ascorbate acidAscorbate acid

Page 583: USMLE Step 1 clues 2- 800 slides

What is Vitamin C needed What is Vitamin C needed for?for?

• Collagen synthesisCollagen synthesis

Page 584: USMLE Step 1 clues 2- 800 slides

What happens with Vitamin C What happens with Vitamin C deficiency?deficiency?

• ScurvyScurvy

Page 585: USMLE Step 1 clues 2- 800 slides

What is the CLUE for What is the CLUE for Scurvy?Scurvy?

• Bleeding gums Bleeding gums

• Bleeding hair folliclesBleeding hair follicles

Page 586: USMLE Step 1 clues 2- 800 slides

What is the most common What is the most common cause of Vitamin C deficiency?cause of Vitamin C deficiency?

• Diet deficient in citrus fruit Diet deficient in citrus fruit

• Diet deficient in green vegetablesDiet deficient in green vegetables

• Over cooked green vegetablesOver cooked green vegetables

Page 587: USMLE Step 1 clues 2- 800 slides

What does Vitamin D do with What does Vitamin D do with Ca+?Ca+?

• Controls Ca+Controls Ca+

• Absorbes Ca+ from GIAbsorbes Ca+ from GI

• Reabsorbs Ca+ in KidneysReabsorbs Ca+ in Kidneys

• Controls osteoblastic activity Controls osteoblastic activity

Page 588: USMLE Step 1 clues 2- 800 slides

What does Vitamin D What does Vitamin D deficiency cause in Children?deficiency cause in Children?

• RickettsRicketts

• What does it cause in ADULTS?What does it cause in ADULTS?

• OsteomalciaOsteomalcia

Page 589: USMLE Step 1 clues 2- 800 slides

What is the CLUE for What is the CLUE for RICKETTS?RICKETTS?

• Lateral Bowing of the LegsLateral Bowing of the Legs

• X-linked dominantX-linked dominant

Page 590: USMLE Step 1 clues 2- 800 slides

What is Vitamin E needed What is Vitamin E needed for?for?

• HairHair

• SkinSkin

• EyesEyes

• Protection against free radicalsProtection against free radicals

• #1 antioxidant#1 antioxidant

Page 591: USMLE Step 1 clues 2- 800 slides

What does a deficiency of What does a deficiency of Vitamin E cause in newborns?Vitamin E cause in newborns?

• RetinopathyRetinopathy

Page 592: USMLE Step 1 clues 2- 800 slides

What are the vitamins from GI What are the vitamins from GI that are normal flora?that are normal flora?

• FolateFolate

• Vitamin KVitamin K– 90%90%

• BiotinBiotin

• Panothenic acidPanothenic acid

• Helps with absorption of B12Helps with absorption of B12

Page 593: USMLE Step 1 clues 2- 800 slides

What are the Vitamin K What are the Vitamin K dependent clotting factors?dependent clotting factors?• 19721972• Protein CProtein C• Protein SProtein S

• Which one has the shortest half-life?Which one has the shortest half-life?• Protein CProtein C

• Which one has the 2Which one has the 2ndnd shortest half-life? shortest half-life?• 77

Page 594: USMLE Step 1 clues 2- 800 slides

What are the TRACE What are the TRACE elements?elements?

• ChromiumChromium

• SeleniumSelenium

• Manganese MolebdenumManganese Molebdenum

• TinTin

Page 595: USMLE Step 1 clues 2- 800 slides

What is Chromium needed What is Chromium needed for?for?

• Insulin actionInsulin action

Page 596: USMLE Step 1 clues 2- 800 slides

What organ needs What organ needs Selenium?Selenium?

• HeartHeart

Page 597: USMLE Step 1 clues 2- 800 slides

What trace element is an What trace element is an enzyme in glycolsis?enzyme in glycolsis?

• Manganese MolebdenumManganese Molebdenum

Page 598: USMLE Step 1 clues 2- 800 slides

What organ needs Tin?What organ needs Tin?

• HairHair

Page 599: USMLE Step 1 clues 2- 800 slides

What does a deficiency in Zinc What does a deficiency in Zinc cause?cause?

• DysguziaDysguzia

• Decrease spermDecrease sperm

• Dry hairDry hair

• Dry skinDry skin

Page 600: USMLE Step 1 clues 2- 800 slides

Cofactor for ALL Kinases?Cofactor for ALL Kinases?

• Mg+Mg+

Page 601: USMLE Step 1 clues 2- 800 slides

Cofactor for ALL Cofactor for ALL Carboxylases?Carboxylases?

• BiotinBiotin

Page 602: USMLE Step 1 clues 2- 800 slides

Cofactor for ALL Cofactor for ALL Transaminases?Transaminases?

• PyridoxinePyridoxine– B6B6

Page 603: USMLE Step 1 clues 2- 800 slides

What is Biotin a cofactor What is Biotin a cofactor for?for?

• ALL carboxylases ALL carboxylases

Page 604: USMLE Step 1 clues 2- 800 slides

What is Mg+ a cofactor for?What is Mg+ a cofactor for?

• ALL kinasesALL kinases

• Parathyroid along with Vitamin AParathyroid along with Vitamin A

Page 605: USMLE Step 1 clues 2- 800 slides

What is Ca+ needed for?What is Ca+ needed for?

• Muscle contractionMuscle contraction

• Axonal transportAxonal transport

• 22ndnd messengers messengers

Page 606: USMLE Step 1 clues 2- 800 slides

What tracts are affected due to What tracts are affected due to deficiency in Methyl Malonyl deficiency in Methyl Malonyl CoA Mutase?CoA Mutase?

• Dorsal ColumnsDorsal Columns

• Cortical Spinal TractsCortical Spinal Tracts

• Why are these affected?Why are these affected?

• Because they are the longest Because they are the longest

• Because they need the most myelin Because they need the most myelin

Page 607: USMLE Step 1 clues 2- 800 slides

What enzyme does Zanthein What enzyme does Zanthein Oxidase need?Oxidase need?

• Maganese MolebdenumMaganese Molebdenum

Page 608: USMLE Step 1 clues 2- 800 slides

How are drugs that are How are drugs that are bioavailable ALWAYS excreted?bioavailable ALWAYS excreted?

• By the liverBy the liver

• Always HepatotoxicAlways Hepatotoxic

Page 609: USMLE Step 1 clues 2- 800 slides

How are soluble drugs ALWAYS How are soluble drugs ALWAYS excreted?excreted?

• By the kidneyBy the kidney

• Always nephrotoxicAlways nephrotoxic

Page 610: USMLE Step 1 clues 2- 800 slides

What are the 5 P’S OF What are the 5 P’S OF COMPARTMENT SYNDROME?COMPARTMENT SYNDROME?

• PainPain

• PallorPallor

• ParesthesiaParesthesia

• PulselessnessPulselessness

• PoikilothermiaPoikilothermia

Page 611: USMLE Step 1 clues 2- 800 slides

What are 5 skin infections What are 5 skin infections were Strep. Pyogenes is the were Strep. Pyogenes is the number one cause?number one cause?• LympangitisLympangitis

• Impetigo (not bullous)Impetigo (not bullous)

• Necrotizing fascitisNecrotizing fascitis

• ErysepelasErysepelas

• Scarlet feverScarlet fever

Page 612: USMLE Step 1 clues 2- 800 slides

What are 5 skin infections were What are 5 skin infections were Staph. aureus is the number Staph. aureus is the number two cause?two cause?• LympangitisLympangitis

• Impetigo (not bullous)Impetigo (not bullous)

• Necrotizing fascitisNecrotizing fascitis

• ErysepelasErysepelas

• Scarlet feverScarlet fever

Page 613: USMLE Step 1 clues 2- 800 slides

What is the #1 bacteria What is the #1 bacteria causing infection associated in causing infection associated in shunts and central lines?shunts and central lines?

• Staph epidermitisStaph epidermitis

Page 614: USMLE Step 1 clues 2- 800 slides

What is the #1 bacteria What is the #1 bacteria causing infection in peripheral causing infection in peripheral lines?lines?• Staph aureusStaph aureus

Page 615: USMLE Step 1 clues 2- 800 slides

Why do we need E. COLI in the Why do we need E. COLI in the gut?gut?

• Absorption ofAbsorption of Vit. B12 Vit. B12

• Synthesis ofSynthesis of::

• -Vitamin K-Vitamin K

• -Biotin-Biotin

• -Folate-Folate

• -Pantothenic acid -Pantothenic acid – B5B5

Page 616: USMLE Step 1 clues 2- 800 slides

Answer the following questions Answer the following questions about RESTRICTION ENZYMES?about RESTRICTION ENZYMES?• Trypsin cuts where?Trypsin cuts where?• cuts to R of cuts to R of • ArgArg• LysLys

• Chymotrypsin cuts where?Chymotrypsin cuts where?• cuts to R of bulky aa’s cuts to R of bulky aa’s (aromatics)(aromatics)• PhePhe• TyrTyr• TrpTrp

Page 617: USMLE Step 1 clues 2- 800 slides

Answer the following questions Answer the following questions about RESTRICTION ENZYMES?about RESTRICTION ENZYMES?

•Elastase cuts where?Elastase cuts where?• Cuts to R of (“SAG”)Cuts to R of (“SAG”)• SSerer• AAlala• GGlyly

• CNBr cuts where?CNBr cuts where?• Cuts to R ofCuts to R of• MethionineMethionine

Page 618: USMLE Step 1 clues 2- 800 slides

Answer the following questions Answer the following questions about RESTRICTION ENZYMES?about RESTRICTION ENZYMES?

• Mercaptoethanol cuts where?Mercaptoethanol cuts where?

• Cuts to R of: disulfide bonds Cuts to R of: disulfide bonds – CysteineCysteine– methioninemethionine

Page 619: USMLE Step 1 clues 2- 800 slides

Answer the following questions Answer the following questions about RESTRICTION ENZYMES?about RESTRICTION ENZYMES?

• Aminopeptidase cuts where?Aminopeptidase cuts where?• Cuts to R ofCuts to R of• amino acid terminalamino acid terminal

• Caboxypeptidase cuts where?Caboxypeptidase cuts where?

• Cuts to L ofCuts to L of

• carboxy terminal carboxy terminal

Page 620: USMLE Step 1 clues 2- 800 slides

What is THE What is THE ONLY LIVE ONLY LIVE VACCINE INDICATED IN VACCINE INDICATED IN AIDS AIDS PATIENTS?PATIENTS?• MMRMMR

Page 621: USMLE Step 1 clues 2- 800 slides

What VACCINE is What VACCINE is NOTNOT GIVEN IF GIVEN IF pt. is Allergic to EGG?pt. is Allergic to EGG?

• MMR & INFLUENZAMMR & INFLUENZA

Page 622: USMLE Step 1 clues 2- 800 slides

What VACCINE is What VACCINE is NOT NOT GIVEN GIVEN IF patient HAS YEAST IF patient HAS YEAST ALLERGIES?ALLERGIES?• Hepatitis BHepatitis B

Page 623: USMLE Step 1 clues 2- 800 slides

What 3 VACCINES DROP OUT What 3 VACCINES DROP OUT AFTER 6 YEARS OF AGE?AFTER 6 YEARS OF AGE?

• HibHib

• DiphtheriaDiphtheria

• PertussisPertussis

Page 624: USMLE Step 1 clues 2- 800 slides

What is the MC STRAIN OF What is the MC STRAIN OF STREP PYOGENES TO CAUSE STREP PYOGENES TO CAUSE GN?GN?• Strain 12Strain 12

Page 625: USMLE Step 1 clues 2- 800 slides

What 2 substances are in What 2 substances are in NEUTROPHILS?NEUTROPHILS?

• MyeloperoxidaseMyeloperoxidase

• NADPH OxidaseNADPH Oxidase

Page 626: USMLE Step 1 clues 2- 800 slides

MACROPHAGES CONTAIN what MACROPHAGES CONTAIN what SUBSTANCE?SUBSTANCE?

• NADPH OxidaseNADPH Oxidase

• Which means they only kill what?Which means they only kill what?

• Kills only G -veKills only G -ve

Page 627: USMLE Step 1 clues 2- 800 slides

What do MACROPHAGES What do MACROPHAGES SECRETE?SECRETE?

• IL-1IL-1

• IL-6??IL-6??

Page 628: USMLE Step 1 clues 2- 800 slides

What DRUGS CAUSE PAINFUL What DRUGS CAUSE PAINFUL NEUROPATHY?NEUROPATHY?

• DDI>DDCDDI>DDC

• PancreatitisPancreatitis

Page 629: USMLE Step 1 clues 2- 800 slides

What are the MITOCHONDRIAL What are the MITOCHONDRIAL DISEASES?DISEASES?• Leigh’s DiseaseLeigh’s Disease• What is another name?What is another name?• Subacute necrotizing encephalomyelopathySubacute necrotizing encephalomyelopathy• What are the signs & symptoms?What are the signs & symptoms?• Progressively decreasing IQ Progressively decreasing IQ • SeizureSeizure• AtaxiaAtaxia• What is the deficiency?What is the deficiency?• Cytochrome oxidase deficiencyCytochrome oxidase deficiency

Page 630: USMLE Step 1 clues 2- 800 slides

What are the MITOCHONDRIAL What are the MITOCHONDRIAL DISEASES?DISEASES?• Leber’s Hereditary Optic Atrophy (LHON)Leber’s Hereditary Optic Atrophy (LHON)• They all dieThey all die

Page 631: USMLE Step 1 clues 2- 800 slides

What is the What is the ONLYONLY G +ve WITH G +ve WITH ENDOTOXIN?ENDOTOXIN?

•ListeriaListeria

• What part is toxic?What part is toxic?• Lipid ALipid A

• Does it cross the placenta?Does it cross the placenta?• YesYes

Page 632: USMLE Step 1 clues 2- 800 slides

What does Listeria What does Listeria activate?activate?

• T-cells & Macrophages, T-cells & Macrophages, therefore, therefore, have granulomashave granulomas

Page 633: USMLE Step 1 clues 2- 800 slides

What are the Associations in What are the Associations in contracting the Listeria bug?contracting the Listeria bug?

• Raw cabbageRaw cabbage

• Spoiled milkSpoiled milk

• Migrant workersMigrant workers

Page 634: USMLE Step 1 clues 2- 800 slides

What are the PERIODS OF What are the PERIODS OF RAPID GROWTH/RAPIDLY RAPID GROWTH/RAPIDLY DIVIDING CELLS?DIVIDING CELLS?

• Birth – 2 monthsBirth – 2 months

• 4 – 7 years old4 – 7 years old

• PubertyPuberty

Page 635: USMLE Step 1 clues 2- 800 slides

What is THE What is THE ONLY ONLY IMMUNE IMMUNE DEFICIENCY WITH DEFICIENCY WITH LOW CALCIUM LOW CALCIUM and Increase Phosphate?and Increase Phosphate?• DiGeorge’s SyndromeDiGeorge’s Syndrome

Page 636: USMLE Step 1 clues 2- 800 slides

What are the BASIC AMINO What are the BASIC AMINO ACIDS?ACIDS?

• LysineLysine

• ArginineArginine

Page 637: USMLE Step 1 clues 2- 800 slides

What are the ACIDIC AMINO What are the ACIDIC AMINO ACIDS?ACIDS?

• GlutamateGlutamate

• AspartateAspartate

Page 638: USMLE Step 1 clues 2- 800 slides

What are the Ketogenic + What are the Ketogenic + Glucogenic Amino Acids?Glucogenic Amino Acids?

• PhenylalaninePhenylalanine

• IsoleucineIsoleucine

• TrptophanTrptophan

• ThreonineThreonine

Page 639: USMLE Step 1 clues 2- 800 slides

What are the AROMATIC What are the AROMATIC AMINO ACIDS?AMINO ACIDS?

• PhenylalaninPhenylalanin

• TyrosineTyrosine

• TryptophanTryptophan

Page 640: USMLE Step 1 clues 2- 800 slides

What are the AMINO ACIDS What are the AMINO ACIDS with DISULFIDE BONDS?with DISULFIDE BONDS?

• MetMet

• CystCyst

Page 641: USMLE Step 1 clues 2- 800 slides

What are the “KINKY” AMINO What are the “KINKY” AMINO ACID?ACID?

• ProlineProline

Page 642: USMLE Step 1 clues 2- 800 slides

What are the SMALLEST AMINO What are the SMALLEST AMINO ACID?ACID?

• GlyGly

Page 643: USMLE Step 1 clues 2- 800 slides

What are the AMINO ACIDS What are the AMINO ACIDS with O-BONDS?with O-BONDS?

• SerineSerine

• ThreonineThreonine

• TyrosineTyrosine

Page 644: USMLE Step 1 clues 2- 800 slides

What are the AMINO ACIDS What are the AMINO ACIDS with N-BONDS?with N-BONDS?

• AsparagineAsparagine

• GlutamineGlutamine

Page 645: USMLE Step 1 clues 2- 800 slides

What are the BRANCHED-What are the BRANCHED-CHAIN AMINO ACIDS?CHAIN AMINO ACIDS?

• LeuLeu

• IsoIso

• ValVal

Page 646: USMLE Step 1 clues 2- 800 slides

What are the KETOGENIC What are the KETOGENIC AMINO ACIDS?AMINO ACIDS?

• LeuLeu

• LysLys

Page 647: USMLE Step 1 clues 2- 800 slides

What CONDITIONS are What CONDITIONS are ASSOCIATED WITH ASSOCIATED WITH HLA-B27?HLA-B27?• Psoriasis (with arthritis)Psoriasis (with arthritis)

• Ankylosing SpondylitisAnkylosing Spondylitis

• Irritable Bowel SyndromeIrritable Bowel Syndrome

• Reiter’s SyndromeReiter’s Syndrome

Page 648: USMLE Step 1 clues 2- 800 slides

What is associated with HLA-What is associated with HLA-B13?B13?

• Psoriasis with out arthritisPsoriasis with out arthritis

Page 649: USMLE Step 1 clues 2- 800 slides

What are the ORGANISMS What are the ORGANISMS WITH IgA PROTEASE WITH IgA PROTEASE (resistant to (resistant to

IgA)IgA)??• Strep. PneumoniaeStrep. Pneumoniae

• H. influenzaH. influenza

• Neisseria catarrhalisNeisseria catarrhalis

Page 650: USMLE Step 1 clues 2- 800 slides

What do EOSINOPHILS What do EOSINOPHILS SECRETE?SECRETE?

• HistaminaseHistaminase

• ArylsulfataseArylsulfatase

• HeparinHeparin

• Major Basic ProteinMajor Basic Protein

Page 651: USMLE Step 1 clues 2- 800 slides

What do MAST CELLS What do MAST CELLS SECRETE?SECRETE?

• In an Acute Reaction?In an Acute Reaction?

• HistamineHistamine

• In a Late ReactionIn a Late Reaction ??

• SRS-ASRS-A

• ECF-AECF-A

Page 652: USMLE Step 1 clues 2- 800 slides

What is the MCC of ATYPICAL What is the MCC of ATYPICAL PNEUMONIA?PNEUMONIA?

• 0 – 2 months?0 – 2 months?

• chlamydia pneumoniachlamydia pneumonia

Page 653: USMLE Step 1 clues 2- 800 slides

What does chlamydia What does chlamydia pneumonia cause?pneumonia cause?

• Intersitital pneumoniaIntersitital pneumonia

Page 654: USMLE Step 1 clues 2- 800 slides

What is the CLUE for HEART What is the CLUE for HEART BLOCK?BLOCK?

• High temperature with High temperature with NORMAL NORMAL pulse rate!pulse rate!

((This should never be!This should never be! Each degree Each degree ↑ in temp. → ↑ in temp. → 10 beats/min ↑ in pulse rate)10 beats/min ↑ in pulse rate)

Page 655: USMLE Step 1 clues 2- 800 slides

What are the clues for IL-1?What are the clues for IL-1?

• FEVERFEVER

• NONSPECIFIC ILLNESSNONSPECIFIC ILLNESS

• RECRUITS TH CELLS for LINKING with RECRUITS TH CELLS for LINKING with MHC II COMPLEXMHC II COMPLEX

• SECRETED BY MACROPHAGESSECRETED BY MACROPHAGES

Page 656: USMLE Step 1 clues 2- 800 slides

What are the clues for IL-2?What are the clues for IL-2?

• MOST POTENT OF THE InterleukinsMOST POTENT OF THE Interleukins

• RECRUITS EVERYBODYRECRUITS EVERYBODY

• MOST POWERFUL MOST POWERFUL CHEMO-ATTRACTANTCHEMO-ATTRACTANT

• MUST MUST BE INACTIVATEDBE INACTIVATED

• When must you inactivate it?When must you inactivate it?

• PRIOR TO TRANSPLANTATIONPRIOR TO TRANSPLANTATIONby cyclosporinby cyclosporin

• SECRETED BY TH1 CELLSSECRETED BY TH1 CELLS

Page 657: USMLE Step 1 clues 2- 800 slides

What are the clues for IL-3?What are the clues for IL-3?

• ENERGIZED MACROPHAGESENERGIZED MACROPHAGES

• CAUSES B-CELL PROLIFERATIONCAUSES B-CELL PROLIFERATION

• LABELED BY THYMIDINE (USE LABELED BY THYMIDINE (USE POKEWEED MITOGEN OR POKEWEED MITOGEN OR ENDOTOXIN)ENDOTOXIN)

• SECRETED BY ACTIVATED T CELLSSECRETED BY ACTIVATED T CELLS

Page 658: USMLE Step 1 clues 2- 800 slides

What are the clues for IL-4?What are the clues for IL-4?

• B-CELL DIFFERENTIATIONB-CELL DIFFERENTIATION

• RESPONSIBLE FOR CLASS SWITCHINGRESPONSIBLE FOR CLASS SWITCHING

• SECRETED BY TH2 CELLSSECRETED BY TH2 CELLS

Page 659: USMLE Step 1 clues 2- 800 slides

What are the clues for IL-5 thru What are the clues for IL-5 thru 14?14?

• They do exactly what IL-1 thru IL4 They do exactly what IL-1 thru IL4

Page 660: USMLE Step 1 clues 2- 800 slides

What are the clues for IL-What are the clues for IL-10?10?

• SUPPRESSES CELL-MEDIATED SUPPRESSES CELL-MEDIATED RESPONSE (tells macrophages and RESPONSE (tells macrophages and fibroblasts to stay away if bacterial)fibroblasts to stay away if bacterial)

• INHIBITS INHIBITS MACMAC ACTIVATION ACTIVATION

Page 661: USMLE Step 1 clues 2- 800 slides

What are the clues for IL-What are the clues for IL-12?12?• PROMOTES CELL-MEDIATED RESPONSE PROMOTES CELL-MEDIATED RESPONSE

(recruits macs & fibroblasts if NOT (recruits macs & fibroblasts if NOT bacterial)bacterial)

• ACTIVATES ACTIVATES NKNK CELLS TO SECRETE IF- CELLS TO SECRETE IF-γγ

• INHIBITS IL-4 INDUCED IgE SECRETIONINHIBITS IL-4 INDUCED IgE SECRETION

• CHANGES TH CELLS to TH1 CELLS CHANGES TH CELLS to TH1 CELLS – secretes IL-2 & IF-secretes IL-2 & IF-γγ → inhib. TH2, therefore, ↑ → inhib. TH2, therefore, ↑

host defenses against delayed hypersensitivityhost defenses against delayed hypersensitivity

Page 662: USMLE Step 1 clues 2- 800 slides

What are the clues for IF-What are the clues for IF-αα??

• Where is it from?Where is it from?• LEUKOCYTESLEUKOCYTES• ↓↓VIRAL REPLICATION AND TUMOR GROWTHVIRAL REPLICATION AND TUMOR GROWTH• ↑ ↑ NK NK ACTVITY ACTVITY

– secretes perforins and granzymes to kill infected secretes perforins and granzymes to kill infected cellcell

• ↑↑MHC CLASS I & II EXPRESSIONMHC CLASS I & II EXPRESSION• ↓ ↓ PROTEIN SYNTHESISPROTEIN SYNTHESIS

– translation inhibited, therefore, defective protein translation inhibited, therefore, defective protein synthesissynthesis

Page 663: USMLE Step 1 clues 2- 800 slides

Summary of clues for IF-Summary of clues for IF-αα??

• Increase NK activityIncrease NK activity

• Increase MHC class I & IIIncrease MHC class I & II

• Decrease protein synthesisDecrease protein synthesis

• Decrease viral replication and growthDecrease viral replication and growth

Page 664: USMLE Step 1 clues 2- 800 slides

What are the clues for IF-B?What are the clues for IF-B?

• Where is it from?Where is it from?

• FIBROBLASTSFIBROBLASTS

• Increase NK activityIncrease NK activity

• Increase MHC class I & IIIncrease MHC class I & II

• Decrease protein synthesisDecrease protein synthesis

• Decrease viral replication and growthDecrease viral replication and growth

Page 665: USMLE Step 1 clues 2- 800 slides

What are the clues for IF-What are the clues for IF-gama?gama?• Where is it from?Where is it from?

• T-CELLS & NK CELLST-CELLS & NK CELLS

• ↑ ↑ NK NK ACTIVITYACTIVITY

• ↑↑MHC MHC CLASS I & IICLASS I & II

• ↑ ↑ MACROPHAGE ACTIVITYMACROPHAGE ACTIVITY

• CO-STIMULATES B-CELL GROWTH & CO-STIMULATES B-CELL GROWTH & DIFFERENTIATIONDIFFERENTIATION

• ↓ ↓ IgE SECRETIONIgE SECRETION

Page 666: USMLE Step 1 clues 2- 800 slides

What are the clues for TNF-What are the clues for TNF-alpha?alpha?• Where is it from?Where is it from?• MONOCYTES & MONOCYTES & MACROPHAGESMACROPHAGES• What is another name for TNF-alpha?What is another name for TNF-alpha?• CACHECTINCACHECTIN• INDUCES INDUCES IL-1IL-1• ↑ ↑ ADHESION MOLECULES & MHC CLASS I ON ADHESION MOLECULES & MHC CLASS I ON

ENDOTHELIAL CELLSENDOTHELIAL CELLS• PYROGENPYROGEN• INDUCES IF-INDUCES IF-γγ SECRETION SECRETION• CYTOTOXICCYTOTOXIC/CYTOSTATIC EFFECT/CYTOSTATIC EFFECT

Page 667: USMLE Step 1 clues 2- 800 slides

What are the clues for TNF-What are the clues for TNF-beta?beta?

• Where is it from?Where is it from?

• T-CELLST-CELLS

• What is another name for it?What is another name for it?

• LYMPHOTOXINLYMPHOTOXIN

• CYTOTOXIC FACTOR CYTOTOXIC FACTOR

Page 668: USMLE Step 1 clues 2- 800 slides

What are the clues for TGF-What are the clues for TGF-αα??

• Where is it from?Where is it from?

• SOLID TUMORS SOLID TUMORS (CARCINOMA > (CARCINOMA > SARCOMA)SARCOMA)

• MONOCYTESMONOCYTES

• What is another name for it?What is another name for it?

• TRANSFORMING GROWTH FACTORSTRANSFORMING GROWTH FACTORS

Page 669: USMLE Step 1 clues 2- 800 slides

What are the clues for TGF-What are the clues for TGF-αα??• What does it INDUCE?What does it INDUCE?

• ANGIOGENESISANGIOGENESIS

• KERATINOCYTE PROLIFERATIONKERATINOCYTE PROLIFERATION

• BONE RESORPTIONBONE RESORPTION

• TUMOR GROWTHTUMOR GROWTH

• What is it mainly for?What is it mainly for?

• MAINLY FOR TUMOR GROWTHMAINLY FOR TUMOR GROWTH

Page 670: USMLE Step 1 clues 2- 800 slides

What are the clues for TGF-What are the clues for TGF-ββ??

• Where is it from?Where is it from?

• PLATELETSPLATELETS

• PLACENTAPLACENTA

• KIDNEYKIDNEY

• BONEBONE

• T & B CELLST & B CELLS

Page 671: USMLE Step 1 clues 2- 800 slides

What are the clues for TGF-What are the clues for TGF-ββ??

• What What INDUCES it?INDUCES it?

• FIBROBLAST PROLIFERATIONFIBROBLAST PROLIFERATION

• COLLAGENCOLLAGEN

• FIBRONECTIN SYNTHESISFIBRONECTIN SYNTHESIS

Page 672: USMLE Step 1 clues 2- 800 slides

What are the clues for TGF-What are the clues for TGF-ββ??

• What INHIBITS it?What INHIBITS it?

• NKNK

• LAKLAK

• CTLCTL

• T & B CELL PROLIFERATIONT & B CELL PROLIFERATION

Page 673: USMLE Step 1 clues 2- 800 slides

What are the clues for TGF-What are the clues for TGF-ββ??

• What ENHANCES it?What ENHANCES it?

• WOUND HEALING WOUND HEALING

• ANGIOGENESISANGIOGENESIS

Page 674: USMLE Step 1 clues 2- 800 slides

What are the clues for TGF-What are the clues for TGF-ββ??

• What does it suppress?What does it suppress?

• SUPPRESSES IR AFTER INFECTION & SUPPRESSES IR AFTER INFECTION & PROMOTES HEALING PROCESSPROMOTES HEALING PROCESS

• What is it mainly for?What is it mainly for?

• MAINLY FOR WOUND HEALINGMAINLY FOR WOUND HEALING

Page 675: USMLE Step 1 clues 2- 800 slides

What does LAK stand for?What does LAK stand for?

• LYMPHOKINE ACTIVATED KILLER LYMPHOKINE ACTIVATED KILLER CELLSCELLS

Page 676: USMLE Step 1 clues 2- 800 slides

What does CTL stand for?What does CTL stand for?

• CYTOTOXIC T-LYMPHOCYTESCYTOTOXIC T-LYMPHOCYTES

Page 677: USMLE Step 1 clues 2- 800 slides

What does mitochondrial What does mitochondrial inheritance affect?inheritance affect?

• CNSCNS

• HeartHeart

• Skeletal muscleSkeletal muscle

• Why does it affect these particular Why does it affect these particular places?places?

• Due to uneven cytokinesis during Due to uneven cytokinesis during meiosis or oogenesismeiosis or oogenesis

Page 678: USMLE Step 1 clues 2- 800 slides

Answer the following about Answer the following about Mitochondrial diseases?Mitochondrial diseases?

• Who are affected?Who are affected?

• All offspringAll offspring

• Who passes the disease?Who passes the disease?

• MOMMOM

• Who has no transmission?Who has no transmission?

• DadDad

Page 679: USMLE Step 1 clues 2- 800 slides

Answer the following about Answer the following about Autosomal Recessive Autosomal Recessive inheritance?inheritance?• Who does it show in?Who does it show in?• Not parentsNot parents• Siblings/uncles may show diseaseSiblings/uncles may show disease

• When is onset?When is onset?• Early in life (childhood diagnosis)Early in life (childhood diagnosis)

• Is it complete on incomplete penetrance?Is it complete on incomplete penetrance?• COMPLETECOMPLETE

Page 680: USMLE Step 1 clues 2- 800 slides

Answer the following about Answer the following about Autosomal Recessive Autosomal Recessive inheritance?inheritance?• How are they acquired?How are they acquired?

• Almost ALL are inborn error of Almost ALL are inborn error of metabolismmetabolism

• When does it occur?When does it occur?

• Only when both alleles at a locus are Only when both alleles at a locus are mutantmutant

Page 681: USMLE Step 1 clues 2- 800 slides

Answer the following about Answer the following about Autosomal Recessive Autosomal Recessive inheritance?inheritance?• How is it transmitted?How is it transmitted?

• Horizontal TmHorizontal Tm

• Are there malformations present?Are there malformations present?

• Physical malformations are uncommonPhysical malformations are uncommon

• What type of defect?What type of defect?

• Enzyme defectEnzyme defect

Page 682: USMLE Step 1 clues 2- 800 slides

Answer the following about Answer the following about Autosomal Dominant Autosomal Dominant inheritance?inheritance?• Who does it affect the most?Who does it affect the most?

• M=FM=F

• How does it manifest?How does it manifest?

• Heterozygote stateHeterozygote state

• Who can transmit the disease?Who can transmit the disease?

• Both parentsBoth parents

Page 683: USMLE Step 1 clues 2- 800 slides

Answer the following about Answer the following about Autosomal Dominant Autosomal Dominant inheritance?inheritance?• Where is the new mutation?Where is the new mutation?

• Often in germ cells of older fathersOften in germ cells of older fathers

• When is onset?When is onset?

• Often delayed (adult diagnosis)Often delayed (adult diagnosis)– Example = Huntington’sExample = Huntington’s

Page 684: USMLE Step 1 clues 2- 800 slides

Answer the following about Answer the following about Autosomal Dominant Autosomal Dominant inheritance?inheritance?• What is penetrance?What is penetrance?

• Reduced penetranceReduced penetrance

• How is it expressed?How is it expressed?

• Variable expressinVariable expressin– Different in each individualDifferent in each individual

Page 685: USMLE Step 1 clues 2- 800 slides

Answer the following about Answer the following about Autosomal Dominant Autosomal Dominant inheritance?inheritance?• How is it transmitted?How is it transmitted?

• Vertical TMVertical TM

• Is there malformation present?Is there malformation present?

• Physical malformation commonPhysical malformation common

• What type of defect?What type of defect?

• Structural Structural

Page 686: USMLE Step 1 clues 2- 800 slides

Who is affected in the family Who is affected in the family with an X-Linked disease?with an X-Linked disease?

• Maternal grandfatherMaternal grandfather

• Maternal uncleMaternal uncle

Page 687: USMLE Step 1 clues 2- 800 slides

Immune System Time Line for Immune System Time Line for viral & cell-mediated.viral & cell-mediated.

• What happens <24hrs?What happens <24hrs?

• SwellingSwelling

• What happens at 24 hrs?What happens at 24 hrs?

• Neutrophils show upNeutrophils show up

• What happens at day 3?What happens at day 3?

• Neutrophils peakNeutrophils peak

Page 688: USMLE Step 1 clues 2- 800 slides

Immune System Time Line for Immune System Time Line for viral & cell-mediated.viral & cell-mediated.

• What happens at day 4?What happens at day 4?

• T cells and Macrophages show upT cells and Macrophages show up

• What happens at day 7?What happens at day 7?

• Fibroblasts show upFibroblasts show up

• What happens in 1 month?What happens in 1 month?

• Fibroblast peakFibroblast peak

Page 689: USMLE Step 1 clues 2- 800 slides

Immune System Time Line for Immune System Time Line for viral & cell-mediated.viral & cell-mediated.

• What happens at 3-6 months?What happens at 3-6 months?

• Fibroblasts are goneFibroblasts are gone

Page 690: USMLE Step 1 clues 2- 800 slides

What is the general CLUE for What is the general CLUE for any Lysosomal Storage any Lysosomal Storage Disease?Disease?

• Lysosomal Inclusion BodiesLysosomal Inclusion Bodies

Page 691: USMLE Step 1 clues 2- 800 slides

What are the Lysosomal What are the Lysosomal Storage diseases?Storage diseases?• GauchersGauchers• FabrysFabrys• KrabbeKrabbe• Tay SachsTay Sachs• SandhoffsSandhoffs• HurlersHurlers• HuntersHunters• Neiman PickNeiman Pick• Metachromatic LeukodystropyMetachromatic Leukodystropy

Page 692: USMLE Step 1 clues 2- 800 slides

What is missing in What is missing in Gauchers?Gauchers?• Beta-GlucocerberosidaseBeta-Glucocerberosidase

• What Accumulates?What Accumulates?• GlucocebrosideGlucocebroside

• Where?Where?• BrainBrain• LiverLiver• Bone MarrowBone Marrow• Spleen Spleen

Page 693: USMLE Step 1 clues 2- 800 slides

What are the CLUES for What are the CLUES for Gauchers?Gauchers?

• Ask. JewAsk. Jew

• GargolsGargols

• Gaucher cellsGaucher cells– Macrophages looking like Crinkeled Macrophages looking like Crinkeled

paperpaper

• Erlin myoflask legsErlin myoflask legs

• PseudohypertrophyPseudohypertrophy

Page 694: USMLE Step 1 clues 2- 800 slides

What is missing in Fabrys?What is missing in Fabrys?

• Alpha-galactosidaseAlpha-galactosidase

• What accumulates?What accumulates?

• Ceramide TrihexosideCeramide Trihexoside

Page 695: USMLE Step 1 clues 2- 800 slides

What are the CLUES for What are the CLUES for Fabrys?Fabrys?

• X-Linked recessiveX-Linked recessive

• Presents with cataracts as a childPresents with cataracts as a child

• Presents with renal failure as a childPresents with renal failure as a child

Page 696: USMLE Step 1 clues 2- 800 slides

What is missing in Krabbes?What is missing in Krabbes?

• Galactosylceramide B-GalactosidaseGalactosylceramide B-Galactosidase

• What accumulates?What accumulates?

• GalactocerebrosidaseGalactocerebrosidase

• Where?Where?

• BrainBrain

Page 697: USMLE Step 1 clues 2- 800 slides

What are the CLUES for What are the CLUES for Krabbes?Krabbes?

• Early deathEarly death

• Globoid bodiesGloboid bodies– Fat cellsFat cells

Page 698: USMLE Step 1 clues 2- 800 slides

What is missing in Tay What is missing in Tay Sachs?Sachs?

• Hexoseaminidase AHexoseaminidase A

• What accumulates?What accumulates?

• GM2 GangliosideGM2 Ganglioside

Page 699: USMLE Step 1 clues 2- 800 slides

What is the CLUE for Tay What is the CLUE for Tay Sachs?Sachs?

• Ask. JewsAsk. Jews

• Cherry red maculaCherry red macula

• Death by 3Death by 3

Page 700: USMLE Step 1 clues 2- 800 slides

What is missing in What is missing in Sandhoffs?Sandhoffs?

• Hexoseaminadase A & BHexoseaminadase A & B

Page 701: USMLE Step 1 clues 2- 800 slides

What is missing in Hurlers?What is missing in Hurlers?

• IduronidaseIduronidase

Page 702: USMLE Step 1 clues 2- 800 slides

What are the CLUES for What are the CLUES for Hurlers?Hurlers?

• Corneal CloudingCorneal Clouding

• Mental RetardationMental Retardation

Page 703: USMLE Step 1 clues 2- 800 slides

What is missing in Hunters?What is missing in Hunters?

• Iduronate SulfataseIduronate Sulfatase

Page 704: USMLE Step 1 clues 2- 800 slides

What are the CLUES for What are the CLUES for Hunters?Hunters?

• Mild mental retardationMild mental retardation

• No corneal cloudingNo corneal clouding

• Mild form of HurlersMild form of Hurlers

• X-linked recessiveX-linked recessive

Page 705: USMLE Step 1 clues 2- 800 slides

What is missing in Niemann What is missing in Niemann Picks?Picks?

• SpingomyelinaseSpingomyelinase

• What accumulates?What accumulates?

• Spingomyelin Spingomyelin

• CholesterolCholesterol

Page 706: USMLE Step 1 clues 2- 800 slides

What are the CLUES for What are the CLUES for Niemann Picks?Niemann Picks?

• Zebra bodiesZebra bodies

• Cherrry red maculaCherrry red macula

• Die by 3Die by 3

Page 707: USMLE Step 1 clues 2- 800 slides

What is missing in What is missing in Metachromatic Metachromatic Leukodystrophy?Leukodystrophy?• Arylsulfatase AArylsulfatase A

Page 708: USMLE Step 1 clues 2- 800 slides

What is the CLUE for What is the CLUE for Metachromatic Metachromatic Leukodystrophy?Leukodystrophy?• Visual DisturbanceVisual Disturbance

• Presents like MS in 5 to 10 years of Presents like MS in 5 to 10 years of ageage

Page 709: USMLE Step 1 clues 2- 800 slides

What are the Glycogen Storage What are the Glycogen Storage Diseases?Diseases?

• Von GierkesVon Gierkes

• AndersonsAndersons

• CorysCorys

• McCardlesMcCardles

• PompesPompes

• HersHers

Page 710: USMLE Step 1 clues 2- 800 slides

What is deficient in Von What is deficient in Von Gierkes?Gierkes?

• G-6-Pase Deficiency G-6-Pase Deficiency

Page 711: USMLE Step 1 clues 2- 800 slides

What is the CLUE for Von What is the CLUE for Von Gierkes?Gierkes?

• Big LiverBig Liver

• Big KidneyBig Kidney

• Severe hypoglycemiaSevere hypoglycemia

• Can NEVER raise their blood sugarCan NEVER raise their blood sugar

Page 712: USMLE Step 1 clues 2- 800 slides

What is deficient in What is deficient in Andersons?Andersons?

• Branching enzyme deficiency Branching enzyme deficiency

Page 713: USMLE Step 1 clues 2- 800 slides

What is the CLUE for What is the CLUE for Andersons?Andersons?

• Glycogen will be ALL LONG chains on Glycogen will be ALL LONG chains on liver biopsyliver biopsy

Page 714: USMLE Step 1 clues 2- 800 slides

What is missing in Corys?What is missing in Corys?

• Debranching enzymeDebranching enzyme

Page 715: USMLE Step 1 clues 2- 800 slides

What is the CLUE for Corys?What is the CLUE for Corys?

• Glycogen from liver biopsy will be Glycogen from liver biopsy will be ALL SHORT branchesALL SHORT branches

Page 716: USMLE Step 1 clues 2- 800 slides

What is missing in What is missing in McCardles?McCardles?

• Muscle phosporalase Muscle phosporalase

Page 717: USMLE Step 1 clues 2- 800 slides

What is the CLUE for What is the CLUE for McCardles?McCardles?

• Severe muscle cramps when Severe muscle cramps when exercisingexercising

• High CPKHigh CPK

Page 718: USMLE Step 1 clues 2- 800 slides

What is missing in Pompes?What is missing in Pompes?

• Cardiac alpha-1,4 glucocydaseCardiac alpha-1,4 glucocydase

Page 719: USMLE Step 1 clues 2- 800 slides

What is the CLUE for What is the CLUE for Pompes?Pompes?

• Heart problemsHeart problems

• Die earlyDie early

Page 720: USMLE Step 1 clues 2- 800 slides

What is missing in Hers?What is missing in Hers?

• Liver phosphoralaseLiver phosphoralase

Page 721: USMLE Step 1 clues 2- 800 slides

What is the CLUE for Hers?What is the CLUE for Hers?

• Big LiverBig Liver

• NO big kidneyNO big kidney

Page 722: USMLE Step 1 clues 2- 800 slides

Pagets disease is associated Pagets disease is associated with what cancer?with what cancer?

• Intraductal CaIntraductal Ca

Page 723: USMLE Step 1 clues 2- 800 slides

What MUST you rule out with a What MUST you rule out with a decrease AVO2?decrease AVO2?

• AV FistulaAV Fistula

• VasodilationVasodilation

Page 724: USMLE Step 1 clues 2- 800 slides

What diseases have a cherry What diseases have a cherry red macula?red macula?

• Tay SachsTay Sachs

• SandhoffsSandhoffs

• Niemann PickNiemann Pick

Page 725: USMLE Step 1 clues 2- 800 slides

What is translocation 9;22?What is translocation 9;22?

• CMLCML

Page 726: USMLE Step 1 clues 2- 800 slides

What is translocation 11;22?What is translocation 11;22?

• Ewing’s sarcomaEwing’s sarcoma

Page 727: USMLE Step 1 clues 2- 800 slides

What is translocation 8;14?What is translocation 8;14?

• Burketts lymphomaBurketts lymphoma

Page 728: USMLE Step 1 clues 2- 800 slides

What is translocation 14;18?What is translocation 14;18?

• Follicular lymphomaFollicular lymphoma

Page 729: USMLE Step 1 clues 2- 800 slides

What are the causes of What are the causes of restrictive cardiomyopathy?restrictive cardiomyopathy?

• SarcoidSarcoid

• AmyloidAmyloid

• HemochromatosisHemochromatosis

• CancerCancer

• FibrosisFibrosis

• Thanks STAN!!Thanks STAN!!

Page 730: USMLE Step 1 clues 2- 800 slides

What are the CLUES for What are the CLUES for Vasulitis or Intravascular Vasulitis or Intravascular Hemolysis?Hemolysis?• ShistocytesShistocytes

– Burr cellsBurr cells– Helmet cellsHelmet cells

Page 731: USMLE Step 1 clues 2- 800 slides

What is the CLUE for What is the CLUE for Extravascular Hemolysis?Extravascular Hemolysis?

• Splenomagely Splenomagely

Page 732: USMLE Step 1 clues 2- 800 slides

Where is Glucose 6-Pase Where is Glucose 6-Pase present?present?

• AdrenalAdrenal

• LiverLiver

Page 733: USMLE Step 1 clues 2- 800 slides

What is the Heinz body What is the Heinz body CLUE?CLUE?

• G6PDG6PD

Page 734: USMLE Step 1 clues 2- 800 slides

If you see the CLUE basophilic If you see the CLUE basophilic stippling, what should you be stippling, what should you be thinking?thinking?

• Lead poisoningLead poisoning

Page 735: USMLE Step 1 clues 2- 800 slides

What are the Microcytic What are the Microcytic Hypochromic Anemias?Hypochromic Anemias?

• Iron deficiencyIron deficiency

• Anemia of Chronic diseaseAnemia of Chronic disease

• Lead poisoningLead poisoning

• HemoglobinopathyHemoglobinopathy

• Thallasemia’sThallasemia’s

• Sideroblastic anemiaSideroblastic anemia

Page 736: USMLE Step 1 clues 2- 800 slides

What is primary sideroblastic What is primary sideroblastic anemia due to?anemia due to?

• GeneticGenetic

• ADAD

Page 737: USMLE Step 1 clues 2- 800 slides

What is secondary What is secondary Sideroblastic anemia due to?Sideroblastic anemia due to?

• Blood transfusionsBlood transfusions

Page 738: USMLE Step 1 clues 2- 800 slides

What are the Microcytic What are the Microcytic Hyperchromic Anemia’s?Hyperchromic Anemia’s?

• Hereditary SpherocytosisHereditary Spherocytosis

Page 739: USMLE Step 1 clues 2- 800 slides

What are the Normocytic What are the Normocytic Normochromic Anemia’s?Normochromic Anemia’s?

• Acute hemorrhageAcute hemorrhage

• Anemia of Chronic DiseaseAnemia of Chronic Disease

• Hypothyrodism Hypothyrodism – EarlyEarly

• Renal FailureRenal Failure

Page 740: USMLE Step 1 clues 2- 800 slides

What are the Macrocytic What are the Macrocytic anemia’s?anemia’s?

• Folate deficiencyFolate deficiency

• B12 deficiencyB12 deficiency

• ReticulocytosisReticulocytosis

• ETOHETOH

• Hemolytic AnemiasHemolytic Anemias

• Chemo TreatmentChemo Treatment

• AnticonvulsantsAnticonvulsants

• MyelodysplasiaMyelodysplasia

Page 741: USMLE Step 1 clues 2- 800 slides

What are the anticonvulsants What are the anticonvulsants causing a Macrocytic Anemia?causing a Macrocytic Anemia?

• PhenytoinPhenytoin

• EthusuximideEthusuximide

• CarbamyazapineCarbamyazapine

• ValproateValproate

Page 742: USMLE Step 1 clues 2- 800 slides

What anemia is caused by What anemia is caused by blood transfusions?blood transfusions?

• Sideroblastic anemiaSideroblastic anemia

Page 743: USMLE Step 1 clues 2- 800 slides

What is the problem if you see What is the problem if you see Eliptocytes?Eliptocytes?

• Something is wrong with the RBC Something is wrong with the RBC membranemembrane– ExtravascularExtravascular

• Heridatary EllitocytosisHeridatary Ellitocytosis

• Increased RET countIncreased RET count

Page 744: USMLE Step 1 clues 2- 800 slides

What disease do you get if you What disease do you get if you have an EXCESS in Cu+?have an EXCESS in Cu+?

• Wilson’s DiseaseWilson’s Disease

Page 745: USMLE Step 1 clues 2- 800 slides

What is the CLUE for Wilson’s What is the CLUE for Wilson’s Disease?Disease?

• Hepato/Lenticular DegenerationHepato/Lenticular Degeneration

• Kayser Fleishner RingsKayser Fleishner Rings– Copper in eyesCopper in eyes

• Hepato = LiverHepato = Liver

• Lenticular = Movement problemLenticular = Movement problem

Page 746: USMLE Step 1 clues 2- 800 slides

What is Copper needed for?What is Copper needed for?

• Collagen synthesisCollagen synthesis

Page 747: USMLE Step 1 clues 2- 800 slides

What disease manifest with What disease manifest with Cu+ deficiency?Cu+ deficiency?

• Minky Kinky Hair SyndromeMinky Kinky Hair Syndrome

Page 748: USMLE Step 1 clues 2- 800 slides

What are the plasma What are the plasma catecholamines?catecholamines?

• EpinephrineEpinephrine

• NorepinephrineNorepinephrine

• DopamineDopamine

Page 749: USMLE Step 1 clues 2- 800 slides

What are Plasma What are Plasma Catecholamines derived from?Catecholamines derived from?

• TyrosineTyrosine

Page 750: USMLE Step 1 clues 2- 800 slides

A patient with episodic HTN leading to A patient with episodic HTN leading to headache with arrhythmias leading to headache with arrhythmias leading to palpitations most likely is diagnosed palpitations most likely is diagnosed with?with?

• Adrenal PheochromocytomaAdrenal Pheochromocytoma

Page 751: USMLE Step 1 clues 2- 800 slides

What is the MOA for What is the MOA for Fluroquinolones?Fluroquinolones?

• Blocks DNA gyrase (topoisomerase II)Blocks DNA gyrase (topoisomerase II)

• Inhibits p450Inhibits p450

Page 752: USMLE Step 1 clues 2- 800 slides

What do Fluroquinolones What do Fluroquinolones cover?cover?

• All Gram + including staph auerusAll Gram + including staph auerus

• All Gram –All Gram –

• AtypicalsAtypicals

Page 753: USMLE Step 1 clues 2- 800 slides

What are the atypicals?What are the atypicals?

• ChlamydiaChlamydia

• UreoplasmaUreoplasma

• MycoplasmaMycoplasma

• LegionellaLegionella

Page 754: USMLE Step 1 clues 2- 800 slides

What induces What induces Gluconeogensis?Gluconeogensis?

• CortisolCortisol

• EpinephrineEpinephrine

• GlucagonGlucagon

Page 755: USMLE Step 1 clues 2- 800 slides

What is Dermatan Sulfate?What is Dermatan Sulfate?

• Glycosaminoglycan chain that helps Glycosaminoglycan chain that helps form proteoglycansform proteoglycans

Page 756: USMLE Step 1 clues 2- 800 slides

What is Hyaluronic Acid?What is Hyaluronic Acid?

• Glycosaminoglycan chain that helps Glycosaminoglycan chain that helps form proteoglycansform proteoglycans

Page 757: USMLE Step 1 clues 2- 800 slides

What is the MOA of What is the MOA of Methotrexate?Methotrexate?

• Inhibits dihydrofolate reductaseInhibits dihydrofolate reductase

• Inhibits DNA synthesis in the S phase Inhibits DNA synthesis in the S phase of cycle.of cycle.

• Prevents reduction of folic acid Prevents reduction of folic acid needed to produce THFneeded to produce THF

Page 758: USMLE Step 1 clues 2- 800 slides

What are THF derivatives used What are THF derivatives used in?in?

• Purine nucleotide synthesisPurine nucleotide synthesis

• Methylation of dUMP to for dTMPMethylation of dUMP to for dTMP

Page 759: USMLE Step 1 clues 2- 800 slides

What happens when What happens when Dihydrofolate Reductase is Dihydrofolate Reductase is inhibited?inhibited?• Obstructs one carbon methylation Obstructs one carbon methylation

which deprives DNA polymerase of which deprives DNA polymerase of essential substratesessential substrates

Page 760: USMLE Step 1 clues 2- 800 slides

What diseases present as What diseases present as Failure to Thrive?Failure to Thrive?

• CFCF

• GalactosemiaGalactosemia

Page 761: USMLE Step 1 clues 2- 800 slides

In general, what should you In general, what should you always associate Hemolytic always associate Hemolytic Anemia’s with?Anemia’s with?

• Defects in GlycolysisDefects in Glycolysis

• Defects in Hexose Monophosphate Defects in Hexose Monophosphate ShuntShunt

Page 762: USMLE Step 1 clues 2- 800 slides

What is required for the What is required for the conversion of Homocysteine to conversion of Homocysteine to Methionine?Methionine?

• B12B12

Page 763: USMLE Step 1 clues 2- 800 slides

What is required for the What is required for the conversion of methylmalonyl conversion of methylmalonyl CoA to Succinyl CoA?CoA to Succinyl CoA?

• B12B12

Page 764: USMLE Step 1 clues 2- 800 slides

What is required for the What is required for the degradation of cystathionine?degradation of cystathionine?

• Vitamin B6Vitamin B6

Page 765: USMLE Step 1 clues 2- 800 slides

What does the hydroxalation of What does the hydroxalation of Purines require?Purines require?

• Vitamin CVitamin C

Page 766: USMLE Step 1 clues 2- 800 slides

When does the carboxyalation When does the carboxyalation of Glutamic acid occurs and of Glutamic acid occurs and what is required for this what is required for this carboxyalation?carboxyalation?

• Occurs in the synthesis of Blood Occurs in the synthesis of Blood Clotting factorsClotting factors

• Requires Vitamin KRequires Vitamin K

Page 767: USMLE Step 1 clues 2- 800 slides

Decarboxylation of alpha-Decarboxylation of alpha-ketoacids requires what?ketoacids requires what?

• ThiamineThiamine

Page 768: USMLE Step 1 clues 2- 800 slides

Synthesis of 1,25-d-Synthesis of 1,25-d-hydroxycholecalciferol requires hydroxycholecalciferol requires what?what?

• Vitamin DVitamin D

Page 769: USMLE Step 1 clues 2- 800 slides

Synthesis of Rhodopsin Synthesis of Rhodopsin requires what?requires what?

• Vitamin AVitamin A

Page 770: USMLE Step 1 clues 2- 800 slides

Pyruvate Decarboxylase Pyruvate Decarboxylase requires what as a cofactor?requires what as a cofactor?

• ThiamineThiamine

Page 771: USMLE Step 1 clues 2- 800 slides

What is CN1?What is CN1?

• OlfactoryOlfactory• What is its function?What is its function?• Sensory for smellSensory for smell• What if lesioned?What if lesioned?• AnosmiaAnosmia• Where does it Exit/Enter the Cranium?Where does it Exit/Enter the Cranium?• Cribriform plateCribriform plate• What does it innervate?What does it innervate?• Nasal Cavity Nasal Cavity

Page 772: USMLE Step 1 clues 2- 800 slides

What is CN2?What is CN2?

• OpticOptic

• What is its function?What is its function?

• Sensory for sightSensory for sight

• What if lesioned?What if lesioned?

• Anopsia Anopsia – Visual field defectVisual field defect

• Loss of light reflex with CN IIILoss of light reflex with CN III

• Only nerve affected by MSOnly nerve affected by MS

Page 773: USMLE Step 1 clues 2- 800 slides

Cont. CN2Cont. CN2

• Where does it Exit/Enter the Where does it Exit/Enter the Cranium?Cranium?

• Optic CanalOptic Canal

• What does it innervate?What does it innervate?

• OrbitOrbit

Page 774: USMLE Step 1 clues 2- 800 slides

What is CN3?What is CN3?

• OcculomotorOcculomotor

• What are the functions?What are the functions?

• MotorMotor

• Moves the eyeball in ALL directionsMoves the eyeball in ALL directions

• Adduction Most important action (MR)Adduction Most important action (MR)

• Constricts the pupil (Spincter Pupillae)Constricts the pupil (Spincter Pupillae)

• Accomodates (Cililary Muscle)Accomodates (Cililary Muscle)

• Raises eyelid (Levator Palpebrae)Raises eyelid (Levator Palpebrae)

Page 775: USMLE Step 1 clues 2- 800 slides

Cont. CN 3Cont. CN 3

• What if lesioned?What if lesioned?

• DiplopiaDiplopia

• Loss of parallel gazeLoss of parallel gaze

• Dilated pupilDilated pupil

• Loss of light reflexLoss of light reflex

• Loss of near responseLoss of near response

• PtosisPtosis

Page 776: USMLE Step 1 clues 2- 800 slides

Cont. CN 3Cont. CN 3

• Where does it Exit/Enter the Where does it Exit/Enter the Cranium?Cranium?

• Superior Orbital FissureSuperior Orbital Fissure

• What does it innervate?What does it innervate?

• OrbitOrbit

Page 777: USMLE Step 1 clues 2- 800 slides

What is CN 4?What is CN 4?

• TrochlearTrochlear

• What is its function?What is its function?

• MotorMotor

• Superior ObliqueSuperior Oblique

• Depresses and abducts the eyeballsDepresses and abducts the eyeballs

• IntortsIntorts

Page 778: USMLE Step 1 clues 2- 800 slides

Cont. CN 4Cont. CN 4

• What if lesioned?What if lesioned?

• Weakness looking down w/ adducted eyeWeakness looking down w/ adducted eye

• Trouble going down stairsTrouble going down stairs

• Head tilts away from lesioned sideHead tilts away from lesioned side

• Where does it Exit/Enter the Cranium?Where does it Exit/Enter the Cranium?

• Superior Orbital FissureSuperior Orbital Fissure

Page 779: USMLE Step 1 clues 2- 800 slides

Cont. CN 4Cont. CN 4

• What does it innervate?What does it innervate?

• OrbitOrbit

Page 780: USMLE Step 1 clues 2- 800 slides

What is CN 5?What is CN 5?

• TrigeminalTrigeminal

Page 781: USMLE Step 1 clues 2- 800 slides

What are the different What are the different branches of CN 5?branches of CN 5?

• V1?V1?

• OpthalmicOpthalmic

• V2?V2?

• MaxillaryMaxillary

• V3?V3?

• MandibularMandibular

Page 782: USMLE Step 1 clues 2- 800 slides

What is the function of CN V1?What is the function of CN V1?

• MixedMixed• General sensation (touch, pain, temperature) of General sensation (touch, pain, temperature) of

the forehead, scalp, & corneathe forehead, scalp, & cornea

• What if lesioned?What if lesioned?• Loss of general sensation of the forehead/scalpLoss of general sensation of the forehead/scalp• Loss of blink reflex w/ VIILoss of blink reflex w/ VII

• Where does it Exit/Enter the Cranium?Where does it Exit/Enter the Cranium?• Superior orbital FissureSuperior orbital Fissure

– Ophthalmic divisionOphthalmic division

Page 783: USMLE Step 1 clues 2- 800 slides

Cont. CN V1Cont. CN V1

• What does it innervate?What does it innervate?

• OrbitOrbit

• ScalpScalp

Page 784: USMLE Step 1 clues 2- 800 slides

What is the function of CN What is the function of CN V2?V2?• MixedMixed

• General sensation of Palat, Nasal General sensation of Palat, Nasal cavity, Maxillary face, and Maxillary cavity, Maxillary face, and Maxillary teethteeth

• What if lesioned?What if lesioned?

• Loss of general sensation in skin over Loss of general sensation in skin over maxilla & maxillary teethmaxilla & maxillary teeth

Page 785: USMLE Step 1 clues 2- 800 slides

Cont. CN V2Cont. CN V2

• Where does it Exit/Enter the Cranium?Where does it Exit/Enter the Cranium?

• Foramen RotundumForamen Rotundum

• What does it innervate?What does it innervate?

• PterygopalatinePterygopalatine– Leaves by openings to face, oral & nasal Leaves by openings to face, oral & nasal

cavitycavity

Page 786: USMLE Step 1 clues 2- 800 slides

What is the function of CN What is the function of CN V3?V3?• What is its function?What is its function?

• MixedMixed

• General sensation of anterior 2/3 of tongue, General sensation of anterior 2/3 of tongue, mandibular face & mandibular teethmandibular face & mandibular teeth

• MotorMotor

• Muscles of Mastication and anterior belly of Muscles of Mastication and anterior belly of digastric, mylohyoid, tensor tympani, digastric, mylohyoid, tensor tympani, tensor palatitensor palati

Page 787: USMLE Step 1 clues 2- 800 slides

Cont. CN V3Cont. CN V3

• What if lesioned?What if lesioned?

• Loss of general sensation in skin over Loss of general sensation in skin over mandible, mandibular teeth, tongue, mandible, mandibular teeth, tongue, weakness in chewingweakness in chewing

• Jaw deviation to weak sideJaw deviation to weak side

• Trigeminal neuralgiaTrigeminal neuralgia– Intractable pain in V2 or V3 territoryIntractable pain in V2 or V3 territory

Page 788: USMLE Step 1 clues 2- 800 slides

Cont. CN V3Cont. CN V3

• Where does it Exit/Enter the Where does it Exit/Enter the Cranium?Cranium?

• Foramen Ovale Foramen Ovale

• What does it innervate?What does it innervate?

• Infratemporal FossaInfratemporal Fossa

Page 789: USMLE Step 1 clues 2- 800 slides

What is CN VI?What is CN VI?

• AbducensAbducens

• What is its function?What is its function?

• MotorMotor

• Lateral rectusLateral rectus– Abducts eyeAbducts eye

Page 790: USMLE Step 1 clues 2- 800 slides

Cont. CN VICont. CN VI

• What if lesioned?What if lesioned?• DiplopiaDiplopia

– Internal strabismusInternal strabismus• Loss of parallel gazeLoss of parallel gaze

– PseudoptosisPseudoptosis

• Where does it Exit/Enter the Cranium?Where does it Exit/Enter the Cranium?• Superior orbital fissureSuperior orbital fissure

• What does it innervate?What does it innervate?• OrbitOrbit

Page 791: USMLE Step 1 clues 2- 800 slides

What is CN VII?What is CN VII?

• FacialFacial• What is its function?What is its function?• MixedMixed• To muscles of facial expressionTo muscles of facial expression• Posterior belly of diagastricPosterior belly of diagastric• Stylohyoid & StapediusStylohyoid & Stapedius• Tastes anterior 2/3 of tongue/palateTastes anterior 2/3 of tongue/palate• Salivates (submandibular & sublingual glands)Salivates (submandibular & sublingual glands)• Tears (Lacrimal glands)Tears (Lacrimal glands)• Makes mucous (nasal & palatine glands)Makes mucous (nasal & palatine glands)

Page 792: USMLE Step 1 clues 2- 800 slides

Cont. CN VIICont. CN VII

• What if lesioned?What if lesioned?• Corner of mouth droopsCorner of mouth droops• Can’t close eyeCan’t close eye• Can’t wrinkle foreheadCan’t wrinkle forehead• Loss of blink reflexLoss of blink reflex• HypeacusisHypeacusis• Loss or alteration of taste (ageusia)Loss or alteration of taste (ageusia)• Eye dry and redEye dry and red• Bell PalsyBell Palsy

– Lesion of nerve in facial canalLesion of nerve in facial canal

Page 793: USMLE Step 1 clues 2- 800 slides

Cont. CN VIICont. CN VII

• Where does it Exit/Enter the Cranium?Where does it Exit/Enter the Cranium?• Internal Auditory meatusInternal Auditory meatus

• What does it innervate?What does it innervate?• FaceFace• Nasal & oral cavity Nasal & oral cavity

– Branches leave skull in stylomastoid Branches leave skull in stylomastoid foramen, petrotympanic fissure, or Hiatus of foramen, petrotympanic fissure, or Hiatus of facial canalfacial canal

Page 794: USMLE Step 1 clues 2- 800 slides

What is CN VIII?What is CN VIII?

• VestibulocochlearVestibulocochlear

• What is its function?What is its function?

• SensorySensory

• HearsHears

• Linear acceleration (Gravity)Linear acceleration (Gravity)

• Angular acceleration (Head Turning)Angular acceleration (Head Turning)

Page 795: USMLE Step 1 clues 2- 800 slides

Cont. CN VIIICont. CN VIII

• What if lesioned?What if lesioned?• Loss of BalanceLoss of Balance• NystagmusNystagmus

• Where does it Exit/Enter the Cranium?Where does it Exit/Enter the Cranium?• Internal Auditory MeatusInternal Auditory Meatus

• What does it innervate?What does it innervate?• Inner earInner ear

Page 796: USMLE Step 1 clues 2- 800 slides

What is CN IX?What is CN IX?

• GlossopharyngealGlossopharyngeal

• What is its function?What is its function?

• MixedMixed

• Sense PharynxSense Pharynx

• Carotid sinus/bodyCarotid sinus/body

• Salivates (parotid glands)Salivates (parotid glands)

• Tastes and senses posterior 1/3 of tongueTastes and senses posterior 1/3 of tongue

Page 797: USMLE Step 1 clues 2- 800 slides

Cont. CN IXCont. CN IX

• What is its function?What is its function?

• To one muscle only (stylopharyngeus)To one muscle only (stylopharyngeus)

• What if lesioned?What if lesioned?

• Loss of Gag Reflex with XLoss of Gag Reflex with X

• Where does it Exit/Enter the Cranium?Where does it Exit/Enter the Cranium?

• Jugular ForamenJugular Foramen

Page 798: USMLE Step 1 clues 2- 800 slides

Cont. CN IXCont. CN IX

• What does it innervate?What does it innervate?

• NeckNeck

• Pharynx/TonguePharynx/Tongue

Page 799: USMLE Step 1 clues 2- 800 slides

What is CN X?What is CN X?

• VagusVagus• What is its function?What is its function?• MixedMixed• To muscles of palate & pharynx for swallowing To muscles of palate & pharynx for swallowing

except tensor palate (V) & Stylopharynegeus (IX)except tensor palate (V) & Stylopharynegeus (IX)• To all muscles of Larynx (phonates)To all muscles of Larynx (phonates)• Senses Larynx & LaryngopharynxSenses Larynx & Laryngopharynx• Senses Larynx & GI tractSenses Larynx & GI tract• To GI tract smooth muscle & glands in forgut & To GI tract smooth muscle & glands in forgut &

midgutmidgut

Page 800: USMLE Step 1 clues 2- 800 slides

Cont. CN XCont. CN X

• What if lesioned?What if lesioned?• Nasal speechNasal speech• Nasal regurgitationNasal regurgitation• DysphagiaDysphagia• Palate dropPalate drop• Uvula points away from pathologyUvula points away from pathology• Hoarseness/fixed vocal cordHoarseness/fixed vocal cord• Loss of gag reflex w/ IXLoss of gag reflex w/ IX• Loss of cough reflexLoss of cough reflex

Page 801: USMLE Step 1 clues 2- 800 slides

Cont. CN XCont. CN X

• Where does it Exit/Enter the Cranium?Where does it Exit/Enter the Cranium?

• Jugular ForamenJugular Foramen

• What does it innervate?What does it innervate?

• NeckNeck

• Pharynx/LarynzPharynx/Larynz

• Thorax/AbdoThorax/Abdo

Page 802: USMLE Step 1 clues 2- 800 slides

CN X – Sympathetics to CN X – Sympathetics to HeadHead

• What is its function?What is its function?

• MotorMotor

• Raises eyelid (superior tarsal muscle)Raises eyelid (superior tarsal muscle)

• Dilates pupilDilates pupil

• Innervates sweat glands of face & Innervates sweat glands of face & scalpscalp

• Constricts blood vessels in headConstricts blood vessels in head

Page 803: USMLE Step 1 clues 2- 800 slides

Cont. CN XCont. CN X

• What if lesioned?What if lesioned?

• Horner syndromeHorner syndrome– Eyelid droop (ptosis)Eyelid droop (ptosis)– Constricted pupil (miosis)Constricted pupil (miosis)– Loss of sweating (anhydrosis)Loss of sweating (anhydrosis)– Flushed faceFlushed face

Page 804: USMLE Step 1 clues 2- 800 slides

Cont. CN XCont. CN X

• Where does it Exit/Enter the Cranium?Where does it Exit/Enter the Cranium?

• Carotid canal on internal carotid arteryCarotid canal on internal carotid artery

• What does it innervate?What does it innervate?

• OrbitOrbit

• Face Face

• ScalpScalp

Page 805: USMLE Step 1 clues 2- 800 slides

What is CN XI?What is CN XI?

• AccessoryAccessory

• What is its function?What is its function?

• Turns head to opposite sideTurns head to opposite side– sternocleidomastoidsternocleidomastoid

• Elevates & Rotates scapulaElevates & Rotates scapula– TrapeziusTrapezius

Page 806: USMLE Step 1 clues 2- 800 slides

Cont. CN XICont. CN XI

• What if lesioned?What if lesioned?• Weakness turning head to opposite sideWeakness turning head to opposite side• Shoulder droopShoulder droop

• Where does it Exit/Enter the Cranium?Where does it Exit/Enter the Cranium?• Jugular ForamenJugular Foramen

• What does it innervate?What does it innervate?• NeckNeck

Page 807: USMLE Step 1 clues 2- 800 slides

What is CN XII?What is CN XII?

• HypoglossalHypoglossal

• What is its function?What is its function?

• Moves tongueMoves tongue

• What if lesioned?What if lesioned?

• Tongue points toward pathology on Tongue points toward pathology on protrusionprotrusion

Page 808: USMLE Step 1 clues 2- 800 slides

What is CN XII?What is CN XII?

• Where does it Exit/Enter the Where does it Exit/Enter the Cranium?Cranium?

• Hypoglossal CanalHypoglossal Canal

• What does it innervate?What does it innervate?

• TongueTongue

Page 809: USMLE Step 1 clues 2- 800 slides

What are the muscles of What are the muscles of mastication?mastication?

• TemporalisTemporalis

• MasseterMasseter

• Medial PterygoidsMedial Pterygoids

• Lateral PterygoidsLateral Pterygoids

Page 810: USMLE Step 1 clues 2- 800 slides

What part of the brain deals What part of the brain deals with problem solving?with problem solving?

• Frontal LobeFrontal Lobe

Page 811: USMLE Step 1 clues 2- 800 slides

What happens if there is a What happens if there is a lesion to the Optic nerve?lesion to the Optic nerve?

• Unilateral BlindnessUnilateral Blindness

Page 812: USMLE Step 1 clues 2- 800 slides

What happens if there is a What happens if there is a lesion to the Optic Chiasm?lesion to the Optic Chiasm?

• Bitemporal HemianopiaBitemporal Hemianopia

Page 813: USMLE Step 1 clues 2- 800 slides

What is a Berry Aneurysm due What is a Berry Aneurysm due to in the Circle of Willis?to in the Circle of Willis?

• SUBARACHNOID Hemorrhage SUBARACHNOID Hemorrhage

Page 814: USMLE Step 1 clues 2- 800 slides

What is a CLUE for What is a CLUE for SUBARACHNOID Hemorrhage?SUBARACHNOID Hemorrhage?

• The worse headache of my lifeThe worse headache of my life

Page 815: USMLE Step 1 clues 2- 800 slides

What effect does Pernicious What effect does Pernicious Anemia have on the nervous Anemia have on the nervous system?system?

• Causes degeneration of the posterior Causes degeneration of the posterior columnscolumns

• Causes degeneration of the CSTCauses degeneration of the CST

• Loss of proprioceptionLoss of proprioception

• Upper motor neuron defectUpper motor neuron defect

Page 816: USMLE Step 1 clues 2- 800 slides

What does the diencephalon What does the diencephalon originate from?originate from?

• ForebrainForebrain

Page 817: USMLE Step 1 clues 2- 800 slides

What originates from the What originates from the Diencephalon?Diencephalon?

• ThalamusThalamus

• 33rdrd Ventricle Ventricle

Page 818: USMLE Step 1 clues 2- 800 slides

Where does the Telencephalon Where does the Telencephalon originate from?originate from?

• Forebrain Forebrain – ProsencephalonProsencephalon

Page 819: USMLE Step 1 clues 2- 800 slides

What originates from the What originates from the Telencephalon?Telencephalon?

• Cerebral HemispheresCerebral Hemispheres

• Lateral VentriclesLateral Ventricles

Page 820: USMLE Step 1 clues 2- 800 slides

What originates from the What originates from the Mesencephalon?Mesencephalon?

• MidbrainMidbrain

• AqueductAqueduct

Page 821: USMLE Step 1 clues 2- 800 slides

What originates from the What originates from the Hindbrain?Hindbrain?

• MetencephalonMetencephalon

• MyerencephalonMyerencephalon

Page 822: USMLE Step 1 clues 2- 800 slides

What originates from the What originates from the Metencephalon?Metencephalon?

• PonsPons

• CerebrumCerebrum

Page 823: USMLE Step 1 clues 2- 800 slides

What originates from the What originates from the Myerencephalon?Myerencephalon?

• MedullaMedulla

Page 824: USMLE Step 1 clues 2- 800 slides

What does the Jugular Foramen What does the Jugular Foramen contain?contain?

• CN IXCN IX

• CN XCN X

• CN XICN XI

• Internal Jugular VeinInternal Jugular Vein

• Spinal accessory nervesSpinal accessory nerves

Page 825: USMLE Step 1 clues 2- 800 slides

What does the Foramen What does the Foramen Spinosum contain?Spinosum contain?

• Middle menningeal arteryMiddle menningeal artery– Branch of the maxillary arteryBranch of the maxillary artery

Page 826: USMLE Step 1 clues 2- 800 slides

What does the Foramen Ovale What does the Foramen Ovale contain?contain?

• CN V3CN V3

Page 827: USMLE Step 1 clues 2- 800 slides

What does the Foramen What does the Foramen Magnum contain?Magnum contain?

• Vertebral arteriesVertebral arteries

• Brain stemBrain stem

• Spinal roots of CN XISpinal roots of CN XI

• Spinal cordSpinal cord

Page 828: USMLE Step 1 clues 2- 800 slides

What does the Optic Canal What does the Optic Canal contain?contain?

• Opthalmic arteryOpthalmic artery

• Central retinal veinCentral retinal vein

• CN IICN II

Page 829: USMLE Step 1 clues 2- 800 slides

What does the Hypoglossal What does the Hypoglossal Canal contain?Canal contain?

• CN XIICN XII

• Hypoglossal nerveHypoglossal nerve

Page 830: USMLE Step 1 clues 2- 800 slides

What does the Carotid Canal What does the Carotid Canal contain?contain?

• Internal Carotid arteryInternal Carotid artery

Page 831: USMLE Step 1 clues 2- 800 slides

What does the Posterior What does the Posterior Condylar Canal contain?Condylar Canal contain?

• Large Emissary VeinLarge Emissary Vein

Page 832: USMLE Step 1 clues 2- 800 slides

What are signs of an UPPER What are signs of an UPPER motor neuron damage?motor neuron damage?

• + Babinski sign+ Babinski sign

• Spastic ParalysisSpastic Paralysis

• Hyperactive Deep Tendon ReflexesHyperactive Deep Tendon Reflexes

Page 833: USMLE Step 1 clues 2- 800 slides

What are signs of a lower What are signs of a lower motor neuron defect?motor neuron defect?

• AtrophyAtrophy

• FasciullationsFasciullations

• Flaccid ParalysisFlaccid Paralysis

• Loss of deep tendon reflexesLoss of deep tendon reflexes

Page 834: USMLE Step 1 clues 2- 800 slides

What does it mean to see a What does it mean to see a physis on radiograph?physis on radiograph?

• Means the skeleton is not fully Means the skeleton is not fully maturemature

Page 835: USMLE Step 1 clues 2- 800 slides

When does Physis When does Physis disappear?disappear?

• Once growth is completeOnce growth is complete

Page 836: USMLE Step 1 clues 2- 800 slides

What is a nonunion What is a nonunion fracture?fracture?

• Fracture that does not heal with in 6 Fracture that does not heal with in 6 monthsmonths

Page 837: USMLE Step 1 clues 2- 800 slides

What does Malunion mean?What does Malunion mean?

• A fracture that heals in an Abnorman A fracture that heals in an Abnorman position position

Page 838: USMLE Step 1 clues 2- 800 slides

What is a characteristic feature What is a characteristic feature of cancellous (spongy) bone?of cancellous (spongy) bone?

• TrabeculaeTrabeculae

Page 839: USMLE Step 1 clues 2- 800 slides

Which CN’s control eye Which CN’s control eye movement?movement?

• CN IIICN III– Oculomotor nerveOculomotor nerve

• CN IVCN IV– Trochlear nerveTrochlear nerve

• CN VICN VI– Abducens nerveAbducens nerve

Page 840: USMLE Step 1 clues 2- 800 slides

What CN is responsible for What CN is responsible for turning the head and turning the head and shrugging the shoulders?shrugging the shoulders?

• CN XICN XI– Accessory nerveAccessory nerve

Page 841: USMLE Step 1 clues 2- 800 slides

What CN has sensory fibers for What CN has sensory fibers for Face and Motor fibers for Face and Motor fibers for muscles of mastication? muscles of mastication?

• CN VCN V– Tigeminal NerveTigeminal Nerve

Page 842: USMLE Step 1 clues 2- 800 slides

What CN controls tongue What CN controls tongue movement?movement?

• CN XIICN XII– Hypoglossal nerveHypoglossal nerve

Page 843: USMLE Step 1 clues 2- 800 slides

What CN controls sensory What CN controls sensory fibers for Visonfibers for Vison

• CN IICN II– Optic nerveOptic nerve

Page 844: USMLE Step 1 clues 2- 800 slides

What CN senses fiber for What CN senses fiber for smelling?smelling?

• CN ICN I– OlfactoryOlfactory

Page 845: USMLE Step 1 clues 2- 800 slides

What does the Mesenteric What does the Mesenteric Artery Supply?Artery Supply?

• Distal 1/3 of transverse colonDistal 1/3 of transverse colon

• Descending colonDescending colon

• Sigmoid colonSigmoid colon

• Upper Portion of RectumUpper Portion of Rectum

Page 846: USMLE Step 1 clues 2- 800 slides

What does the Superior What does the Superior Mesenteric Artery Supply?Mesenteric Artery Supply?

• DuodenumDuodenum

• JejunumJejunum

• IleumIleum

• CecumCecum

• AppendixAppendix

• Ascending colonAscending colon

• Proximal 2/3 of transverse colonProximal 2/3 of transverse colon

Page 847: USMLE Step 1 clues 2- 800 slides

What does the common Iliac What does the common Iliac artery supply?artery supply?

• PelvisPelvis

• PerineumPerineum

• LegLeg

Page 848: USMLE Step 1 clues 2- 800 slides

What does the Celiac trunck What does the Celiac trunck give rise to?give rise to?

• Left gastric arteryLeft gastric artery

• Splenic arterySplenic artery

• Common hepatic arteryCommon hepatic artery

Page 849: USMLE Step 1 clues 2- 800 slides

What do the Left. Gastric What do the Left. Gastric Artery, Splenic Artery, & Artery, Splenic Artery, & Common Hepatic artery Common Hepatic artery supply?supply?

• EsophagusaEsophagusa

• StomachStomach

• DuodenumDuodenum

• LiverLiver

• GallbladderGallbladder

• PancreasPancreas

Page 850: USMLE Step 1 clues 2- 800 slides

If there is a lesion in the frontal If there is a lesion in the frontal lobe will you have motor or lobe will you have motor or sensory defects?sensory defects?

• MotorMotor

Page 851: USMLE Step 1 clues 2- 800 slides

If patient has a visual field If patient has a visual field defect with cognitive defect with cognitive Distrubance, what part of the Distrubance, what part of the brain will be affected?brain will be affected?

• Temporal or Partial LobeTemporal or Partial Lobe